1. Trang chủ
  2. » Mẫu Slide

Bo de thi hoc sinh gioi tieng anh lop 12

114 275 1

Đang tải... (xem toàn văn)

Tài liệu hạn chế xem trước, để xem đầy đủ mời bạn chọn Tải xuống

THÔNG TIN TÀI LIỆU

Nội dung

Read the following passage and decide which option A, B, C or D best fits each sentence.. HSG 12- 70.[r]

(1)

Đinh Quang Hòa SDT: 01692783980

Yahoo: Meteor_heaven09

Email: dinhquanghoa209@gmail.com

ĐỀ HỌC SINH GIỎI VÒNG TỈNH 2008-2009

SECTION 1: LISTENING

I Listen to the dialogue between Joyce and a woman and then fill in the each blank with ONE word You will hear the recording two times.

W: So tell me about your (1) _, Joyce J: Well, it’s really small town

W: What is it like there?

J: Oh, I think it’s very (2) _ place W: Really? Why?

J: Well, there is nothing to No good (3) _ No nightlife of any kind W: Uh, that’s too bad But small towns are pretty (4) _ to live in

J: Well, yeah, it is (5) _ cheap And lots of people love it because it’s very (6) _

W: Yeah?

J: Uh-huh It has great (7) _- lots of mountains and rivers, lakes, trees… W: Well, I don’t know, Joyce It (8) _ like a lovely place!

J: Well, yeah, if you like to go hiking in the summer and (9) _ in the winter But, you know, I’m not the (10) _ type! I’m a real city person

II Listen to the letter Circle the words you hear You will hear the recording two times.

(2)

a delicious b happy c new d nice e cold f tall

g warm h thin i long j straight k jealous l small

m big n loud o dangerous p safe q fair

r interesting

s friendly t lucky u sharp

SECTION 2: GRAMMAR AND VOCABULARY:

I Choose the best answer among A, B, C or D to complete each sentence: (10Ms)

1 The technological and economic changes of the 19th century had a marked ……… on

workers

A cause B effect C impact D consequence

2 The government ……… the flood victims with food, clothes and money

A gave B provided C offered D presented

3 More and more people ……… of food poisoning nowadays

A exist B survive C die D starve

4 The first sign of vitamin A disorder is night ………

A loss of sight B lack of vision C invisibility D blindness When you take ice out of the freezer, it ………

A melts B dissolves C softens D disappears

6 There are many ……… of pollution in our modern world

A resources B sources C bases D foundation

7 Many people ……… that natural resources will never be use up

A view B regard C consider D believe

8 Using the computer competently is an important ……… to help one get a good job

A reason B aspect C factor D issue

9 George wants £1000 for his car, but I don’t think it’s ……… as much as that

A worth B cost C valued D priced

10 “Did Jenny say anything about her sister?”-No, she didn’t ……… her at all

A remind B remark C refer D mention

II Choose the underlined part ( A, B, C, or D ) that is incorrect: (15Ms)

1 Families who are enough fortunate to own a historic home may be able to get restoration A B C D

funds from the government

2 The first things a new international student must include renting an apartment,

A B registering for classes, and to get to know the city

C D

3 When parents allow his children to spend many hours watching television, the children

A B are not likely to be physically fit

C D

4 Christianity has become a worldwide religion since it has begun almost two thousand A B C D

years ago

5 As the population of Africa continues to grow, animals will continue to lose theirs natural A B C D

habitat

(3)

A B C D reached the New World

7 The most Americans were killed in World War II than in any other war since the birth of A B C D

the nation

8 Antarctica, which is largely covered by ice, receive hardly any rainfall A B C D

9 Jessica is only an amateur, but she sings sweeter than most professionals A B C D

10 Mrs Adams was surprising that her son and his friend had gone to the mountains to ski A B C D 11 A cure for the common cold, causing by a virus, has not been found

A B C D

12 Amelia Earhart, that was one of the pioneers in aviation, attempted to fly the world in A B C

1937, but she and her plane mysteriously disappeared over the Pacific Ocean D

13 His last test results showed he was the quickest typist, yet he was the less accurate of all the applicants

A B C D 14 That secretary of mine is so efficient that she always amazes myself with her speed A B C D

15 A problem it has caused setbacks and we have been forced to change the final date of A B C

completion if we want to avoid being penalized D

III Supply the suitable form of the words in brackets: (10Ms)

Different (1) _ (conserve) efforts have been made in order to (2) _ (danger) species The Red List – (3) _ (globe) list of endangered and vulnerable animal species – has been introduced to raise people’s awareness of conservation needs Governments have enacted laws to protect wildlife from commercial trade and (4) _(hunt) Another _ (conserve) effort is the development of wildlife habitat (5) _ (reserve) as they are suitable for the (6) _ (survive) of a wide range of species

The current globe extinction crisis is one of the (7) (great) challenges to mankind To preserve the earth’s (8) (value) biodiversity for future generations, the protection of (danger) species and their habitats should be a top (9) _ (prior) for international organizations, (10) (govern) agencies, industry and individuals

IV Put the verb in parentheses in correct tenses: (10Ms)

John has always traveled a lot In fact, he was only two years old when he first flew to the US His mother is Italian and his father is American John was born in France, but his parents 1

(meet) in Cologne, Germany after they (live)………… there for five

(4)

He lives in New York now, but (visit)…….……… his parents for the last few weeks He really enjoys living in New York, but he also loves coming to visit his parents at least once a year This year he (fly) over 5,000 miles for his job He has been working for Jackson & Co for almost two years now He (be)……… … … pretty sure that he 8

(work) for them next year as well His job requires a lot of travel In fact, by the

end of this year, he (travel)… ……….…… over 120,000 miles! His next journey will be to Australia He really doesn’t like going to Australia because it is so far This time he 10

(fly) from Paris after a meeting with the company’s French partner He will

have been sitting for over 18 hours by the time he arrives!

V Choose the word (A, B, C or D) that best completes each sentence: (25Ms) Do you know _?

A who how many people go on Sundays to church B who go to church on Sundays how many people C how many people who go on Sundays to church D how many people who go to church on Sundays We watch the cat the tree

A climbed B climb C had climbed D was climbing

3 If we had known your new address, we to see you

A came B will come C would come D would have come

4 You may borrow as many books as you like, provided you show them to is at the desk

A whoever B who C whom D which

5 He looked forward to his first pay packet

A receive B have received C be receiving D receiving Carol refused; , her answer was “no”

A in other words B otherwise C words for words D however

7 If you see Tom _ you mind _ him to get in touch with me? A will / reminding B will / to remind C would / reminding D would / to remind _ I hear that song, I think of you

A Whatever B Forever C Whenever D However

9 “Let’s go dancing, _?” – “Yes, let’s.”

A will we B don’t we C we D shall we

10 I wish I _ all about this matter a week ago

A knew B know C had known D B & C are correct 11 He was _ he could not wake up

A very tired that B such tired that C too tired that D so tired that 12 Joan asked _

A if there was coffee B there was coffee C was there coffee D where was the coffee 13 She didn’t know _ to have fish or chicken for lunch

A both B whether C neither D as

14 For lunch, you may have _ fish or chicken

A both B neither C not only D either

15 I suppose there’s a lot _ between now and the first show

A to be done B done C we D to have done

16 I my house That is why there is all this mess

A had – paint B have – paint C had had - paint D am having – painted 17 I warned him _

A to be not late B don’t be late C not to be late D he is not late

18 I should have _ my hair cut weeks ago, but just don’t seem to have had time

(5)

A let B made C had D gotten

19 I could not cut the grass because the machine _ a few days previously A broke down B has been broken C had broken down D breaks down

20 If you want to develop inner tranquility, you have to stop _ by every little thing that happens

A bothering B being bothered C to bother D to be bothered 21 He was made _ for two hours

A to wait B wait C waiting D waited

22 My boss is angry with me I didn’t all the work that I _ last week

A should B should have done

C must have done D might have done

23 Ann is not at home She’s to dinner

A been B gone C went D being

24 “Where are my jeans?” “They _ at the moment Sorry.”

A are washing B were washed C are washed D are being washed 25 The film was There was so much blood in it

A horrify B horrified C horrifying D being horrified

SECTION 3: READING

I Rearrange the following five paragraphs.

A One should be wary, however, of assuming that silent reading came about simply because reading aloud is a distraction to others Examination that factors related to the historical development of silent reading reveals that it became the usual mode of reading for most adult reading tasks mainly because the tasks themselves changed in character

B By the end of the century students were being recommended to adopt attitudes to books and to use skills in reading them which were inappropriate, if not impossible, for the oral reader The social, cultural, and technological changes in the century had greatly altered what the term “reading” implied

C Reading to oneself is a modern activity which was almost unknown to the scholars of the classical and medieval worlds, while during the fifteen century the term “reading” undoubtedly meant reading aloud Only during the nineteen century did silent reading become commonplace

D Towards the end of the century there was still considerable argument over whether books should be used for information or treated respectfully, and over whether the reading of material such as newspapers was in some way mentally weakening Indeed this argument remains with us still in education However, whatever its virtues, the old shared literary culture had gone and was replaced by the printed mass media on the one hand and by books and periodicals for a specialized readership on the other

E The last century saw a steady gradual increase in literacy, and thus in the number of readers As readers increased, so the number of potential listeners declined, and thus there was some reduction in the need to read aloud As reading for the benefit of listeners grew less common, so came the flourishing of reading as a private activity in such public places as libraries, railway carriages and offices, where reading aloud would cause distraction to other readers

Your answers:

1 -4 -

(6)

II Read the passage and choose the best answer to complete it.

An unlucky day

Helen packed a small suitcase, said goodbye to her mother and hurried out of the house to (1) to the station There was no one (2) waiting at the bus stop, so it looked as if a bus had just left Helen looked at her watch anxiously; it was already two o’clock Her train left at two thirty, and since it would (3) at least twenty minutes to reach the station, she did not have much time to (4) , even if a bus came along at once

Just then a taxi came slowly down the road Helen knew that the (5) to the station was at least two dollars, which was more than she could (6) _, but she quickly made her (7) _ that it would be well (8) _ the extra expense in order to be sure of catching the train So she stopped the taxi and got in she told the driver that she had to catch the train which left at half past two The man nodded and said that he would take a short (9) _ to get her to the station in good time

All went well until, just as they were coming out of a side street into the (10) _ that lead to the station, the taxi ran (11) _ a car There was a loud crash and Helen was thrown (12) _ so violently that she hit her head on the front seat Both drivers got out and began (13) _ each other Helen got out as (14) _ to ask them to stop quarrelling, but (15) _ took any notice of her at all

1 A go on bus B get the bus C drive taxi D catch the bus

2 A else B other C person D more

3 A be B take C waste D lose

4 A spend B wait for C spare D go

5 A cost B expense C fare D money

6 A pay B afford C give D have

7 A decision B mind C head D own

8 A cheap B done C for D worth

9 A time B way C cut D road

10 A main road B main side street C alley D train

11 A off B up C for D into

12 A forward B over C up D hard

13 A shouting B to shout at C to shout for D shouting to

14 A well B soon C much D if

15 A none B no one C neither D both of them

III Read the two passages and choose the best answer to the questions or to complete the statements

PASSAGE 1:

A pilot cannot fly by sight alone In many conditions, such as flying at night and landing in dense fog, a pilot must use radar, an alternative way of navigating Since human eyes are not very good at determining speeds of approaching objects, radar can show a pilot how fast nearby planes are moving The basic principle of radar is exemplified by what happens when one

shouts in a cave The echo of the sounds against the walls helps a person determine the size of

(7)

is essential for air traffic control, tracking ships at sea, and for tracking weather systems and storms

1 What is the main topic of this passage?

A The nature of radar B History of radar C Alternatives to radar D Types of ranging

2 According to the passage, what can radar detect besides location of objects?

A Shape B Size C Speed D Weight

3 The word “exemplified” in the passage can be replaced by _

A “specified” B “resembled” C “illustrated” D “justified” The word “shouts” in the passage most closely means _

A “exclaims” B “yells” C “shoots” D “whispers”

5 Which of the following words best describes the tone of this passage?

A argumentative B explanatory C humorous D imaginative According to the passage, the distance between a radar set and an object can be determined by _

A the time it takes for a burst of radio waves to produce echoes when the waves bounce off the object

B the term “ranging” used for detection of the distance between an object and the radar set C the time it takes for the radio waves to produce echoes and bounce off the object

D the time it takes for the echoes produced by the radio waves to return to the radar set Which type of waves does radar use?

A tidal B sound C heat D radio

8 The word “tracking” in the passage most closely means _

A sending B searching for C ranging D repairing Which of the following would most likely be the topic of the next paragraph?

A A history of flying B Other uses of radar C The technology used by pilots D Uses of some technology 10 What might be inferred about radar?

A It takes the place of a radio

B It has improved navigational safety

C It was developed from a study of sound waves D It gave birth to the invention of the airplane

PASSAGE 2:

Today’s cars are smaller, safer, cleaner, and more economical than their predecessors, but the car of the future will be far more pollution-free than those on the road today Several new types of automobile engines have already been developed than run on alternative sources of power, such as electricity, compressed natural gas, methanol, steam, hydrogen, and propane Electricity, however, is the only zero-emission option presently available

Although electric vehicles will not be truly practical until a powerful, compact battery or other dependable source of current is available, transport experts foresee a new assortment of electric vehicles entering everyday life: shorter-range commuter electric cars, three-wheeled neighborhood cars, electric delivery vans, bikes and trolleys

(8)

the use of electric vehicles, the most convenient parking in transportation centers might be reserved for electric cars

Planners foresee electric shuttle buses, trains, buses and neighborhood vehicles all meeting at transit centers that would have facilities for charging and renting Commuters will be able to rent a variety of electric cars to suit their needs: light trucks, one-person three-wheelers, small cars, or electric/gasoline hybrid cars for longer trips, which will no doubt take place on automated freeways capable of handling five times the number of vehicles that can be carried by freeway today

1 The following electrical vehicles are all mentioned in the passage EXCEPT

A vans B trains C planes D trolleys

2 The author’s purpose in the passage is to A criticize conventional vehicles B support the invention of electric cars

C narrate a story about alternative energy vehicles

D describe the possibilities for transportation in the future The passage would most likely be followed by details about

A automated freeways B pollution restrictions in the future C the neighborhood of the future D electric shuttle buses

4 The word “compact” in the second paragraph is closest in meaning to

A long-range B inexpensive C concentrated D squared In the second paragraph, the author implies that

A a dependable source of electric energy will eventually be developed B everyday life will stay much the same in the future

C a single electric vehicle will eventually replace several modes of transportation D electric vehicles are not practical for the future

6 According to the passage, public parking lots of the future will be

A more convenient than they are today B equipped with charging devices C much larger than they are today D as common as today’s gas stations 7 The word “charging” in this passage refers to

A electricity B credit cards C aggression D lightning The word “foresee” in this passage could best be replaced with

A count on B invent C imagine D rely on

9 The word “commuters” in paragraph refers to

A daily travelers B visitors C cab drivers D shoppers 10 The word “hybrid” in paragraph is closest in meaning to

A combination B hazardous C futuristic D automated

SECTION 4: WRITING

I Rewrite the sentence as similar in meaning as possible: (20Ms) It is cheaper to go by train than by car

- Going by car ……… We must go now because our train leaves at A.M

- It’s ………… ……… Although he made great efforts, he failed to win the race

- In spite ……… Since she was careless, Mary lost her job

- Because of ……… ……… Workers are building a new factory

(9)

6 Who broke the window?

- By ……… “What’s your job?” Mary asked Tom

- Mary wanted to know ……… The bust couldn’t run because of the fog

- The fog ……… ……… Tim will be eighteen next week

- It is Tim’s ……… 10 Peter said to Paul, “I didn’t use your mobile phone! Someone else did, not me”

- Peter denied ……… II Write a report of about 150 words describing the information given in the chart Choose only ONE chart (A or B) to write (20Ms)

A The chart below illustrates heart attacks by age and gender in the USA.

B The chart below shows the amount of time an average student spends on various subjects Study the chart carefully and comment on the proportion of time they spend on these subjects

…… ……… ………

(10)

……… ……… ……… ……… ……… ……… ……… ……… ……… ……… ……… ……… ……… ……… ……… ……… ……… ……… ……… ……… ……… ……… ……… ……… ……… ……… ……… ……… ……… ……… ………

III Make all the changes and additions necessary, to produce from the following sets of words or phrases, sentences that together make a complete letter: (20Ms)

Dear Mary,

1 Thank you/ much/ your letter/ arrive/ few days ago

- ……… It/ be lovely/ hear/ you

- ……… I/ be sorry/ I not write/ such/ long time/ but I/ be/ very busy

- ……… As you know/ we buy/ new house/ May

- ……… ……… It/ be/ very bad condition/ and it need/ a lot/ work

- ……… We finish/ most/ it now/ and it look/ very nice

- ……… John and I/ decide/ give/ house – warming party/ October 2nd.

- ……… You thinks/ you/ able/ come?

(11)

9 Please give me/ ring/ let/ know/ you/ make it

- ……… 10 I/ really/ look forward/ see you again

- ……… Love,

Bob

THE END

(12)

Đáp án 2008-2009:

SECTION 1: LISTENING

I Listen to the dialogue between Joyce and a woman and then fill in the each blank with ONE word You will hear the recording two times.

(1) hometown (2) boring (3) restaurants (4) inexpensive (5) fairly

(6) pretty (7) scenery (8) sounds (9) skiing (10) outdoors

(13)

II Listen to the letter Circle the words you hear You will hear the recording two times. a, b, c, d, f, h, i, j, l, m, n, p, r, s, t

Dear Gramma Lulu,

Thanks so much for the cookies They’re delicious I’m happy here at Oregon State My roommate, Yoko, is from Japan He’s a new student, too She’s very nice She’s tall and thin Her hair is long and straight

We have a small apartment and a big dog Our dog’s name is Rocky He’s big and loud We’re safe with Rocky around

My classes are interesting and my teachers are friendly I’m lucky to be here I hope you’re fine Please write

(14)

SECTION 2: GRAMMAR AND VOCABULARY:

I Choose the best answer among A, B, C or D to complete each sentence: C B C D A B D C A 10 D

II Choose the underlined part ( A, B, C, or D ) that is incorrect. B D A C D C A B C 10 A 11 B 12 A 13 A 14 C 15 A III Supply the suitable form of the words in brackets: conservation endangered global overhunting reserves survival greatest valuable priority 10 governmental IV Put the verb in parentheses in correct tenses.

1 had met

2 had been living sat

4 travels is visiting

6 has flown is

8 will be working will have traveled 10 is going to fly V Choose the word (A, B, C or D) that best completes each sentence. D B D A D A C C D 10 C 11 D 12 A 13 B 14 D 15 A 16 C 17 C 18 C 19 C 20 B 21 A 22 C 23 B 24 D 25 C

(15)

I Rearrange the following five paragraphs. C

2 A E D B

II Read the passage and choose the best answer to complete it. D

2 A B C B B D C

9 A 10 D 11 A 12 B 13 B 14 A 15 C

III Read the two passages and choose the best answer to the questions or to complete the statements

PASSAGE 1:

1 A C C B B

6 D D B B 10 B

PASSAGE 2:

1 C D A C A B A C A 10 A

(16)

D WRITING:

I Rewrite the sentence as similar in meaning as possible: Going by car is more expensive than (going) by train It’s time we went because our train leaves at A.M In spite of his great efforts, he failed to win the race Because of her carelessness, Mary lost her job A new factory is being built

6 By whom was the window broken?

7 Mary wanted to know what Tom’s job was The fog prevented/ stopped the bus from running It is Tim’ eighteenth birthday next week

10 Peter denied using/ having used Paul’s mobile phone (and said that some else had done) II Write a report of about 150 words describing the information given in the chart (2ms)

Describing a chart - 150 words + Topic sentence : 0.25 m + Supporting sentences : 1.5 ms + Concluding sentence : 0.25 m

III Make all the changes and additions necessary, to produce from the following sets of words or phrases, sentences that together make a complete letter: (20Ms)

Dear Mary,

1 Thank you very much for your letter which arrived a few days ago It was lovely to hear from you

3 I am sorry I haven’t written for such a long time but I’ve been very busy As you know, we bought a new house in May

5 It was in very bad condition and it needed a lot of work We have finished most of it now and it looks very nice

7 John and I have decided to give a house –warming party on October 2nd

8 Do you think you will be able to come?

9 Please give me a ring and let me know if you can make it 10 I’m really looking forward to seeing you again

Love Bob

Sở GD-ĐT Hà Nội

Trờng THPT Xuân Mai

- -Đề THI CHọN Đội tuyển học sinh giỏi lớp 12

Năm: 2008-2009

M«n: TiÕng Anh

Thời gian làm bài: 150 phút (không kể thời gian phát đề)

(Đề thi gồm trang

Thí sinh làm vào đề thi

)

Họ tên:……… Lớp:……….

PART I Tr¾c nghiƯm

I PHONETICS Choose the word whose stress pattern is different from that of the others.

1 A weather B unique C highland D ladder

2 A experience B hurricane C volcano D material

3 A dependence B prepare C prediction D forecast

4 A tidal B ancient C amount D heavy

5 A tropical B commercial C disatrous D extensive

II VOCABULARY Choose the best option (A, B, C or D) which can best replace the underlined word/words.

(17)

A outer B entire C wet D stiff The object is to make our reseearch readable and understandable

A comprehension B comprehensible C comprehensive D comprehend

8 The home of Helen Keller was a crossroads where people gathered to discuss art, politics, and the state of the world

A huge rural estate B national monument C central meeting place D quiet refuge

9 The American Medical Assiciation has called for the sport of boxing to be banned

A forbidden B regulated C reorganised D encouraged

10 The Litlle Foxes, a drama by Lillian Hellman, was first introduced in New York in 1939.

A play B novel C music D dance

11 The policy of separation of church and state has limited direct government support of private schools in the United States

A restricted B reversed C demanded D imposed

12 In the twentieth century, drug markedly improved health throughout the world

A recently B consistently C supposedly D noticeably

13 We finally managed to get her to join us in our games

A persuade her B convince her C insist her D A and B

14 Their attempt to gain a seat in the Parliament came to a success in the end

A showed off B went off C came off D broke off

15 The most dramatic example of static electricity is lightning

A examination B distance C instance D purpose

16 Surveying is the science of calculating exact distances and directions between points of the Earth’s surface

A precise B local C obscure D great

17 Bone and ivory are light, strong, and accessible materials for Inuit artists

A available B beautiful C economic D natural

18 Ants follow scent trails so precisely they can locate their nests without hesitation

A build B find C clean D fill

19 Although its takeoff in April in 1981 was besets with delays, the space shuttle Columbia returned triumphantly two days later from the its mission in the Earth’s orbit

A staff B rescue C funding D launching

20 In the first decades of the 20th century,the individual gene could not be seen, but could be worked with

fruitfully

A blindly B completely C productively D carefully

III GRAMMAR AND STRUCTURES Choose the most suitable answer. 21 “Did you finish the report?” – Yes, but I wish I had known………

A how hard it would be B how hard would it be C how would it be hard D how would it be 22 “Why did Bob get into trouble?” – “Someone heard him……… his views of the boss”

A to express B was expressing C expressed D expressing

23 If you want to join the History Society, you must……… this application form

A wake up B fill in C write down D up

24 “This traffic is terrible”, she said “You’re right People should……… ”

A less use their cars B use less their cars C use their cars less D their cars use less 25 “Computers certainly have changed the world” - “Yes, they make……… to store information” A it easier for people B it more easily for C people easier D easier for people 26 “Are your parents coming to your concert?” – “Yes, but they seem more nervous about…….playing”

A watching me than B watching me C watching than I am about D watch me

27 “I want to buy that car” - “……… its good qualities, it has one drawback”

A Although B Whether C However D Despite

28 ………the students here are serious about studying

A Most B Almost C Almost of D Most of

29 “Which is more important, luck or effort?” - “ Luck is……….effort”

A of the same importance B the same importance as

C of the same importance as D as the same important as

30 He drives so fast that I am afraid one day he will…………somebody crossing the street

A crash down B knock down C turn over D run across

IV READING.

A Read the following passage and answer the questions by choosing A, B, C or D

1

st

passage

NOTICE: To all patrons of the Blue Wave Fitness Center

(18)

31 Where would this notice be posted?

A In a changing room B At a railway station C In a restroom D In an office 32 What members need to open their lockers?

A a series of numbers B coins C keys D Nothing

33 Why you think this notice was posted?

A The Blue Wave has suffered from thefts recently B The Blue Wave has bought new lockers C The Blue Wave changed its management D The Blue Wave is looking for new members

2

nd

passage

Sometimes you know things about people the first time you see them, for example, that you want to be friends with them or that you don't trust them But perhaps this kind of intuition isn't as hard to explain as it may seem For instance, people give out body language signals all the time The way you hold your body, head and arms tells people about your mood If you hold your arms tightly at your sides, or fold them across your chest, people will generally feel that you are being defensive Holding your head to one side shows interest in the other, while an easy, open posture indicates that you are self-confident All this affects the way you feel about someone

Also, a stranger may remind you of a meeting with someone This may be because of something as simple as the fact that he or she is physically similar to someone who treated you well or badly Your feelings about a stranger could be influenced by a smell in the air that brings to mind a place where you were happy as a child Since even a single word can bring back a memory such as that, you may never realize it is happening

34 What does the word "open" in the passage most closely mean?

A Unrestrained B Relaxed C Confined D Unlimited 35

What influences your impression of a person you meet the first time?

A Intuition B Familiarity C Knowledge D Feeling

36 What one feels about a stranger may be influenced by something that .

A strengthens one's past behaviours B reminds one of one's past treatment

C revives one's past memories D points to one's childhood

37 What does the second paragraph discuss?

A Meanings of signals one implies towards a stranger B Factors that may influence one's feelings about a stranger C How people usually behave to a stranger

D Factors that cause people to act differently

38 Intuition described in the passage can be explained by means of _.

A styles B languages C patterns D behaviours

B Read the passage, then choose one word (A, B, C or D) to fill in each blank. MENTORING

Many adults in America and increasing numbers elsewhere(39)……part in mentoring schemes A mentor is an adult who provides support and friendship to a young person There are(40)……different ways of mentoring: passing on skills, sharing experiences, offering guidance Sometimes the most(41)… thing to is just listen Mentoring is open to anybody – no particular(42)…… experience required, just a desire to make a(43)……to the life of a young people who needs help This may seem a difficult thing at first, but many people find they have a real(44)……….for it

The support of a mentor can play an important(45)………in a child’s development and can often make up(46)…………a lack of guidance in a young person’s life It can also improve young people’s(47)…… towards society and build up their confidence in dealing with life’s challenges For the mentor, it can be incredibly rewarding to know that they have had a significant(48)……… on a child and helped to give them the best chance in life.(49)………,it is not only adults who are(50)………… of taking on this role There is now an increasing (51)………for teenagers to mentor young children, for example by helping them with reading or other school work

39 A hold B give C take D form

40 A number B numerous C double D massive

41 A helpful B willing C kind D unlikely

42 A trained B difficult C professional D skilled

43 A home B switch C difference D distance

44 A skill B strength C talent D ability

45 A piece B section C group D part

46 A to B for C with D over

47 A attitude B impression C agreement D conduct

48 A desire B consequence C experience D influence

(19)

50 A able B capable C possible D efficient

51 A want B wish C demand D lack

V WRITING Choose A , B, C or D whose underlined part is not grammatically right. 52 The song to that we listened last night was beautiful

A B C D

53 He gave me the pen in which I wrote my lessons

A B C D

54 Can you explain what the numbers of these animals have become small?

A B C D

55 The sun releases large numbers of energy every day

A B C D

56 The energy of the sun can be changed to electricity A B C D

PART II Tù luËn

I READING

A Supply the correct forms of the words in brackets to complete the passage MEN AND WOMEN ARE NOT EQUAL HERE!

There is a big (57 DIFFER)………in the Bijago Islands off the west coast of Africa in the roles that men and women play For example, men look after the children and wear jewlry and perfume when they go out Women find building houses (58 PREFER)………to shopping and all kinds of job which men would normally in other countries If they want a husband, they ask his mother for (59 PERMIT)……….and not need to obtain his (60 AGREE)……… when they plan their (61 MARRY)……….to him Most men spend hours every day standing in front of a mirror bushing their hair and choosing nice clothes to wear

“I don’t want to make any (62 CRITIC)……….about the women in our society, but I think it’s time that sexual (63 DISCRIMINATE)……….against men stopped”, one man said “Some visitors to our islands find our customs very (64 AMUSE)……… but I feel very (65 NERVE)……….when any girl visits my mother”, another man said

B Complete the following passage by filling each blank with one suitable word.

In Western countries, electricity, gas, and water are not luxuries but necessities Companies now realise that consumers want products that will not (66)………….…work effectively, but save money as well

For most North American households, lighting accounts for 10% to 15% of the electricity bill However, this amount can be (67)……….by replacing an old ordinary 100-watt bulb with an energy-saving one These bulbs use a quarter of the electricity of standard (68)………, and last eight times longer Therefore consumers can (69)……… about US$7 to US$21 per bulb

In Europe, there is a labeling scheme for fridges, freezers, washing machines and tumble dryers The lable tells the consumers how (70)……….energy effeciency each model has, compared with other appliances in the same category

II WRITING

A Rewrite the following sentences beginning with word/words or phrases provided. 71 He tries hard but he cannot master English pronunciation

 No matter ….……… 72 Wherever he goes, his mother goes too

 His mother….……… ….……… 73 The garage is going to repair the car for us next week

 We…… ……….….……… 74 It’s your decision whether we stay in or go out

 It’s up….……… ……… 75 No explanation is needed

 It is…… ……… ……… 76 I can meet you if you arrive before eleven

 So……….….… 77 He would prefer you to pay him immediately

 He’d rather ……… …… 78 Nobody can deny that he has a beautiful voice

 It can’t….……… …… 79 The furniture was so expensive that I didn’t buy it

(20)

80 He didn’t arrive in England untill the end of December

 It wasn’t……… B Use the word given and other words to complete the second sentence so that it has similar meaning to the first Do not change the word given.

81 Tom shares a lot of the same things with Ken (common)

 Tom and Ken… ……… ………… 82 Their house has been broken into three times this year (have)

 They………

83 I hope you enjoy yourself while you stay with Mabel (during)

 I hope you… ……… ……

84 He looks remarkably like his father (resemblance)

 He bears.……… 85 I started studying law in 2003, and now I’m still studying law (since)

 I have….………

86 I am sorry that I have caused you so much trouble (apologize)

 I……… ………

87 Jill doesn’t often walk to the college (unusual)

 It……… ……… 88 My sister always talks when I am watching my favourite programme (habit)

 My sister….………

89 Our team won and the visitors lost (beaten)

 The visitors.……… 90 The number of road accidents has increased drammatically recently (drammatic)

 There… ……… C Write a passage of about 100-120 words on the benefits of the Internet Your passage must cover the following:

- The Internet as a source of information - The Internet as a source of entertainment - The Internet as a source of education Your writing.

(21)

……… ……… ……… ……… ………

§Ị luyÖn thi HSG sè

(School year: 2007-2008)

Question I: Pick out the word with the underlined part

pronounced differently from the rest.

1 A.match B market C tap D cap

2 A.book B look C floor D hook

3 A.hire B find C file D film

4 A.hate B fat C cat D bat

5 A.sun B pull C fun D hunt

6 A.pardon B hard C parent D park

7 A.rose B cover C nose D over

8 A.stage B manage C village D baggage

9 A.supply B supermaket C support D suppose

10 A.blood B mood C moon D soon

Question II: Chooe the word or phrase (A, B, C or D) which best

completes each sentence.

1 " I thought that the tour began at 3:00

"Oh no, you're It began at 1:30 "

A too much late here C here too much late B here much too late D too much here late Although he is intelligent, he doesn't well at school

A Despite being intelligent, he doesn't well at school B Although his intelligence, he does well at school C In spite of intelligent, he doesn't well at school D In spite he is intelligent, he doesn't well at school Jill drives more carefully than Rose

A Rose drives less careful than Rose B Jill is a more careful driver than Rose C Rose drives carelessly than Rose D Jill is a more carefully driver than Rose Jim is five centimeters _ than Tom

A higher B tall C taller D tallest

5 I asked her _ she understood the lesson

A if not B if C even if D if only

6 The man we met yesterday was the manager of a bicycle factory

A whose B which C who D when

7 London is _ of England

A one capital B a capital C capital D the capital I don't believe a word she says, I think she

A is laying B is telling lie C is lying D lied AIDS is a(n) disease

A endanger B dangerous C danger D endangered

10 I knew they were talking about me they stopped when I entered the room

A so that B therefore C because D despite

11 You will have to work hard if you want to _

A successful B succeed C successfully D success 12 She has worked as a secretary _ she graduated from college

A since B until C while D before

13 When I came to visit her last night, she _ a bath

A was having B has C is having D had

14 He has really worked hard so far, he?

A hasn't B has C doesn't D does

15 He is learning English _ he can study in England

(22)

A is spoken B is speaking C was spoken D speaks 17 The children _ to the zoo

A were enjoyed taking B were enjoyed taken

C enjoyed taking D enjoyed being taken

18 My parents first _ each other at the Olympic Games in 1982

A meet B met C had meet D have met

19 The following day she felt well to go to work

A sufficient B sufficiency C suffice D sufficiently 20 he comes in half an hour, we shall go alone

A If B Unless C Because D When

21 Whether we go out will depend the weather

A about B with C in D on

22 Their _ has lasted for more than 20 years

A friends B friendship C friend D friendly

20 If I had time, I to the beach with you this weekend

A would have gone B would go C will have gone D will go 24 She failed the test, _she studied hard

A despite B as though C as D although

25 He took his seat quietly _

A in order that not to disturb their conversation B in order not disturb their conversation

C so as not to disturb their conversation D so as to disturb their conversation

Question III: Read the text below Use the words given in

capitals at the end of each line to form a word that fits in the

space in the same line

My ideal job

One thing I know is that I wouldn't like to have an occupation OCCUPY that has anything to with physics, (1) or maths; CHEMIST I am not the (2) type at all In fact at school, I was a SCIENCE complete (3) in these subjects Neither am I very FAIL

good at dealing with people, nor am I (4) , so jobs in AMBITION business, administration and (5) don't really interest MANAGE me either Moreover I find it (6) to be surrounded by IRRITATE a lot of people; I would much rather have a job involving creative

work or (7) skills of some sort I'd like to have the ART

chance to work outdoors (8) and perhaps a bit of OCCASION travelling too I am not (9) concerned about becoming PARTICULAR rich but I would like to have a (10) income - enough REASON to live comfortably

Question IV: Read the following passage and choose the most suitable word for each space

According to a magazine article I read recently, we (1) live in an age of increasing leisure Not only are more and more people reaching (2) age with their taste for enjoyment and even adventure relatively (3) but the working week is becoming shorter and the

opportunities for (4) are becoming greater and greater all the time Not to mention the fact that people (5) to spend less time travelling to work or may even be working from home What I can't understand, however, is who these people are As far as I can (6) the whole thing is another one of (7) journalistic fictions I admit that there are a lot of retired people

(23)

about it I seem to be working longer and longer hours (12) the time The little leisure time I have is eaten into by sitting in the traffic jams or waiting for trains to (13) up at rain-swept platforms I haven't noticed any dramatic improvements in my (14) either, but perhaps I just have to wait until I get my (15)

A presently B at the moment C now D at this time

A retirement B their C later D third

A present B survived C free D intact

A this B longer C leisure D people

A use B tend C have D demand

A concern B imagine C expect D tell

A the B those C these D some

A in our days B in these times C nowadays D now and again

A round B over C through D into

10 A have B use C the D spend

11 A would B to C had D might

12 A at B for C take D all

13 A keep B line C show D set

14 A cost of living B lifestyle C lifeline D livelihood 15 A pension B retirement C insurance D salary

Question V: Complete the sentence with the most appropriate word in the box.

MTV stands for Music Television It's a television (1) dedicated to pop music It was (2) on 1st August 1981 in the United States Because of MTV's instant success in the

US, the company expanded into other (3) MTV Europe (4) operating on 1st

August 1987 MTV Europe(5) 24 hours a day from its London studios It can be seen in 33 countries and reaches an estimated (6) of 110 million viewers

People of 19 different nationalities work at London headquarters, and they try to offer a (7) of music from all over Europe The channel broadcasts in (8) but Germany provides the biggest number of viewers Currently, one fifth of the (9) is by German artists

Most of TV output is video and concerts, but there is also a programme (10) Unplugged, where major artists play (11) and acoustic in front of a small studio

audience

In addition to music, the channel's programmes (12) with news, movie information and comedy MTV has also broadcast special report (13) racism, immigration and unemployed teenagers

Question VI: Rewrite each sentence so that it contains the word in capitals and so that the meaning stays the same.

1 Everyone thought the book was written by the princess herself

 The princess was I can't lift this table on my own

 I can't lift this table unless The coins are believed to have been buried for safe-keeping

 It is believed that They suspended Jackson for the next two matches

 Jackson was I like cheese best for breakfast

 What I like It's very important for me to know the answer

 To know the answer Why don't we go to the cinema tonight?

 Let's

born music audience deal

called areas mixture on

(24)

8 One of my favourite pastimes is watching plays

 Going Americans are usually quite self-confident

 People in the United States 10 Children don't attend classes on Saturday

 Children don't

11 Andrew said the mistake was my fault

 Andrew blamed me 12 It was raining all evening

 It didn't 13 When he got to the party everyone was chatting and eating

 On his arrival 14 They ate dinner and discussed the problem at the same time

 While 15 Cats are not as loyal as dogs

 Dogs

Question VII: In an essay of about 200 words, name the harmful as well as the beneficial effects of advertising.

(25)

Keys answers:

Question V: Complete the sentence with the most appropriate word in the box.

1 channel broadcast English 11 live

2 born audience music 12 deal

3 areas mixture 10 called 13 on

4 began/ started

Question IV: Complete the sentence with the most appropriate word in the box.

Q # 10 11 12 13 14 15

(26)

SỞ GIÁO DỤC VÀ ĐÀO TẠO KỲ THI CHỌN HỌC SINH GIỎI TỈNH THỪA THIÊN - HUẾ LỚP 12 THPT NĂM HỌC 2003 - 2004

* * * ĐỀ CHÍNH THỨC Mơn: TIẾNG ANH - (Vịng 2)

-HƯỚNG DẪN CHẤM

II PHONOLOGY (10 points)

Q # 10

Key A C D B D C D D C A

III VOCABULARY & GRAMMAR: (20 points ) Part (10 points)

Q # 10

Key B D D D D C A C D B

Part (10 points)

1 put me up putting them across bring her round

2 catch me out bring it up 10 held me up

3 made it up pass it on get it over drop them off IV READING (20 points)

Part (7 points)

Q #

Key D B B C D B D

Part (13 points) V WRITING: (30 points) Part (10 points)

The princess was thought to have written the book herself I can't lift this table unless you help me

4 It is believed that the coins were buried for safe-keeping Jackson was banned (from playing) for the next two matches What I like best for breakfast is cheese

8 Under no circumstances are you to leave the hospital

Part (20 points)

The marks of the essay : 20

(Markers are requested to use the Marking Scheme for writing tasks) Total marks : 100

-UBND TỈNH THỪA THIÊN HUẾ KỲ THI CHỌN HỌC SINH GIỎI TỈNH

(27)

* * * -ĐỀ CHÍNH THỨC Mơn: TIẾNG ANH - (Vịng 2)

Thời gian: 150 phút

(không k th i gian giao

ể ờ

đề

)

ĐIỂM

NHẬN XÉT PHÁCH

-I LISTENING (20 points)

Part 1: Listen to the phone-in Number the opinions in the order that you hear them Which two opinions are not expressed You will hear the recording twice

A It is difficult to control what children watch

B Children can tell the difference between fantasy and reality C Television should show real life

D people should be allowed to watch anything they want E There's too much bad language on TV

F There's too much unnecessary violence on TV G Only sick people watch violence film

H People blame TV for violence so that they can censor it

I Children naturally act out scenes from films and TV programmes K Children need to learn that life isn't always pleasant

Answers:

The two opinions not expressed are: 10

Part 2: Listen to five situations and complete the table You will hear the recording twice

Situation Problem Action offered Choice

1 replacement or refund?

2 smoked salmon

3 left gloves behind

4 put in new battery?

(28)

III VOCABULARY & GRAMMAR (25 points)

Part 1: Choose the best answer to complete each sentence Circle the letter next to the answer you choose

1 These figures give you some idea of the cost of … your car for one year

A controlling B handling C managing D maintaining It can take up to three months to … a man to this specialist work

A guide B raise C train D learn In today’s paper it … that we shall have an election this year

A says B admits C expresses D proposes I had to pay … on a carpet I bought in through the Customs today

A taxes B rates C fines D duty Tropical diseases are comparatively … in Europe

A scarce B rare C slight D few

6 The … charged by the architect for the plans of the new building were unusually high A hire B price C fees D sum

7 It takes a great deal of … for the class to make a trip abroad

A arrangement B organisation C expense D business You shouldn’t eat so many sweets They’re … for you

A bad B unhealthy C unsuitable D disagreeable I wondered whether you would like to … to the theatre tomorrow

A visit B go away C go out D walk out 10 Do you think he is … of doing the job?

A capable B competent C able D suited Part 2: Use the correct forms of the words in the brackets to complete the passage below Centuries ago, when gold coins (1) (exclusive) ……… were used as the money of nations and city-states, the (2) (compare) ……… value of each nation’s money was (3) (determine) ……… by the ratio of gold content of each coin Today, gold coins are “used” only by (4) (collect)……… (5) (nation) ……… money is printed on paper Each country has its own currency, with names such as pound, dollar, dong, yuan and so on (6) (decide) ……… the rate for the international exchange of money is one of the most complex, and to many (7) (observe) ………, one of the most (8) (fascinate) ……… aspects of international banking Each major currency has a “par value” that is (9) (usual) ……… defined, (10) (office) ………., in terms of gold

Part 3: The following sentences are badly constructed Rewrite them in better style and correct any grammatical errors

1 All students must pay their fees, except foreigners, to the university bursar.

2 Swimming is a sport to any healthy person I would recommend.

3 The president said that the new education programme was essential addressing a large

audience in the provincial capital yesterday

4 Talks have begun to plan a railway linking the provincial capitals between the representatives

of the provincial assemblies

5 The advertisement said that they wished to employ a secretary for an expanding company

(29)

III READING: (25 points)

Part 3: Read the text and fill in each blank with ONE suitable word

Our classes take place for three hours every morning from Monday to Friday The maximum class size is twelve (1) the average is ten We use modern methods of (2) and learning, and the school has a language laboratory, a video camera and recorders You will only be successful in improving (3) English, however, if you work hard and (4) speaking English as much as you can You will take a short (5) in English as soon as you arrive In this way, we can put you in a (6) at the most suitable level

There are two classes at the Elementary level; one is for complete (7) and the other is for students who know only a little English, in both classes you will practise simple

conversations In the class (8) the intermediate level you will have a lot of practice in communication in real-life situation because we help you to use the English you have previously (9) in your own country, You will also have the chance to improve your

(10) of English grammar and to build up your vocabulary IV WRITING (30 points)

Part 1: Rewrite each sentence so that it contains the word in capitals and so that the meaning stays the same

Why don't we go to the cinema tonight? FILM

Let's One of my favourite pastimes is watching plays THEATRE

Going Americans are usually quite self-confident STATES

People John is still receiving treatment in the clinic HOSPITAL

John Children don't attend classes on Saturday SCHOOL

Children don't

Andrew said the mistake was my fault BLAMED

Andrew

It was raining all evening STOP

It When he got to the party everyone was chatting and eating ARRIVAL On They ate dinner and discussed the problem at the same time WERE

While

10 Cats are not as loyal as dogs MORE

Dogs Part 2: COMPOSITION (You should spend about 30 minutes on this task.)

(30)(31)

(32)

SỞ GIÁO DỤC VÀ ĐÀO TẠO KỲ THI CHỌN HỌC SINH GIỎI TỈNH THỪA THIÊN - HUẾ LỚP 12 THPT NĂM HỌC 2004 - 2005

* * * ĐỀ CHÍNH THỨC Mơn: TIẾNG ANH - (Vịng 2)

-HƯỚNG DẪN CHẤM

I LISTENING: (20 points )

Part 1: (10 points): one point for each correct answer

1 I D A E H B C F G 10 K

Part 2: (10 points): one point for each correct answer

Situation Problem Action offered Choice

1 broken zip on trousers refund

2 salty soup take it away

Choose something else

3 Keep them or send them on? send them on

4 watch has stopped new battery

5 repair and repaint it today accepts offer

III VOCABULARY & GRAMMAR: (25 points ) Part (10 points)

Q # 10

Key D C A D B C B A C A

Part (10 points) one point for each correct answer

1 exclusively comparative determined collectors national Deciding observers fascinating usually 10 officially Part (5 points) one point for each correct answer

1 All students, except foreigners, must pay their fees to the university bursar Swimming is a sport I would recommend to any healthy person

3 Addressing a large audience in the provincial capital yesterday, the president said that the new education programme was essential

4 Talks have begun between the representatives of the provincial assemblies to plan a railway linking the provincial capitals

5 The advertisement said that an expanding company wished to employ a secretary with good shorthand and typing speeds at their head office

IV READING (25 points)

Part (10 points) one point for each correct answer

1 and your test beginners learnt

2 teaching practise class at 10 knowledge

V WRITING: (30 points)

Part (10 points) one point for each correct sentence let's go to see/ and see a film

(33)

People in/ from the States are usually quite self-confident John is still in hospital receiving treatment

5 Children don't go to school on Saturday Andrew blamed me for the mistake It did not stop raining all evening

8 On his arrival at the party everyone was chatting and eating While they were eating dinner, they discussed the problem 10 Dogs are more loyal than cats

Part (20 points)

The marks of the essay : 20

(Markers are requested to use the Marking Scheme for writing tasks) Total marks : 100

-M«n thi

: TiÕng Anh

Thời gian thi

: 180 phút ( không kể thời gian giao đề)

Ngày thi

: 3/11/2008

Đề thi gồm có trang

Điểm thi

Họ, tên chữ kí giám khảo

(do chủ tịch HĐCT ghi)

Số phách

Bằng số:

Giám khảo 1:

Bằng chữ:

Giám khảo 2:

part I : Listening

Question I: You are going to hear an interview on transportation As you listen, fill in

the form below with no more than three words or numbers:

Questions

Names

How you

get to

school ?

How far is

it from your

home to

school ?

How long

does it take

you to get

to school ?

Are you

ever late

because of

transporta-tion

problems ?

Suggestions

for

improving

the

transpor-station ?

(34)

Liz

1.

4.

It depends

No

10.

Tom

2.

A few

blocks

6.

8.

Question II: You are going to hear an interview on booking a holiday Listen and

tick() whether the statements are True(T), False(F) or Not Given(NI):

Statements

T

F

NI

1 They want to book a holiday for July.

2 They have decided where to go for the holiday.

3 Both customers are free to travel in the first week.

4 Last year, both of them visited France.

5 They would like to go to the mountains for skiing this year.

6 They don't want to go to Italy because the dates don't suit them.

7 They don't like to go to Sweden because there are no beaches.

8 It would be 385 pounds for them to visit Portugal

9 The customers prefer to visit Portugal by flight from London.

10 The flight stops at Manchester on the way to Portugal.

PART II : LEXICO - GRAMMAR:

Question I:

Choose the word or phrase which best completes each sentence Circle the

letter A, B, C or D next to the correct word or phrase.

1 We don't allow in the classrooms.

A people smoke B smoke

C people to smoke

D to

smoking

2 What a dangerous thing to do! You have been killed!

A may B can ` C must

D might

3 By the time I applied, all the holiday vouchers used up.

A had been

B have been C were

D are

4 I felt as if I a confidence.

A have betrayed

B would betray C had betrayed

D am

betraying

5 The candidate to have withdrawn her application.

A claimed

B said

C denied

D reported

6 How could you have him for your brother?

A confused

B considered C thought

D mistaken

7 It is vital that we a change in people's attitudes.

A bring down

B bring back C bring about

D look

after

8 We'll have to down the options before coming to a decision.

A slow B narrow C bring

D wind

9 The grocer's shop since the supermarket opened.

(35)

closed

10 In ten year's time the population of the world to 500 million.

A grows B is growing C has grown D will have grown

11 According to the rules of this contest, all employees and their immediate family member

are

not fit to be chosen to enter the contest.

a preferable

b qualified

c eligible

d desirable

12 When Sarah walked into the room and greeted everyone with a cheery “ hello” , it

seemed as

if all the gloom in that place was driven out.

a expelled B ejected

c banished

d removed

13 When we heard the unexpected news, we were at a loss for words.

a could not speak for a while b did not know what to say

c became dumb

d panicked

14 “ Mum, could you please help me with my homework?”

“ Could you ask your father? I have my hands full at the moment.”

a am very busy

b am very tired

c have to go out

d have a lot on my mind

15 “ Who can tell me what man is? Is he a carnivore or a plant-eating animal or both?” The

science teacher asked the class.

a omnivore b predator

c herbivore

D equator

Your answers:

1………

2………….

3…………

4…………

5………

6………

7………….

8…………

9…………

10…………

11………

12…………

13……….

14……….

15…………

Question II : Give the correct tense and form of the verb to fit each gap.

1 It was essential that the union (1.moderate)………its demand for a shorter working

week.

2 Were I (2 know)……… the answer, I would tell you right away.

3 It was our fault to keep Mary waiting so long We (3.inform)………her in advance.

4 For the past few days, I (4.work)………in Jack's office, as my own office

( redecorate)………….

5 I wish I (6.know)………you needed the book I (7.buy)………it for you in

London.

6 Too big and too heavy (8.pull)………… behind passenger cars, these mobile homes are

moved by tow trucks.

7 The Amazon valley is extremely important to the ecology of the earth Forty percent of

the

world's oxygen ( produce)………… there.

8 I was such a beautiful child that my parents used to have me (10 kidnap)……… just to

see

my picture in the newspapers.

Your answers:

1……… 2………… 3………… 4………… 5………

6………

7………… 8………… 9………… 10…………

Question III : Read the passage and fill in each blank with the correct form of the

(36)

provided

Modern advertisements contain hidden messages (1) IMPLY in the advertisements

showing the pretty girl in the new car or the smiling children round the packet of washing

powder is the message that if we buy the (2) PRODUCE , we also achieve success and

happiness It is a subtle approach since it seeks to exploit our secret teams, and it is (3)

ESCAPE since advertising is ubiquitous, giant street hoardings and (4) CATCH jingles on

TV bombard us form all sides They brainwash us into believing that we can realize our

ambition quickly and (5) EASE On the other hand, defenders of advertising say that it is

(6) BENEFIT Advertising is (7) INFORM Advertisements tell us about useful new

products They (8) BRIGHT our lives with colour and music They increase demand,

stimulate industry and so keep prices down Whether for or against advertising, most people

would agree that some kind of watch-dog body, (9) APPOINTMENT by government or by

the advertising industry itself, necessary to maintain standards or honesty and to discourage

the more blatant types of (10) LEAD advertisements.

Your answers:

1……… 2………… 3………… 4………… 5………

6………

7………… 8………… 9………… 10…………

Question IV : There are ten mistakes in the following passage Underline the mistakes,

number and correct them.

Priscilla Presley, actress and widower of Elvis Presley, entered the acting profession

quiet accidentally On being asked to a television commerce for a well-known shampoo,

she took which turned out to be an important decision: to take actor classes.

She did this in an attempt to overcome her acute shyness at being in the public eye,

but to her amazement she loved every minutes Her extreme lack of confidence stemmed

from the early days of her marriage with Elvis, which she remembers were often spent

sitting in dark, depressed hotel rooms away from the glare of publicity, who Elvis was so

anxious to avoid.

Film work, including the high successful slapstick comedy The naked Gun, quickly

followed the television contract.

Your answers:

1………

2………….

3…………

4…………

5………

6………

7………….

8…………

9…………

10…………

Question V: Read the following passage and use ONE word to fill in each gap.

The Great (1)………of Giza was built as a (2)……… of Pharaoh Cheops in 2720

BC (3)……… it was built a long time ago, its (4)……….makes it one of the true (5)

………… of the world The four (6)……….of the pyramid are put almost exactly on true

north, south, east and west.

Explorations and detailed examinations of the base of the structure reveal many

interesting lines Further (7)……… study indicates that (8)………… represent a type of line

of events-past, present and future Many of the (9)………….have been interpreted and found

to coincide with known facts of the past.

Was this super structure made by ordinary beings or one built by a race (10)…………

superior to any known today?

(37)

1………

2………….

3…………

4…………

5………

6………

7………….

8…………

9…………

10…………

part III : READING.

Question I Read the passage carefully and then choose the best answer to each

sentence.

Hundreds of thousands of persons each year fall prey to some type of cancer, but new

methods of radiation therapy have enabled doctors to save more lives than ever before.

Medical researchers have developed several experimental forms of this time –honored

cancer treatment that seem effective in fighting the disease.

One promising approach involves exposing cancer cells to radiation by implanting a

radioactive source directly into the malignant tissue This process greatly increases the

dosage and thus the effectiveness of the treatment Another technique utilizes drugs to make

cancer cells more susceptible to the effects of radiation and to make normal cells more

resistant Certain drugs are able to neutralize the genetic framework of cancer cells, thus

making them more easily affected by radiation Both techniques have seen some positive

results in the treatment of inoperable brain tumors.

These and other methods have helped to raised the recovery rate for cancer victims

from 30 percent 40 years ago to around 50 percent today This is encouraging news for those

who fall prey to one of the world’s leading killers.

1 What is the author’s main purpose in the passage ?

A To provide statistical information on cancer.

B To argue for new methods of cancer treatment.

C To illustrate new techniques of radiation therapy.

D To give the results of recent cancer research.

2 According to the passage, which of the following is true about radiation therapy?

A.

There is only one effective form of this therapy.

B.

It saves millions of lives each year.

C.

It is an accepted method of cancer treatment.

D.

It causes the incidence of cancer to raise dramatically.

3 In paragraph 2, the word “ malignant ” most closely means :

A diseased

B experimental

C treated

D porous

According to the passage, radiation therapy is most effective when :

A Drugs are used to relax the cancer patient.

B The cancer is directly exposed to the radioactive material.

C It is used on as many patients as possible.

D The cancer cells are resistant to treatment.

It can be inferred from the passage that :

A Improvements in cancer treatment during the last half century have been relatively

ineffective.

B The number of deaths caused by cancer has decreased substantially.

C Fewer people are susceptible to the effects of cancer.

D Scientists are close to eliminating cancer entirely.

Your answers:

1………

2………….

3…………

4…………

5………

Question II For question from – 5, read the following text carefully and then choose

from the list A- I the best phrase given below to fill each of the spaces Each correct

phrase may only be used once Some of the suggested answers not fit at all The

exercise begins with an example ( ).

MODERN ENGLISH

(38)

unfamiliar in places, the idiom occasionally unusual or old-fashioned, the style elegant or

quaint, and we might feel that the language was in some indefinable way characteristic of a

previous age : but we not need to consult a special edition or historical dictionary at

every turn ( ) Jane Austen makes demands on our modern linguistic intuitions

which seem little different from those required by Catherine Cookson or PD James.

However, despite this apparent continuity, the language at the end of the 18

th

century

is by no means identical to what we find today Many words, though spelt the same, had a

different meaning.

( ) we would also notice several differences in pronunciation, especially in the

way words were stressed And an uninformed modern intuition would achieve only a

superficial reading of literary texts of the period In reading a novel of the 1990s, we can

make an immediate linguistic response to the social and stylistic nuances introduced into the

text, ( ) we recognise the differences between formality and informality, or

educated and uneducated; we can sense when someone is being jocular, ironic, risquÐ,

archaic or insincere We can easily miss such nuances in the writing of the early 19

th

century, especially in those works which take the manners of contemporary society as their

subject That world is more linguistically removed from us than at first it may appear.

A in order to understand the text.

B that deceive in its apparent continuity.

C because the context often enables us to see the intended sense.

D what they are today

E because we are part of its age

F when we know it had an additional meaning at that time

G if we had tape recordings of the time

H before a point of linguistic difference might make us pause

I which distance the Early Modern English period from us

Your answers:

1………

2………….

3…………

4…………

5………

Question III Read the following text and decide which word best fits each blank.

OSCAR’S WINNING PERFORMANCE

Two boats, engines paralysed are drifting helplessly towards rocks in a raging sea.

Gale-force winds are blowing as a distress message is relayed to the (1 ) The west

coast search – and- rescue helicopter takes off from Shannon; its destination is Clew Bay in

County Mayo.

The terrified crews on Sundancer and Healther Berry are only half a mile from

disaster when Hotel Oscar, the Irish Marine Emergency Service helicopter arrives and the

winch crew

( ) saving their lives There is no hope for the boats – the conditions are too bad

for that The threatening rocks will make matchwood of them.

It’s not easy to get the rescue line down on the pitching, rolling decks as the pilot,

Captain Al Lockey hovers directly ( ) By the time the exhausted winchman has

picked up the two crew members of Healther Berry , the helicopter is running

( ) on fuel The pair on Sundancer will have to be abandoned if

( ) else is to survive As if that decision isn’t difficult enough, screaming

winds make for a treacherous flight out of the bay.

For Captain Locky, 25 years a helicopter pilot and veteran of typhoon conditions off

oil rigs in the South China Sea, this was the worst experience in a distinguished

( ) In fact, a change in wind direction was to spare Sundancer its horrible face,

much to the ( ) of the rescue crew whose hearts were breaking as they were forced

to turn their backs and ( ) for home Medals, it is said should be given to those who

have to make that most painful decision to say “ no ” Fortunately, most crews can and

(9 ) say “ yes ” in all conditions and at all

(39)

That was Mission 47, accomplished just over three months after Hotel Oscar’s

contract began in July 1991.

1 A shore

B land

C beach

D seaside

2 A set off

B set up

C set out

D set about

3 A above

B higher

C ahead

D over.

`

4 A low

B clown

C short

D out

5 A no one

B everyone

C someone

D all

6 A job

B role

C profession

D career

7 A satisfaction B comfort

C relief

D gratitude

8 A go

B fly

C head

D lend

9 A should

B do

C may

D need

10 A periods

B moments

C hours

D minutes

Your answers:

1……… 2………… 3………… 4………… 5………

6………

7………… 8………… 9……… 10…………

D Writing

Question I : Rewrite these sentences so that the new one has a similar meaning as

the sentence preceded:

1 He did not pass his driving test until he was nearly 30 years old

It

2 He is determined to campaign for his son’s freedom.

He has no

3 Stock brokers bought the company shares because they assumed that it was making a

lot of profit.

Had

4 The financial crisis affects both developed and third-world countries.

Not only

5 He was very sorry because he didn’t have time to visit all places of interest in London

when he was there last year.

He greatly

Question II: Do not change the given word, use it to rewrite each of the following

sentences so that each has as a similar meaning as the original one.

1 Although he had his legs cut off after a serious accident, he competed in the Olympic

Games and won a medal (Prevent)

2 They suddenly realized that they were on the verge of a bankrupt (Dawned)

3 The rocketing prices have proved too much for most salaried people (Cope)

(40)

4 We must accept the fact that we haven't made much progress in the struggle against

poverty (Terms)

5 Illiteracy has been one of the causes of poverty.( Contributed)

Question III: Chart description:

The bar chart compares the prices of hotels and restaurants in 10 big cities in the world.

Write a report, describing the information shown below.

You should write at least 150 words.

N.B Price of a double room with bath, incl breakfast and service, in a 1

st

class

international hotel.

Price of a dinner for one (rib or sirloin steak with two side dishes and a dessert; without

drinks), incl service, in a good restaurant.

(41)

Question IV: Essay writing:

It is very important that children should study hard at school Time spent playing sports

and having fun is time wasted Do you agree?

Use your own ideas, knowledge and experience and support your arguments with examples

and relevant evidence

You should write at least 250 words.

(42)

Hớng dẫn chấm đề thức

Chọn học sinh giỏi môn tiếng Anh năm học 2008 - 2009

Part I : LISTENING (3)

Question I: (10 x 0.1)

1 by car

2 by bike

3 miles

4 20 miles 15-20 minutes

6 15 minutes

7 Yes

8 No

9 Need more buses 10 Better subway

system

Question II : (10 x 0.2 )

1 T

2 F

3 F

4.T

5 NI

6 T

7 NI

8 T

9 F

10 NI

PART II : LEXICO - GRAMMAR: (6.5)

Question I(15 x 0.1 )

1.C

2.D

3.A

4.C

5.A

6.D

7.C

8.B

9.B

10.D

11.B

12.D

13.B

14.A

15.C

Question II: (10 x 0.2 )

1 moderate/ should moderate

2 to know

3 should have informed

4 have been working

5 is being redecorated

6 had known

7 could/ would have bought

8 to be pulled

9 is produced

10 kidnapped

Question III: (10 x 0.1 )

(43)

1 implication

2 product

3 inescapable

4 catchy

5 easily

6 beneficial

7 informative

8 brighten

9 appointed 10 misleading

Question IV: (10 x 0.1 )

1 widower  widow

2 quiet quite

3 which what

4 commerce commercial

5 actor acting

6 minutes minute

7 with to

8 depressed depressing

9 who which

10 high highly

Question V: (10 x 0.1 )

1 pyramid tomb

3 Although

4 construction

5 wonders sides

7 scientific these

9 events

10 far

Part III : READING (4)

Question I (5 x 0.2)

1 C

C

A

B

5 B

Question II (5 x 0.2)

1.D

2.H

A

4.G

5.E

Question III (10 x 0.2)

1.A

2.D

3.A

4.A

5.B

6.D

7.C

8.C

9.B

10.C

part IV: Writing(6.5)

Question I : (5 x 0.2)

1 It was not until he was nearly 30 that he passed his driving test.

2 He has no intention to stop campaigning for his son’s freedom.

3 Had stock brokers knew that the company was not making profit they wouldn’t have

invested in it/bought its shares.

4 Not only are developed countries affected but also third-world countries in the

financial crisis.

5 He greatly regretted that he didn’t have time / not having (enough) time / to visit all

places of interest in London when he was there last year.

Question II: (5 x 0.2)

1 His disability / lame/ legs being cut off did not/could not prevent him from

competing in the Olympic Games and winning a medal.

2 It suddenly dawned on them that they were on the verge of a bankrupt.

3 Most salaried people haven’t been able /cannot cope with the rocketing prices.

4 We must come to terms with the fact that we have not made much progress in the

struggle against poverty.

5 Illiteracy has contributed to poverty

Question III: Chart description: (1,5)

 Style: appropriate: 0.25

(44)

Question IV: Easy writing: (3 )

 Style: appropriate: 0.5

 Spelling + Grammar: accurate: 0.5

 Answering the question asked: 0.5

 Variety of appropriate language: 0.5

 Justify his /her opinions: 0.5

 Compare and contrast his /her evidence and opinions: 0.5

ĐỀ KHẢO SÁT HSG THPT KIM THÀNH LẦN NĂM HỌC : 2008-2009 MÔN: TIẾNG ANH

THỜI GIAN : 150 phút A LISTENING

1

B PHONETICS

I Choose the word whose underlined part is pronounced differently from those of the others.

7

II Choose the word that has a different stress pattern from the others

9 10

C GRAMMAR AND VOCABULARY

I Choose the word or phrase which best completes each sentence. 11

12 13 14 15 16 17 18 19 20

II Give the correct form of the words in brackets. 21

22 23 24 25 26 27 28 29 30

III Choose the underlined word or phrase in each sentence that needs correcting 31

(45)

35 36 37 38 39 40

D READING

I Read the following passage and decide which option A, B, C or D best fits each sentence. 41

42 43 44 45 46 47 48 49 50 51 52 53 54 55

II From the words given in the box, choose the most suitable word for each blank There are more words than blanks, so you don’t need all of them

III Read the passage and choose the correct answer. E WRITING

I A, B, C and D are four sentences which are written using the given words Choose the most appropriate sentence.

66 67 68 69 70

II Finish each of the following sentence in such a way that it means exactly the same as the sentence printed before it.

71 72 73 74 75 76 77 78 79 80

III Essay

(46)

I READING COMPREHENSION : Choose the best answer Passage 1

Sharks have gained an unfair reputation for being fierce predators of large sea animals Humanity's

unfounded fear and hatred of these ancient creatures is leading to a worldwide slaughter that may result in the extinction of many coastal shark species The shark is the victim of a warped attitude of wildlife protection; we strive only to protect the beautiful, non-threatening parts of our environment And, in our efforts to restore only non-threatening parts of our earth, we ignore other important parts

A perfect illustration of this attitude is the contrasting attitude toward another large sea animal, the dolphin During the 1980s, environmentalists in the United States protested the use of driftnets for tuna fishing in the Pacific Ocean since these nets also caught dolphins The environmentalists generated enough political and economic pressure to prevent tuna companies from buying tuna that had been caught in driftnets In contrast to this effort on behalf of the dolphins, these same environmentalists have done very little to help save the Pacific Ocean sharks whose population has decreased nearly to the point of

extinction Sharks are among the oldest creatures on earth, having survived in the seas for more than 350 million years They are extremely efficient animals, feeding on wounded or dying animals, thus performing an important role in nature of weeding out the weaker animals in a species Just the fact that species such as the Great White Shark have managed to live in the oceans for so many millions of years is enough proof of their efficiency and adaptability to changing environments It is time for us humans, who may not survive another 1,000 years at the rate we are damaging the planet, to cast away our fears and begin considering the protection of sharks as an important part of a program for protection of all 30 our natural environment

1 With which of the following topics is this passage primarily concerned? A Sharks are efficient creatures with bad reputations

B Sharks are some of the oldest creatures on earth C Sharks illustrate a problem in wildlife protection

D The campaign to save dolphins was not extended to save sharks

2 Which of the following is most similar to the meaning of the word "warped"?

A distorted B wasteful C extravagant D wanton

3 The word "protested" is closest in meaning to which of the following? A prescribed B objected to C protected D reflected on

4 The word "generated" could be best replaced by A consumed B absorbed C designated D produced 5 How did environmentalists manage to protect dolphins? A They prevented fishermen from selling them for meat. B They pressured fishermen into protecting dolphins by law C They brought political pressure against tuna companies

D They created sanctuaries where dolphin fishing was not allowed 6 About how long have sharks lived on the planet?

A 25 million years B 150 million years C 350 million years D 500 million years

7 The author uses the phrase "weeding out" to mean A strengthening something that is weak

B feeding something that is hungry C encouraging something that is efficient D getting rid of something that is unwanted

8 The phrase "managed to live" is used to infer that A surviving was difficult B migration was common

C procreation was expanding D roaming was necessary

9 The word "proof" could be best replaced by which of the following? A characteristic B evidence C praise D customary 10 The phrase "to cast away" means most nearly

(47)

A explanatoryB accusatory C gentle D proud

12 Which of the following best describes the organization of this passage?

A order of importance B cause and effect C statement and example D chronological order Passage 2

May 7, 1840, was the birthday of one of the most famous Russian composers of the nineteenth century Peter Illich Tchaikovsky The son of a mining inspector, Tchaikovsky studied music as a child and later studied composition at the St Petersburg Conservatory His greatest period of productivity occurred between 1876 and 1890, during which time he enjoyed the patronage of Madame von Meck, a woman he never met, who gave him a living stipend of about $1,000.00 a year Madame von Meck later terminated her friendship with Tchaikovsky, as well as his living allowance, when she, herself, was facing financial 10 difficulties It was during the time of Madame von Meck's patronage, however, that Tchaikovsky created the music for which he is most famous, including the music for the ballets of Swan Lake and The Sleeping Beauty Tchaikovsky's music, well known for its rich melodic and sometimes melancholy passages, was one of the first that brought 15 serious dramatic music to dance Before this, little attention had been given to the music behind the dance Tchaikovsky died on November 6, 1893, ostensibly of cholera, though there are now some scholars who argue that he committed suicide

13 With what topic is the passage primarily concerned?

A the life and music of Tchaikovsky B development of Tchaikovsky's music for ballets C Tchaikovsky's relationship with Madame Von Meck D the cause of Tchaikovsky's death 14 Tchaikovsky's father was most probably

A a musician B a supervisor C a composer D a soldier

15 Which of the following is closest in meaning to the word "productivity" A fertility B affinity C creativity D maturity

16 The phrase "enjoyed the patronage of" probably means

A liked the company of B was mentally attached to C solicited the advice of D was financially dependent upon 17 Which of the following could best replace the word "terminated"? A discontinued B resolved C exploited D hated

18 According to the passage, all of the following describe Madame von Meck EXCEPT A She had economic troubles B She was generous

C She was never introduced to Tchaikovsky D She enjoyed Tchaikovsky's music 19 Where in the passage does the author mention Tchaikovsky's influence on dance?

A May St Petersburg Conservatory B Madam von Meck later terminated financial difficulties. C It was during The sleeping Beauty D Tchaikovsky's music Committed suicide

20 According to the passage, for what is Tchaikovsky's music most well known?

A its repetitive and monotonous tones B the ballet-like quality of the music C the richness and melodic drama of the music D its lively, capricious melodies 21 According to the passage, "Swan Lake" and "The Sleeping Beauty" are

A dances B songs C plays D operas

22 Which of the following is NOT mentioned in the passage?

A Tchaikovsky's influence on ballet music B Tchaikovsky's unhappiness leading to suicide C the patronage of Madame von Meck D Tchaikovsky's productivity in composing

23 Which of the following is closest in meaning to the word "behind"

A supporting B in back of C going beyond D concealing 24 The word "ostensibly" could be best replaced by

A regretfully B assuredly C tragically D apparently1 II Find the error in each sentence and correct it

25 There is an unresolved controversy as to whom is the real author of the Elizabethan plays commonly credited to William Shakespeare

26 Kiwi birds search the ground with the bills for insects, worms, and snails to eat.

27 In 1903, when the Wright brothers announced they had invented a flying machine, his news was generally ignored.

(48)

30 The rate of the heartbeat has been controlled by a small node of nerve-like muscle tissue called the pacemaker

31 George is not enough intelligent to pass this economics class without help 32 Nancy said that she went to the supermarket before coming home.

33 Some tree frogs can alter their colors in order to blend to their environment.

34 Not one in one hundred children exposed to the disease are likely to develop symptoms of it. 35 A dam stops the flow of water, creating a reservoir and raise the level of water.

36 An organ is a group of tissues capable to perform some special function, as for example, the heart, the liver, or the lungs

37 A City University Professor reported that he discovers a vaccine which has been 80 percent effective in reducing the instances of tooth decay among small children.

38 The extent to which an individual is a product of either heredity or environment cannot proven, but several theories have been proposed.

39 If dinosaurs would have continued roaming the earth, man would have evolved quite differently. 40 Find in 1933, The New York Sun was the first successful penny newspaper.

41 A new form of cocaine, crack attacks the nervous system, brain, and bodily in a sharper fashion than cocaine

42 Cartilage covers the ends of bones helps to protect the joints from wear and tear.

43 Denver is call the "Mile High City" because it is at an altitude of 5,280 feet, or one mile, above sea level

44 The common field mouse is about four inches long and has a three-inched tail.

III GRAMMAR AND VOCABULARY: Choose the best answer to complete each sentence 45 - Grace: I wish your parents invite us for holiday - Monty: They can't They have to accommodate us and the children too

A such a small house B a too small house C very small a house D too small a house 48 Do be careful not to your coffee on this white rug, Bill

A drip B spill C filter D leak

49 She had to leave her family when she went abroad to work

A at a loss B behind C at all costs D out

50 Metal at high temperatures

A grows B increases C enlarges D expands

51 Because of the poor harvest, wheat prices have in the last six months

A gone up B grown up C jumped up D sprung up

52 I'm worried about Jane; she always seems to be tired

A as B so C too D such

53 It's most unwise to in a quarrel between a man and his wife

A involve B poke C mix D interfere

54 I will the idea with the other teachers and see what they think about it

A explain B argue C talk D discuss

55 The farmer was very angry the dogs chasing his sheep

A for B with C because D about

56 I think he's quite honest his intentions

A about B with C in D on

57 It's no use complaining the cold

A on B in C from D of

58 She was disappointed when she learned that she had not got the job

A gravely B fully C highly D bitterly

IV USE OF ENGLISH

Read the following text and decide which word best fits each blank CROCODILES

(49)

penetrate, and hold These animals can attack at any (4) ……… of the year, but they are more active in the warmer months and when in (5) ………of mates Underwater, crocodiles (6) …… their victims at the water's (7) ……….by sensing any movement in the water Once they have a hold on their victim, they drag it deep into the water to (8) ……it They then crush and swallow it

Many battles occur over mates About six weeks after mating, the female (9) …….a nest, often on a river bank, and (10) ………about fifty eggs She then seals the nest for protection and also as a way of (11) …… the temperature After ten to twelve weeks the baby crocodiles come out of the eggs: only about one percent of these (12) ……….it to adulthood, as thousands die in flooding or are eaten by fish or bigger crocodiles In an (13)…… to ensure a source of (14) animals, crocodile farms have been (15) …… up, and a vast industry now exists in crocodile skin and meat

59 A harm’s B danger’s C injury’s D damage’s

60 A various B several C considerable D numerous

61.A composed B constituted C designed D styled

62 time B phase C month D interval

63 A discovery B hunt C exploration D search

64 A prefer B accept C propose D choose

65 A border B rim C edge D shore

66 A sink B drown C capsize D soak

67.A installs B makes C manufacturers D produces

68 A lays B sets C drops D puts

69 dominating B ruling C imposing D controlling

70 A reach B make C get D arrive

71 A attempt B action C aspiration D alternative

72 A well B fine C fit D healthy

73 A put B taken C set D stood

V PHONETICS : Choose the word which has different stressed syllable from the other three words

74 A individual B reputation C experience D scientific

75 A carpenter B revise C ignore D traditional

76 A necessary B achieve C poetic D communicate

77 A influence B modern C consider D different

78 A contain B poisonous C chemical D scientist

79 A discover B unhealthy C amount D realize

80 A avoid

B gesture

C permit

D exact

Children who appear to be intelligent and have normal sight and hearing may

nevertheless have learning disabilities such as dyslexia, difficulty in reading;

dysgraphia, difficulty in writing; dyscalculia, difficulty with numbers; and auditory-

memory problems that prevent the child from remembering what has just been said

Considered an “invisible” handicap, such learning disabilities can be detected by alert

parents before the children go to school If a child at about thirty months is not

developing normal language skills, something amiss A child who cannot puzzles

or put pegs in holes lack perceptual- motor skills Kindergartners should recognize

the ABCs First- graders may commonly reverse their letters, writing a d for a b, but

if they are still doing this at the start of the second grade, they should be tested for

learning disabilities Proper and early treatment is essential

56 The author’s intent in this selection is to _ A describe the various types of learning disabilities B explain why some children have dyslexia

C warn parents of the signs of learning disabilities D describe kindergartner’s skills

57 The selection would most likely appear in _

(50)

58 A child who can’t remember a long question might be _

A dyslexia B dysgraphia C auditory-memory problem D hyperactivity 59 The author emphasizes the need for _

A listening to children B more learning centers to help the disabled C trained personnel to prevent learning disabilities

D early detection of learning disabilities

60 A child who reads from right to left may have _

A a poor diet B poor vision C inadequate food D dyslexia

I Choose the word which has a different stress pattern

l: A typhoid B warfare C provoke D patent

2.A satanic B thesaurus C wisdom D redeem

3 A discover B offshore C propel D proper

4 A promulgate B proofread C mediate D correspond

5 A transfusion B compliment C corruption D ascend

II Choose the best answer among A, B, C or D ……… is that a chicken stands up to lay its eggs

A Many people don't realize that C What many people don't realize B It is that many people don't realize D Because many people don't realize

7 As densely populated as the city is, there is surprisingly few people seen… the streets

A to B at C of' D on

8 You go to university if you want to study medicine

A has to B must C will D ought

9 The members expected would cause controversy

A that the report B from the report C at that report D for the report 10 he will change his job if he goes to the USA

A Perhaps' B Recently C Since D Lately 11 Air pollution almost every major city in the world

A that now afflicts B it now afflicts C now afflicts D 'what now afflicts 12 I … someone ….answer the telephone It's been ringing for about five "minutes.' A wish / can B wish /would C wishes /would D wishes/ will

13 … common nuclear reaction, cold fusion does not require high temperature A Alike B It is unlikely C It is not like D Unlike

14 There are …… art galleries in this city

A a great deal B many C much D lots

15 I had to wait half an hour to get this coffee but it was well worth … A the energy B the wait C the time D the trouble

16…… the rainfall was adequate this· year, the apricot trees still did not produce a high yield

A Since B However C Although ' D Due to

17 I'm going to an interview I hope I get the job, please ….for me A keep your fingers B hold your fingers crossed

C hold your fingers D keep your fingers crossed 18 'Where are you going?” “I’m going to buy … ”

A a bread B any bread C loaf of bread D some bread 19 … , all matter is formed of molecules

A It doesn't matter if the complex B No matter how complex

C How complex is not a matter D It's not a complex matter 20 I haven't seen that coat before … have you had it?

A How old B How _ C .How long D When

21 If you are going down town, I… come with you, that’ll save me from taking the bus A can as well B might a well C as well D could well

22 He wasn't rich by any means ,but he never turned … anyone who needed help .°

A off B on C out D down

23: You'd … hurry if you want to be on time

A better B rather C be D A or C

24 We have French classes … day: Monday, Wednesday, and Friday

A each B every C every other D other

25 I used his phone without his permission I … him first

(51)

26 A virus is so … that it can be seen only with an electron microscope

A infinite B imaginary C minute D irregular

27 People who take on a second job inevitably … themselves to greater stress

A offer B subject C field D place

28 Flowers that are perennials begin… the year after they are planted

A prospering B producing C sprouting D blooming

29 Some parts of the world remain … unexplored

A most B virtually C all D already

30 I didn't care for the play because it ended so …

A abruptly B shock C amiably D affluently

31: Many people think of deserts as …regions, but numerous species of plants and animals have adapted to life there

A dead B bland C.dull D barren

32 The Antarctic is…… because of its bleak climate

A uninhabited B not lived C hard D dead

33 The government clearly had not the slightest ….of changing the legislation, in spite of the continued protest

A desire B wish C willingness D intention

34 Although we live in a fairly … society, there is still a lot of poverty

A abundant B affluent C powerful D poor

35 exposure to sunlight is one of the most common causes of skin cancer A Exclusive B Inclusive C Excessive D Extrovert

II Read the passage and choose the best option among A, B, C or D to fill in each blank

Science no longer holds any absolute truths Even the (36) of physics, whose laws once went unchallenged, has had to (37) … to the indignity of an Uncertainty Principle In this climate of belief, we have begun to doubt (38)… fundamental propositions,

and the old distinction between natural and (39) ….has become meaningless The picture of science as a jigsaw (40)…., with a finite number of pieces that would one day all be slotted neatly into (41) … , has never been appealing Experience indicates that things are

not like that at all Every new development in the (42)… reveals further minute detail in structures (43) ……thought to be indivisible Each (44)… in the power of the telescope adds thousands of galaxies to a list already so (45) …… that it is meaningless to all but

mathematicians Even research into what once seemed to be simple behavior patterns has a way of going on forever

36 A region B area C land D discipline

37 A submit B lead C result D give

38 A just B even C with D too

39 A supernatural B unnatural C unreal D surreal

40 A puzzle B problem C quiz D trouble

41 A time B order C sequence D place

42 A binoculars B glasses C telescopes D lenses

43 A once B past C previous D old

44 A exaggeration B growth C enlargement D stretch

45 A obsolete B ancient C old D long

III Read the passage and choose the best answer

The Hollywood sign in the hills that line the northern border of Los Angeles is a- famous landmark recognized the world over The white-painted, 50-foot-high, sheet metal letters can be seen from great distances across the Los Angeles basin

The sign was not constructed,· as one might suppose, by the movie business as a means of celebrating the importance of Hollywood to this industry; instead, it was first-constructed in 1923 as means

of advertising homes for sale in a 500-acre housing subdivision in a part of Los Angeles called

(52)

The sign suffered from years of disrepair, and in 1973 it needed to be completely replaced, at a cost of $ 27,700 per letter Various celebrities were instrumental in helping to raise needed funds Rock star Alice Cooper, for example, bought an O in memory of Groucho Marx, and Hugh Hefner of Playboy fame held a benefit party to raise the money for the Y The construction of the new sign was finally completed in 1978

46 What is the topic of this passage?

A A famous sign B A famous city C World landmarks D Hollywood versus Hollywoodland 47 The expression "the world over” could best be replaced by

A in the northern parts of the world B on top of the world C in the entire world D in the skies

48 It can be inferred from the passage that most people think that the Hollywood sign was first constructed by …

A an advertising company B the movie industry C a construction company D the city of Los Angeles

49 The pronounce 'it' refers to

A the sign B the movie business C the importance of Hollywood D this industry 50 According to the passage, the Hollywood sign was first built in ……

A 1923 B 1949 C 1973 D 1978

51 Which of the following is NOT mentioned about Hollywoodland?

A It used to be the name of an area of Los Angeles B It was formerly the name on the sign in the hills

C There were houses for sale there D It was the most expensive area of Los Angeles 52 The ,passage indicates that the sign suffered because '

A people damaged it B it was not fixed C the weather was bad D it was poorly constructed 53 It can be inferred from the passage that the Hollywood sign was how old when it was necessary to replace it completely?

A Ten years old B Twenty-six years old C Fifty years old D Fifty-five years old 54 The word "replaced" is closest in meaning to which of the following?

A Moved to a new location B Destroyed C Found again D Exchanged for a newer one 55 According to the passage, how did celebrities help with the new sign?

A They played instruments B They raised the sign

C They helped get the money D They took part in work parties to build the sign Choose the underlined part that needs correction

56 Geochemistry includes the study of the movement of elements from one place to another as a result of process chemical

57 Wind is the motion that occurs when lighter air rises and cools heavier replaces it 58 Oceans of the world exerts strong influences on the weather over the Earth’s surface 59 Kiwi birds search the ground with the bill for insects, worms and snails to eat

60 They are going to have to leave soon, and so we

61 All the students are looking forward spending their free time relaxing in the sun this summer 62 If protect, a solar cell lasts for a long time and is a good source of energy

63 The growth rate of the Pacific Rim countries is five times fast as comparable areas during the Industrial Revolution

64 Drug abuse have become one of America’s most serious social problems

65 In 1903, when Wright brothers announced they had invented a flying machine, his news was generally ignored

Choose the sentence with the nearest meaning to the given one. 66 This affair does not concern you

A This affair is not interesting B Don't this affair

C This affair is no business ,of yours D.Your concern is to this affair 67 Not until I left home did I realize how much my father meant to me

A It was only when I left home that I realized how much my father meant to me B I left home, and didn't realize what my father meant to me

C I realized how much my father meant to me just when I was home D Before I left home, I had realized how much my father meant to me 68 Shall we go for a walk?

(53)

A People didn't show up at the meeting B There was a poor turn-out for the meeting C No people came to the meeting D Too many people turned out at the meeting 70 I would have been on time if I hadn't stopped at the post office

A I was on time even though I stopped at the post office B I wasn't on time because I stopped at the post office C I didn't stop at the post office, but I was late

D All of the above

71 So strong were the winds that the trees were uprooted A The winds were not strong enough to uproot the trees B The trees were uprooted because of the strong winds C There were such strong' winds that the trees were uprooted D B and C

72 His behavior was rather a shock to me

A I find his behavior not a shock at all B His behavior took me aback

C His behavior took me away D I was rather displeased with his behavior 73 Paul is not interested in farming

A Paul would like to farm B Farming is not interesting to Paul C Farming is interesting D Paul knows how to farm

74: "That's a lovely new dress, Jean," said her mother A Jean's mother said she liked her dress

B Jean's mother wanted to buy a lovely new dress C Jean's mother told her to buy that lovely new dress D Jean's mother complimented her on the lovely new dress

75 The film star wore dark glasses so' that no-one would recognize him A The film star wanted to be recognized by wearing dark glasses B The glasses were so dark that the film star couldn't see anyone C The film star wore dark glasses in order not to be recognized

D Every one could recognize the film star because he wore dark glasses Choose the best sentence which is made up from the given cues 76 salesman/ try/ talk! met buy/ car

A The salesman tried to talk to me into buying the car

B The salesman tried to talk me buying the car

C I bought the car because the salesman tried it a talk D The salesman tried to talk me into buying the car

77 Marie Curie first scientist/ win two Nobel Prize/ science

A Marie Curie won two Nobel prizes and the first scientist in science B Marie Curie was the first scientist to win two Nobel Prizes in science C Marie Curie was the first scientist winning two Nobel Prize in science

D Marie Curie became the first scientist who won two Nobel Prize about science 78 Their insomnia/ not cause/ high altitude/ excitement

A Their insomnia was not the cause of the high altitude but of excitement

B Their insomnia was not caused from the high altitude but also from excitement C The high altitude and excitement was not caused their insomnia

D Their insomnia was not caused by the high altitude but by excitement 79 fact/ she/ work/ long hours/ spend/ a lot / time/ family

A In spite of the fact that she works long hours, she spends a lot of time with her family B In fact, she works long hours, and spending a lot of time with her family

C Although the fact that she works long hours, she spends a lot of time with her family D The fact that she works long hours makes her spends? lot of time 'with her family 80 build/ hand/ car/ superb condition

(54)

1: A chocolate

B late C fate D ate

2: A revise B divine C mineral D mine

3: A microscope B microscopic C atrocious D cone

4: A chew B fetch C achieve D echo

5: A tissue B permission C press D pressure

6: A research B ability C companion D understand

7: A women B labor C independent D children

8: A career B enjoy C manufacture D community

9: A hospital B careful C needed D unable

10: A attention B scientific C experienced B describe

11: Traditional sailors many of the same dangers as their predecessors

A face is B is facing C face D are faced

12: … patients should try to reduce needless office visits for colds and minor respiratory illnesses A Doctors that agree B That doctors agree C Doctors agreeing that D Doctors agree that 13: The music next door is very loud I wish they …

A would turn it down B will turn it down C can turn down it D turn down it 14: Let's wait it stops raining

A until B till C for D A or B

15: He's not here on vacation, he came …

A forever B for good C for bad D A or B

17: He hasn't driven a car

A recently B since June C two years ago D A or B 18: The black bear suddenly appeared from

A behind B out C with D after 19 They went to the party without …

A to be invited B inviting C being invited D invited

20 Within the first ten minutes the chairperson knew the meeting …

A would come to nothing B would nothing come to C would come nothing D nothing would come to

21 Bob is … …of the twins

A the biggest B bigger C more big D most big

22 What is the most interesting place … you … to?

A did/ go B had /been C have been D has /been

23: We'd better : or else we will never finish it in time A get the ball rolling- B the ball rolling

C roll the ball D the ball to roll

24: Every …… I have to clean out my closet because I have too many things

A sometimes B now and then C occasionally D then and now

25 I don’t think Jane will leave this Sunday I wish ……

A she won’t B she wouldn’t C she will D she isn’t

26: Researchers are working on alternate energy sources to eventually replace the supplies of fossil fuels

A down B dwindling C withdrawing D lowering

27: A good proofread …… examines a manuscript for errors in spelling and grammar as well as for factual mistakes

(55)

A other B apart C else D except

29 passengers are …… not to leave cases and packages here

A commanded B informed C notified d advised

30 In his absence I would like to thank all concerned on my brother's ……

A interest B part C behalf D business

31: The young men were … guilt of shoplifting A found B convicted C accused D condemned

32: Like radios, automobiles began as …… , but soon became a powerful force for social change A necessities B symbols C playthings _ D experiments

33: I was in the book…… I was reading, and I didn't hear the phone A distracted ' B submerged C gripped D engrossed

34: According to the …… of the contract, tenants must give six months' notice if they intend Jo leave A laws B rules C terms D details

35: Some people believe that the crystals of certain minerals have powers

A medicine B curative C magical D necessary

The most difficult task confronting us, in any period when there is a marked (36) …of social power, is the complicated process of revaluation of the (37) tradition The common language, because in itself it is so (38) … to this matter, provides an excellent instance It is clearly of (39) … ,

importance to a culture that its common language should not (40)… in strength, richness and flexibility; that it should further, be adequate -to express new experience, and to (41) change But a language like English is still evolving, and great (42) can be done to it by the imposition of crude categories of class It is obvious that (43)… the development in the nineteenth century, of the new definition of 'standard English' particular uses of the common language have been taken and (44) for the purposes of class distinction Yet the dialect which is normally (45) with standard English has no necessary (46) over other dialects Some of its grammatical elements have a common importance, (47) not all of them On the other hand, certain selected sounds have, been given an authority which (48) from no known law of language but simply from the (49) that they are (50) … made by persons who, for other reasons, possess social and economic influence

36 37 38 39 40 41 42 43 44 45 46 47 48 49 50

Medical research has shown that mothers are right when they tell children to eat their vegetables Fruits and vegetables have been shown to contain beneficial compounds that may encourage the body to stay healthy Experts think that some of these compounds may even help fight off diseases such as cancer They recommend that people consume a minimum of five servings of fruits and vegetables each day Of course, for maximum benefit this should be combined with an overall reduction of dietary fat (including meats and cheeses) and a regular program of physical activity

51: Which food groups does the article recommend?

A Meat and eggs B Fruits and vegetables C Cheese and fish D Breads and cereal 52: What else experts say about the compounds in these foods?

A They have vitamins B they contain fat C They may fight diseases D They have calories 53: The word "consume" in the paragraph most closely means _.

A buy B sell C eat D stand

54: How many servings of fruits and vegetables should people eat?

(56)

A Cut down on fat B See the doctor C Eat meat `D Drink water

Mars is an inhospitable planet, more similar to Earth's moon than to Earth itself It is frozen solid, blanketed by the mere wisp of an atmosphere- a dry, stark, seemingly lifeless world However, certain scientists believe that these conditions could be changed They have begun to seriously investigate the possibility of transforming Mars into a more Earth-like planet: enriching the "atmosphere with oxygen, obtaining water from polar ice caps, seeding the planet with plants and animal life from Earth, and

eventually establishing permanent human colonies "It was once thought to be so far in the future as to be irrelevant,''' said Christopher McKay, a research scientist at the National Aeronautics and Space

Administration "But now it is starting to look practical We could it in four or five decades." The idea of "terra-forming" Mars, as enthusiasts" call it, has its roots in science fiction But as researchers develop a more profound understanding of how Earth's ecology supports life, they have began to see that it may be possible to create similar conditions on Mars

56: With which of the following is the passage primarily concerned? A The possibility of changing the Martian environment

B The challenge of interplanetary travel

C The advantage of establishing colonies on Mars D The need to study the Martian ecology

57: Which of the following does the author NOT gives as a characteristic of the planet Mars? A it is cold and dry B it is quite similar to Earth's moon

C It apparently has no life D it has a rich atmosphere

58: According to Christopher McKay, the possibility of transforming Mars _ A could only occur in science fiction stories

B will take place in the next few years

C is completely impractical D could be accomplished in 40 to 50 years 59: As used in the passage, the term “terra forming” refers to _

A a process for adapting plants to live on Mars B a means of transporting materials through space C a method of building housing for colonists on Mars

D a system of creating Earth-like conditions on other planets

60: According to the article, the basic knowledge needed to transform Mars comes from _

A the science of astronomy B a knowledge of Earth’s ecology C data from space proves D science fiction stories

61: Laser technology is the heart of a new generation of high-speed copier and printer 62: The symptoms of diabetes in the early states are too slight that people don't notice them 63 Hay fever symptoms, ranged from mild to severe, differ in degree according to the individual 64: International trade, going traveling, and television have lain the ground work for modern global life styles

65: If we don't economy on electricity, there will be power cuts

66 Although Christopher was the stronger of the two, his attacker soon overpowered him A Christopher’s attacker was too weak to defeat him

B Christopher was weaker, but he overpowered his attacker C Christopher lost because he had no power

D Despite his superior strength, Christopher was soon overpowered by his attacker 67: You're under no obligation to accept their offer

A You're obliged to accept their offer

B You're not obliged to accept their offer at all.·

C You can please yourself whether to accept their offer or not D B and C

68: My father finds maps hard to follow A My father always gets lost

B My father can't read maps at all C My father has trouble following maps

D Map-reading is not interesting to my father at all 69: You need not this job

A You have to this job B This needs to be done C This job needs not be done D You need this job 70: She has lost her appetite recently

A She hasn't had any food recently B Her appetite has been very good

C She has gone off food recently D She has eaten a lot of food recently 71: complete/ building/use/ convention center

A When completing, the building will be used as a convention center B Completing the building has used the convention center

C When completed, the building will be used as a convention center D When completed, we use the building as a convention center 72: Mexico earthquake/ 1985/ bad/ 1979

(57)

C Mexico earthquake in 1985 was far worse than that of 1979 D In Mexico the earthquake of 1985 was far worst than that of 1979 73: you/ better/ send/ card/ today/ miss/ mother's/ birthday

A You're better send your card today in case you'll miss your mother's birthday B It's better for you to send your card today not to miss your mother's birthday C You'd better send your card today or you’ll miss your mother's birthday

D It's better for you to send your card today or you won't miss your mother's birthday 74: start/ fire/ no one/ seem! worry

A Start the fire so as no one seemed very worried B At the start of the fire, no one seemed very worried C The start of the fire made no one seem very worrying D In the start of the fire, no-one seemed a worry

75: opinion/ Jane/ make/ story/ meet! President

A My opinion was in that Jane made the story to meet the President B My opinion of Jane made the story and met the President

C In my opinion, Jane made up that story about meeting the President D For my opinion, Jane made up for that story about meeting the President 76: Unlike/ birds/ vultures/ head/neck/ feathers "

A Unlike most birds, the heads and necks of vultures lack feathers

B Unlike most birds, feathers are not found on the heads and necks of vultures " C Unlike most birds, vultures not have feathers on their heads and necks

D Unlike most birds, no feathers are on vultures' heads and necks 77: tablets/ 1/ take/ no effect/ headache "

A The tablets that I took had no effect into my headache B The tablets that I took had no effect on my headache C The tablets' that I was taken had no effect on" my headache D The tablets that I took gave no effect for my headache 78: I / try/ phone/ sister/ Australia/ an hour/ not get

A I've been trying to phone my sister, but Australia took over an hour not to get through B I've been trying to phone my sister in Australia for over an hour, but I can't get through C I tried to phone my sister in Australia for over an hour, but it can't get through " D I tried a phone from my sister in Australia for over an hour, but I can't get 79: challenge/ play tennis/ he/ not stop/ practice

A When the challenge to play tennis, he didn't stop practicing B As a challenge to play tennis, he wouldn't stop practicing C Once challenging to play tennis, he wouldn't stop practicing D Once challenged to play tennis, he wouldn't stop practicing 80: people/ object/ plans/ new car park

(58)

ĐỀ KHẢO SÁT HSG THPT KIM THÀNH LẦN NĂM HỌC : 2008-2009

MÔN: TIẾNG ANH THỜI GIAN : 180 phút A LISTENING

1

B PHONETICS

I Choose the word whose underlined part is pronounced differently from those of the others.

6 A bomb B climb C table D comb

7 A plays B says C stays D lays

II Choose the word that has a different stress pattern from the others

8 A simultaneous B feedback C different D errors

9 A arrangement B distinguish C theorist D collective

10 A optimism B struggle C configuration D talents

C GRAMMAR AND VOCABULARY

I Choose the word or phrase which best completes each sentence. 11 They praised her for her _

A dishonest B honestly C honesty D honest

12 If I make a fool of myself in front of my friends, I’ll never _ down

A let B give C settle D live

13 If you have anything important to do, it straight right away Don’t put it _

A on B off C over D up

14 I was horrified _ his appearance He looked as if he hadn’t slept _ weeks

A at … for B by … for C about …in D A or B

15 Most folk songs are ballads _ have simple words and tell simple stories

A what B although C with D that

16 Solar heat penetrates more deeply into water than it _

A it is penetrating into the air B it does into soil

C does it into soil D that it does into soil

17 Would you be _ my letter while I am away?

A too good as to forward B as good as forward C so good as to forward D so good as forwarding 18 Nowhere _ such cooperative staff

A you can find B you found C you could find D can you find

19 You need to get some job retraining _ it, you risk being laid off

A If so B If not C With D Without

20 The shape of _ snowflake is unique

A x B a C an D the

II Give the correct form of the words in brackets.

21 She always listens _ to what she is told (ATTENTION) attentively 22 In his family, he is a _ child (TROUBLE) troublesome

23 There has been a _ improvement in her writing.( NOTICE) noticeable 24 The price of the house includes many existing _ and fittings (FIX) fixtures 25 It was _ of me to mislead you like that.( FORGIVE) unforgivable

26 He is very _ in everything he does.( SYSTEM) systematic 27 Fruit _ as it ripens (SWEET) sweetens

28 My brother lives in a _ area (RESIDE) residential

29 She is extremely _ about art ( KNOWLEDGE) knowledgeable

(59)

34 Before TV, the common man seldom never had the opportunity to see and hear his leaders express their views

35 As a result of the Women’s movement, women now holds positions that were once restricted to men 36 Jogging, nor dieting, carried to extremes, can be harmful

37 The letter that was sent by special delivery must be importance 38 Lasers are indispensable tools for delicate eyes surgery

39 Alexander Calder, who was originally interested in mechanical engineering, later became a sculpture 40 To building their nests, tailorbirds use their bills as needles

D READING

I Read the following passage and decide which option A, B, C or D best fits each sentence. Attention Deficiency Disorder (ADD) is a neurobiological problem that affects

3-5 % of all children (36) _include inattentiveness and having difficulty getting organized, as well as easily becoming (37) _ Sometimes, A.D.D is accompanied by hyperactivity In these cases, the sufferer exhibits (38) _physical activity

Psychostimulant drugs can be(39) _ to A D D sufferers to assist them with the completion of desired thought professes, although they might cause (40) _ Current theory states that

medication , the only (41) _action that has a sound scientific basis This action should only be taken after an accurate diagnosis is made

Children with A.D.D not (42) _ have trouble learning; their problem is that they involuntarily (43) _ their attention elsewhere It is not only children that are (44) _by this condition Failure to treat A.D.D can lead to lifelong emotional and behavioral problems Early diagnosis and treatment,

however, are the key to successfully overcoming learning difficulties (45 ) _ with A.D.D

41 A Symptoms B Signs C Signals D Sights

42 A dim B distracted C divulged D unattended

43 A exaggerated B over C excessive D rich

44 A given B handed C drunk D taken

45 A disadvantages B damage C side-effects D pain

46 A remedial B medical C health D cure

47 A eternally B absolutely C always D necessarily

48 A switch B move C bring D carry

49 A worsened B spoiled C caused D affected

50.A associated B combined C supplied D made

II Read the passage and choose the correct answer.

Scientists claim that air pollution causes a decline in the world’s average air temperature In order to prove that theory, ecologists have turned to historical data in relation to especially huge volcanic eruptions They suspect that volcanoes effect weather changes that are similar to air pollution

One source of information is the effect of the eruption of Tambora, a volcano in Sumbawa, the Dutch east Indies, in April 1815 the largest recorded volcanic eruption ,Tambora threw 150 million tons of fine ash into the stratosphere The ash from a volcano spreads worldwide in a few days and remains in the air for years Its effect is to turn incoming solar radiation into space and thus cool the earth For example, records of weather in England show that between April and November 1815, the average temperature had fallen 4.5 °F During the next twenty-four months, England suffered one of the coldest periods of its history Farmers' records from April 1815 to December 1818 indicate frost throughout the spring and summer and sharp decreases in crop and livestock markets Since there was a time lag of several years between cause and effect, by the time the world agriculture commodity community had deteriorated, no one realized the cause

Ecologists today warn that we face a twofold menace The ever-present possibility of volcanic eruptions, such as that of Mt St Helens in Washington, added to man's pollution of the atmosphere with 011, gas, coal, and other polluting substances, may bring us increasingly colder weather

51 It is believed that the earth get colder when _

A volcanoes erupt B the air is polluted by modem man

C the rays of the sun are turned into space D all of the above 52 The effects of Tambora's eruption were _

A felt mainly in the Dutch East Indies B of several days' duration worldwide

C evidence of population's cooling the earth D immediately evident to the world’s scientist 53 The cause of cold weather I England from 1815 to 1818 was _

A decreased crop and livestock production B volcanic ash in the atmosphere C population caused by Industrial Revolution D its proximity to the North Sea

54 No one realized the cause of the deterioration of the world’s agriculture commodity market because _

A there was a long delay between cause and effect B the weather is beyond our comprehension

C weather forecasts were inaccurate

D ecologists didn’t exist until modern times

55 If, as some scientists predict, the world ends in ice, what might be the cause?

A modern man’s pollution of the air ` B volcanic eruptions

C obliteration of solar radiation D all of the above

(60)

words than blanks, so you don t need all of them.

lose amounts guess supply invest materials earn thing down examples low increases INVESTMENT IN COMMODITY

A rather risky way of investment is in commodities They are the (56) _ that companies and businesses use Some (57) _ of commodities are wheat, corn, coffee, chickens and copper

Investors buy large (58) _ of a commodity and hope that the price (59) _ They often buy directly from the producer or a farmer before the commodity is produced There are many things that can affect the (60) _ of a commodity, and, therefore, the price If the weather is good next year, there will be a lot of corn, and the price will be (61) The investor will (62) money If the weather is bad, the corn will be more expensive, and the investor will (63) more money People who ( 64) _ in commodities are called speculators To speculate means to (65) _ about the future and to take risks Speculators take many risks when they buy commodities

E WRITING

I Choose the sentence which is closest in meaning to the one given 66 Nothing but the whole story would satisfy Tim.

A Tim wouldn’t be satisfied with anything B Tim wanted to know just the end of the story

C On the whole, Tim was satisfied with the whole story D Tim insisted on being told the complete story

67 He survived the operation thanks to skillful surgery. A He survived because he was a skillful surgeon

B Without skillful surgery, he wouldn’t have survived the operation C There was no skillful surgery, so he died

D In spite of the surgery, he didn’t survive

68 With six children on her hands, she’s extremely busy. A She’s very busy because she always carries her six children B Her six children’s hand always make her busy

C With six children to look after, she’s extremely busy D She’s too busy to look after her six children

69 I’ve yet to meet a more exasperating person than my brother -in -law. A My brother-in-law is the most exasperating person I’ve ever met B My brother-in-law is more exasperating than I am

C I’ve never met a more exasperating person than my brother-in-law D A and C

70 Were it not for the money, this job wouldn’t be worthwhile. A This job is not rewarding at all B This job offers a low salary C Although the salary is poor, the job is worthwhile

D The only thing that makes this job worthwhile is the money

II A, B, C and D are four sentences which are written using the given words Choose the most appropriate sentence.

71 no circumstances/ staff members’ telephone numbers/ give out. A No circumstances are the staff members’ telephone be given out

B Under no circumstances should the staff members’ telephone numbers’ be given out C With no circumstances, the staff members’ telephone numbers are given out

D The staff members’ telephone numbers are to give out no circumstances 72 entire business/ computerize/ over two years.

A The entire business has been computerized for over two years B The entire business has computerized for over two years C The entire business is computerized for over two years D The entire business is being computerized for over two years 73 it/ sign/ fall/ leaves/ trees/ begin/ change/ color.

A It is a sign for fall when the leaves on the trees begin to change color B It is a sign to fall while the leaves on the trees are begun to change color C it is a sign of fall when the leaves on the trees begin to change color D It is a sign of fall if the leaves on the trees begin to change color 74 lack/ exercise/ high-fat diets/ long /know/ factors/ heart/ attacks.

(61)

75 Marie Curie/ show/ woman/ good/ scientist/ man.

A Marie Curie showed that a woman can be as good a scientist as a man can be B A good woman as Marie Curie was shown to be a scientist as man

C Marie Curie showed to be a woman is as good a scientist as a man

D Marie Curie has shown that a woman can be good as a scientist like a man can be

II Finish each of the following sentence in such a way that it means exactly the same as the sentence printed before it (Olympic- page 28)

76 Arthur said he was sorry he had hurt her feelings=> Arthur apologized ……… 77 Far more people live to retirement age in Britain than in the Philippines

Not ………

78 His father is going to fix the ball for him tomorrow.=> He is going……… 79 She never seems to succeed even though she studies much

=> Much………

80 I would love to be rich and famous => If only ………

III Complete the sentences using the cues - Luyen Viet page 38 Dear Sir,

81 I/ write / complain/ dirt/ smoke/come/factory/chimneys.

82 Three days ago / I decide/ my washing.

83 I / wash/ sheets/ put them out/ dry/ as/ it/ nice sunny day/ there / breeze. 84 When/ take/ washing in/ I horrified/ discover/ it/cover/ dirty marks. 85 I assume/ breeze/ mention// carry/ dirty/ you factory smoke

86 This/ be/ not only/ because of/ sheets/ but/ because /have/ two small children/ be /made/ breathe/ same/ air

87 Until /incident/ I / think/ chimneys/ safe/ clean. 88 I / already write/ local council/ this matter.

89 Furthermore/ I / warn/ write/ local newspaper/ tomorrow. 90 I / look/ receive/ reply.

Yours sincerely,

ĐỀ KHẢO SÁT HSG THPT KIM THÀNH LẦN NĂM HỌC : 2008-2009

MÔN: TIẾNG ANH THỜI GIAN : 180 phút A LISTENING

B PHONETICS

I Choose the word whose underlined part is pronounced differently from those of the others.

6 A departure B mature C nature D picture

7 A decided B hatred C warned D sacred

8 A thought B tough C taught D bought

II Choose the word that has a different stress pattern from the others

9 A authority B entrance C selection D emission

10.A involve B guidance C enhance D society

C GRAMMAR AND VOCABULARY

I Choose the word or phrase which best completes each sentence. 11.While I _ TV last night, a mouse ran across the floor

A watch B watched C was watching D am watching

12 Never _ such a sight!

A I did see B have been I C I saw D have I seen

13 I think the prices here are _

A convenient B reasonable C comfortable D available

14 Although _ a country illegally is risky, the alien who finds work may believe the work worthwhile

A when entering B he enters C entering D having entered

15 The stomach of a cow, _ that of other ruminants, is divided into four compartments

A likely B similar C same D like

16.Would you consider _ that car if it had better mileage statistics?

A to buy B bought C buying D buy

17 Not only _ in the field of psychology but anima behavior is examine as well A human behavior is studied B is human behavior studied

(62)

A both / or B not only/ but also C neither/ or D whether/or 19 Scarcely was _ wearing a dinner jacket

A anyone B someone C little D any

20 The building work must be finished by the end of the month _ of cost

A ignorant B thoughtless C uncaring D regardless

II Give the correct form of the words in brackets.

21 He doesn't know much about the subject, but he is very _ (ENTHUSIASM) 22 The charity is totally upon money from the public (DEPEND)

23 The bank has been very in helping me sort out my late husband's finances (COOPERATE) 24 Is military service in your country? (COMPEL)

25 It was of you not to offer her a drink (HOSPITABLE) inhospitable 26 The carpet is available in various (WIDE) widths

27 The plane grounded because of poor (VISIBLE) 28 He resigned for a of reasons (VARIOUS)

29 He wrote so many sentences in his essay (GRAMMAR) ungrammatical 30 They daren't leave their children for even a moment (ATTEND) unattended III Choose the underlined word or phrase in each sentence that needs correcting

31 In order for one to achieve the desired results in this experiment, it is necessary that he(A) work(B) as fastly(C) as(D) possible

32 The(A)new model costs(B) twice more than(C) last year’s(D) model As much as 33 Fuel cells, alike(A) batteries, generate(B) electricity(C) by chemical reaction(D)

34 Of(A) the more than 1,300 volcanoes(B) in the world, only(C) about 600 can classify(D) as active Be classified

35 A huge amount(A) of tourists(B) come(C) to this city(D) every year

36 That man was(A) an easy(B) recognized figure(C) with his long(D), white beard and wide brimmed hat easily

37 Industrial(A) lasers are most often(B) used for(C) cutting, welding, drilling, and measure(D) Ving 38 Not one(A) in one hundred children exposed to the disease are(B) likely(C)

to develop(D) symptoms of it

39 The examination will test(A)your ability to understand spoken(B) English, to read nontechnical language, and writing(C) correctly(D)

40 Computers have(A) made access(B) to information instantly available(C) just by push(D) a few buttons D READING

I Read the following passage and decide which option A, B, C or D best fits each blank

Most people are unaware they possess a quite remarkable skill, which is usually overlooked because it is (41) _ daily, and in the most ordinary of contexts But without it, our lives would be unfulfilled and empty It is the (42) _ to relate to others, to engage them in conversation, to operate as social and sociable individuals and to develop both short-term and long-term relationships which lies at the heart of our very (43) _ as human beings We are not (44) _ with this ability There is nothing wired into the human brain that provides us with (45) _ responses to social situations To perform effectively in a world that relies so heavily on social (46) _, encounters and relationships, we have to learn what to

Small babies , as any parents will remember, are among the least sociable beings that you could imagine They are totally (47) _, utterly selfish and scream with rage if their every whim is not (48) satisfied Somehow this unlikely raw material is (49) _ over the years into a being which relies for survival on being able to form (50) _ bonds with other and to follow complex rules that (51) _ every aspect of its social life The monstrous (52) _ becomes the caring, responsible adult whose life

experiences (53) _ around both the joys and pains, and the (54) _ and receiving, of friendship and other relationships It is this remarkable transformation which is the central characteristics of being (55) _

41 A exercised B done C rubbed D polished

42 A capacity B proficiency C competence D ability

43 A living B survival C existence D sustenance

44 A trained B born C educated D delivered

45 A set B molded C formed D remained

46 A intersection B interaction C conversation D reaction

47 A demanding B difficult C unsatisfied D unhappy

48 A now B immediately C soon D then

49 A altered B transformed C switched D changed

50 A influential B lateral C bilateral D reciprocal

51 A determine B decide C govern D form

52 A kid B infant C boy D girl

53 A circle B focus C concentrate D revolve

54 A giving B offering C distributing D handing

55 A humane B men C human D person

(63)

extinction which inefficient sources discharged survival resources exist made Clean fresh water resources are essential for drinking, bathing, cooking, irrigation, industry and for plant and animal (56) _ Unfortunately, the global supply of fresh water is distributed unevenly Chronic water shortages (57) _ in most of Africa and drought is common over much of the globe The (58) _ of most fresh water supplies- ground water ( water located below the soil surface), reservoirs, and river- are under severe and increasing environmental stress because of overuse, water pollution, and ecosystem degradation Over 95% of urban sewage in developing countries is (59) _ untreated into surface waters such as rivers and harbors

About 65% percent of the global freshwater supply is used in conservation therefore requires a reduction in wasteful practices like (60) _ irrigation, reforms in agriculture and industry, and strict pollution controls worldwide

III Read the passage and choose the correct answer Olympic page 76

Computer programmer David Jones earns 35,000 a year designing new computer games, yet he cannot find a bank prepared to let him have a cheque card Instead, he had been told to wait another two years, until he is 18

The 16-years old works for a small firm in Liverpool, where the problem of most young people iof his age is finding a job David’s firm released two new games for the expanding home computer market each month

But David’s biggest headache is what to with his money Despite his money, earning by inventing new programs within tight schedules, with bonus payment and profit- sharing, he cannot drive a car, take out a mortgage, or obtain credit cards

He lives with his parents in their council house in Liverpool, where his father is a bus-driver His company has to pay 150 a month in taxi fares to get him the five miles to work and back every day because David cannot drive

David got his job with the Liverpool-based company four months ago, a year after leaving school with six 0-levels and working for a time in a computer shop “ I got the job because the people who run the firm knew I had already written some programs,” he said

I suppose 35,000 sounds a lot but actually that’s being pessimistic I hope it will come to records and clothes and gives his mother 20 a week But most of his spare time is spent working “ Unfortunately, computing is not part of our study at school,” he said “ but I had been studying it in books and magazines for four years in my spare time I knew what I wanted to and never considered staying on at school Most people in this business are fairly young anyway.”

David added, “ I would like to earn a million and I suppose early retirement is a possibility You never know when the market might disappear.”

61 What makes David different from other young people of his age?

A he earns an extremely high salary B He is not unemployed

C He does not go out much D He lives at home with his parents

62 David’s greatest problem is _

A making the banks treat him as adult B inventing computer games

C spending his salary D learning to drive

63 He was employed by the company because _

A He had worked in a computer shop B He had written some computer games

C He works very hard D He had learnt to use computer at school

64 He left school after taking O-levels because

A he did not enjoy school B he wanted to work with computers and staying at school did not help him C he was afraid of getting too old to start computing D he wanted to earn a lot of money

65 Why does David think he might retire early? A You have to be young to write computer programs B He wants to stop working when he is a millionaire C He thinks computer games might not always sell well D He thinks his firm might go bankrupt

E WRITING

I A, B, C and D are four sentences which are written using the given words Choose the most appropriate sentence.

66 people/ livewilderness/ the Yukon/ have/ self-sufficient

A The people who lived in the wilderness of the Yukon had to self-sufficient B The people lived in the wilderness of the Yukon had to be self-sufficient C The people who lived in the wilderness of the Yukon had to be self-sufficient D The people living in the wilderness of the Yukon had to self-sufficient 67 rainfall/ adequate/ this year/ apricot trees/ not produce / high yield.

A Since the rainfall was adequate this year, the apricot trees did not produce a high yield B However the rainfall was adequate this year, the apricot trees were not produced a high yield C Although the rainfall was adequate this year, the apricot trees still did not produce a high yield D Due to the rainfall was adequate this year, the apricot trees did not produce a high yield 68 the election/nation/ enter/ new/ stage.

A As the election happened, the nation was entered a new stage B After the election, the nation entered a new stage

(64)

69 all-day boat trip to Easter Egg Rock / tiring/ stimulating. A A all-day trip to Easter Egg Rock is tiring, but stimulating

B When going an all-day boat trip to Easter Egg Rock is tiring, but stimulating C Going on an all-day boat trip to Easter Egg Rock, we’ll feel tiring, yet stimulating D An all-day boat trip to Easter Egg Rock is tiring, yet stimulating

70 professor/ lecture/ longer/ usual/class hour

A The professor lectured far longer than her usual class hour B The professor lectured her usual class hour longer

C The professor lectured far longly than her usual class hour D The professor’s lecture longer than her usual class hour

III For each of the following ,write a new sentence as similar as possible in meaning to the original sentences, using the word given in bracket These words must not be altered in any way.

71 No one has explained why our flight is delayed (reason) No one has ……… the delay to our flight

72 I’d rather you didn’t phone me at work.(prefer) I’d ……… me at work

73 When Mary wanted a new car, she had to save up a year.(Mary) It ……… save up for a new car

74 They are letting David out of hospital next week (released) David ……… hospital next week

75 “Did you leave a tip for the water?” I asked.(he) I asked my father ………

76 “Why don’t you wait by the telephone box, Brenda?’ said Leslie (Brenda) Leslie suggested ……… by the telephone box

77 All the witnesses said that the accident was my fault.( blame)

All the witnesses said that ……… the accident I was to blame for 78 I’m sorry I missed Professor Baker’s lecture.( attended)

I’m sorry not ……… to have attended 79 We couldn’t find George anywhere (found)

George was ……… no where to be found

80 That’s a lovely new dress , Jean”, said her mother (her) Jean’s mother complimented ………

ĐỀ KHẢO SÁT HSG THPT KIM THÀNH LẦN NĂM HỌC : 2008-2009

MÔN: TIẾNG ANH THỜI GIAN : 180 phút A LISTENING

1

B PHONETICS

I Choose the word whose underlined part is pronounced differently from those of the others.

6 A essential B informant C resistance D instant

7 A father B healthy C other D another

II Choose the word that has a different stress pattern from the others

(65)

9 A complete B command C common D community

10 A journal B mourning C calcium D although

C GRAMMAR AND VOCABULARY

I Choose the word or phrase which best completes each sentence. 11 I won’t these excuses any longer! I demand to see the manager

A put up with B put off again C put up for D put off with

12 I never _ any experience of living in the country

A had B wished C done D made

13 Many elderly people have to live on the money they _ when they were working

A laid up B put back C set up D put aside

14 According to a recent survey, most people are on good _ with their neighbors

A relations B relationships C acquaintance D terms

15 We must comply _ the request

A in B at C on D with

16 It is believed causes insomnia

A too much caffeine B that too much caffeine C it is too much caffeine D too much caffeine that

17 Mr Harrison is _ he owns many palaces

A so a rich man that B such an rich man that C such a rich man that D that so rich a man 18 Goddard develop the first rocket faster than sound

A flying B to fly C flew D has flown

19 continental crust older than 200 million years

A It is nowhere the B Nowhere is the C Is nowhere the D Is the nowhere 20 I’ll offer it to Tom _, he may not want it

A Even though B As C When D However

II Give the correct form of the words in brackets -Luyen Tu-page 44 21 This screw needs (TIGHT) tightening

22 Scientists consider it _ that gods create volcanic eruptions (BELIEVE) unbelievable 23 We brought back several carved _ statues from our holiday in Africa (WOOD) wooden 24 The accommodation was _ and comfortable (SPACE) spacious

25 There’s no easy _ to this problem (SOLVE) solution 26 His hands were wet and _ (SLIP) slippery

27 We were lucky enough to get tickets for the first _ of the hit musical (PERFORM) performance 28 What she did was so _ that I can’t hardly describe it.(SHOCK) shocking

29 She was shocked by the _ that he was already married (REVEAL) revelation 30 The palace is closed for work (RESTORE) restoration

III Choose the underlined word or phrase in each sentence that needs correcting Cac dang TN TA NC -page 235

31 Geochemistry includes the study of(A) the movement of elements(B) from one place to another as a(C) result of process chemical(D)

32 Wind is the motion that occurs(A) when lighter(B) air rises and cools(C) heavier replaces it 33 Oceans of the world exerts(A) strong influences on(B) the weather over(C) the Earth’s surface(D) 34 Kiwi birds search(A) the ground with the(B) bill for insects(C), worms and snails to eat(D) 35 They are(A) going to have to(B) leave soon(C), and so do(D) we

36 All the students(A) are(B) looking forward spending(C) their free time relaxing(D) in the sun this summer

37 If protect(A), a solar(B) cell lasts(C) for a long time and is a good source of(D) energy

38 The growth rate(A) of the Pacific Rim countries is five times(B) fast as comparable(C) areas during(D) the Industrial Revolution

39 Drug abuse have(A) become one of(B) America’s most(C) serious social problems(D)

40 In 1903, when(A) Wright brothers announced they had invented(B) a flying machine,

his(C)news was generally(D) ignored.

D READING

I Read the following passage and decide which option A, B, C or D best fits each sentence. Cac dang TN TA NC -page 242

(66)

It is obvious that (48)… the development in the nineteenth century, of the new definition of “standard English” particular uses of the common language have been taken and (49) for the purposes of class distinction Yet the dialect which is normally (50) with standard English has no necessary (51) over other dialects Some of its grammatical elements have a common importance,

(52) not all of them On the other hand, certain selected sounds have been given an authority which (53) from no known law of language but simply from the (54) that they are (55) … made by persons who, for other reasons, possess social and economic influence

41 A shift B alternative C switch D conversion

42 A born B innate C inherited D handed

43 A influential B powerful C able D crucial

44 A positive B crushing C valuable D vital

45 A decrease B decline C drop D slip

46 A notify B clarify C illustrate D exhibit

47.A drawback B disadvantage C harm D curse

48 A since B for C when D as

49 A abused B overused C worn D torn

50 A parallel B matched C equated D equal

51.A excellence B good C rightness D superiority

52.A but B and C either D still

53 A derives B excludes C inherits D deprive

54 A idea B fact C belief D theory

55.A daily B traditionally C habitually D practically

II From the words given in the box, choose the most suitable word for each blank There are more words than blanks, so you don t need all of them.

DIALECTS

about difference words write use within part spoken variation thing

Language have dialects A dialect is a (56) of a language This means that there are differences (57) _ a language For example, French is spoken in Pairs There is a slightly different French in Canada The French spoken in Africa is different However, speakers of these three dialects can understand each other Usually, a dialect is a variety of a language that other speakers of the language can understand; dialects are usually mutually intelligible

Not all dialects of Chinese are mutually intelligible Chinese has many dialects that speakers of Mandarin and Cantonese cannot communicate with each other in the (58) _ language To communicate they must write to each other or use a second language Still, these two Chinese languages are dialects because China is one country and one culture with one language

The differences between American English and British English are not as great as the differences in Chinese dialects Two of the differences between American English and British English are vocabulary and spelling In American we say elevator, truck and apartment The British say lift, lorry and flat Another difference is spelling In American English we (59) _ honor, meter, and realize In British English these words are honour, metre ,and realise.

Dialects are an interesting (60) _ of language study All languages have dialects Usually speakers of one dialect can understand another speakers of another dialect

III Read the passage and choose the correct answer.

Children who appear to be intelligent and have normal sight and hearing may nevertheless have

learning disabilities such as dyslexia, difficulty in reading; dysgraphia, difficulty in writing; dyscalculia, difficulty with numbers; and auditory- memory problems that prevent the child from remembering what has just been said Considered an “invisible” handicap, such learning disabilities can be detected by alert parents before the children go to school If a child at about thirty months is not developing normal language skills, something amiss A child who cannot puzzles or put pegs in holes lack perceptual- motor skills Kindergartners should recognize the ABCs First- graders may commonly reverse their letters, writing a d for a b, but if they are still doing this at the start of the second grade, they should be tested for learning disabilities Proper and early treatment is essential

61 The author’s intent in this selection is to _ A describe the various types of learning disabilities B explain why some children have dyslexia

C warn parents of the signs of learning disabilities D describe kindergartner’s skills

62 The selection would most likely appear in _

A health book B parents magazine’ C medical journal D college yearbook

63 A child who can’t remember a long question might be _

A dyslexia B dysgraphia C auditory-memory problem D hyperactivity 64 The author emphasizes the need for _

A listening to children B more learning centers to help the disabled C trained personnel to prevent learning disabilities

(67)

65 A child who reads from right to left may have _

A a poor diet B poor vision C inadequate food D dyslexia E WRITING

I A, B, C and D are four sentences which are written using the given words Choose the most appropriate sentence Cac dang TN TA NC -page 237

66 salesman/ try/ talk/me buy/ car

A The salesman tried to talk to me into buying the car B The salesman tried to talk me buying the car

C I bought the car because the salesman tried it a talk D The salesman tried to talk me into buying the car

67 Marie Curie first scientist/ win two Nobel Prize/ science

A Marie Curie won two Nobel prizes and the first scientist in science B Marie Curie was the first scientist to win two Nobel Prizes in science C Marie Curie was the first scientist winning two Nobel Prize in science

D Marie Curie became the first scientist who won two Nobel Prize about science 68 Their insomnia/ not cause/ high altitude/ excitement

A Their insomnia was not the cause of the high altitude but of excitement

B Their insomnia was not caused from the high altitude but also from excitement C The high altitude and excitement was not caused their insomnia

D Their insomnia was not caused by the high altitude but by excitement 69 fact/ she/ work/ long hours/ spend/ a lot / time/ family

A In spite of the fact that she works long hours, she spends a lot of time with her family B In fact, she works long hours, and spending a lot of time with her family

C Although the fact that she works long hours, she spends a lot of time with her family D The fact that she works long hours makes her spends? lot of time 'with her family 70 build/ hand/ car/ superb condition

A Building by hand , the car is in a superb condition B Having been built by hand, the car is in superb condition C When built by hand, people put the car in superb condition D As built by hand, the car was superb condition

71 There/ twice/ work /do

A There is twice as much work to B There is twice work to

C There is twice much work to be done D There are twice as work to

72 new regulation/ effect/ no/ extensions/ give

A The new regulation effect as no more extensions will be given B As the new regulation in effect, no more extensions will be given

C After the new regulation goes into effect, no more extensions will be given D Because of the new regulation, the effects of no extensions will be given 73 national parks/ United States/ Yellowstone/ most/ visit

A The national parks in the United States such as Yellowstone most visited B Of all the national parks in the United States, Yellowstone is the most visited C Of all the national parks in the United States, Yellowstone is most visited D The national parks in the United States like Yellowstone are most people visit 74 sooner/ we arrive/ music / start

A No sooner we arrived than the music started B No sooner did we arrive than the music started C The sooner we arrived when the music started D No sooner did we arrive when the music started 75 Half/ members/ we / expect/ show/ last meeting

A Half the members as we expected were shown up at the last meeting B We expected half of the members show at the last meeting

C The show at the last meeting was half of the members we expected D Half of the members we expected showed up at the last meeting

(68)

76 Joan eats very little so as not to put on weight

Joan eats very little because ……… 77 On arrival at the shop, the goods are inspected carefully When the goods………

78 Laurence hasn’t seen Peter since he left for Japan Laurence last ………

79 Peter said he wasn’t feeling very well Peter said, “ ……… ” 80 John is fat because he eats so many chips If ………

81 “ You should take more exercise, Mr Robert,” the doctor said, “ if you want to lose weight.” The doctor advised………

82 Collecting dolls from foreign countries is one of Janet’s interest Janet is ………

83 George is not nearly as energetic as he used to be George used ………

84 If Joe doesn’t change his way, he will end up in prison Unless ………

85 The boy’s very pleased to be going away on holiday soon The boy’s looking………

III Complete the sentences using the cues - Luyen Viet page 56 Dear Mr Salt,

81 I / very interested/ read/ your article/ global warming/ the newspaper. 82 I / afraid/ I not agree/ you/ number/ points.

83 Firstly/ you say/ some/ scientists/ not believe / theory. 84 Who/ be/ these scientists?/ I/ never/ hear then.

85 Secondly/ it/ not true/ global warming/ receive/ too/ much/ publicity. 86 On/ contrary/ we/ need/ make/ public/ aware/ such danger.

87 We/ must/ all put/ pressure/ authorities/ if / we/ want/ something/ do 88 Lastly/ you/ seem/ suggest/ do/ nothing/ case/ scientist/ wrong. 89 Well/ if/ they / right/ it / too late/ by/ time we/ find out.

Yours truly, Dr Pepper

EXTRA EXERCISES - September 1st , 2008

I Sentence transformation :Finish each of the following sentence in such a way that it means exactly the same as the sentence printed before it.

(69)

6 Mary is the most talented student in my school => No student ………

7 The furniture was so expensive that I didn’t buy it => The furniture was too ……… He learnt to drive when he was eighteen => He has ………

9 I would like you to help me to put the chairs away => Do you mind……… 10 He would prefer you to pay him immediately => He’d rather ……… 11 An up-to-date visa is necessary for Andorra => You’ll ……… 12 “Why don’t you put a better lock on the door, Barry?” said John => John suggested …… 13 I haven’t eaten this kind of food before => This is the first ………

14 The architect has drawn plans for an extension to the house => Plans ……… 15 In Stratford -on- Avon we saw Shakespeare’s birthplace => We saw the house ……… 16 It isn’t necessary for you to finish by Friday => You ……… 17 It is essential that Professor John Brown is met at the airport => Professor John Brown ………… 18 Are you sure that jacket is the right size for you? => Does that jacket ……… 19 I gave Ted the message, but he already knew about it => I needn’t ………

20 He is a fascinating lecturer because he knows a lot about his subject => His wide knowledge …… II WORD FORMS

1 Heavy smoking makes him ( breath)

2 I have only a limited of computer programming ( know)

3 You cannot _ about the effects of the drug from the cases of one or two patients (general) 4 Japanese cars are known for their _ (rely).

5 He was always _ to his father’s wishes (obey)

6 Most of the students are making good progress but Jeremy seems a _ case ( hope). 7 We were by what we saw (horror)

8 They left the house in a _ mess (fright)

9 The children were surprised at the sudden _ of their teacher (enter) 10 You are not _ to unemployment benefit if you have never worked (title) 11 They all cheered _ as their team came out ( enthusiasm)

12 Finally a _ is brought about between the two sides( reconcile) 13 Their office equipment needs to be _ ( modern)

14 He will not benefit from the deal ( finance)

15 The inquiry _ was of her handling of the affair ( criticize) 16 Her case was argued ( convince)

17 I don’t find him very _ (communicate) 18 I always more at night ( care)

19 My colleagues are very pleasant , but the manager is a little _ (friend) 20 It seems _ to change the time so often ( logic)

III WRITING: Complete the letter using the words given Dear Carol,

81 Thank you very much/ invitation/ spend/ three weeks/ August / you/ your family/ Scotland 82 I / love/ come/ unfortunately/ I / be/ unable/accept

83 My cousins, / live/ Canada,/ stay/ us/ August 3rd / August 18th

84 As it/ be/ first visit/ this country, / I plan/ show/ them around 85 it/ be/ possible/ visit you/ September/ instead?

86 I certainly/ need/ rest/ after/ cousins/ go back/ Canada 87 Please tell/ September /be convenient

88 I / look forward/ see you all again

1 Anthropologists agree(A) that our primitive ancestors(B) who inhabited the tropics probably have(C) natural protection against(D) the sun

(70)

lives

3 A thunder(A) usually follow lightning by five seconds(B) for every mile(C) between(D) the flash and the observer

4 Forgery, in law, is the fabrication or altering(A) of a written(B) document with(C) the intent to deceive or defraud(D)

5 During the first half of the 19th century, immigrants(A) to the United States were predominant(B) from

Western Europe; after the Civil War, however, new arrivals came mainly from(C) Eastern and Southern Europe, as well as(D)from Asia

6 Bill Gates built his microcomputer software company(A) into one of the largest in the nation(B), and in doing so (C)became one of the country’s wealthiest and most respected man(D)

7 With his many theories, Albert Einstein did(A) a great impact on physics, so much so(B) that he is often called(C) the greatest physicist of all(D) time

8 Since rats are destructive(A) and may carry(B) diseases, therefore many(C) cities try to exterminate them(D)

9 In that age(A) of computer, it is(B) difficult to imagine how tedious the(C) work of accountants and clerks must have been(D) in the past

10 Made up of more than(A) 150 member countries, the organization known(B) as the United Nations were(C) established after World War II to preserve(D) international peace and security

EXTRA EXERCISES - September 4th , 2008 I ERROR CORRECTION: Choose the underlined part that needs correcting

1 The dense Belgian fogs, like the most(A) inland fogs, are caused by(B) the cooling of humid surface air(C) to a relatively(D) low temperature

(71)

3 Dinosaurs were(A) just beginning(B) to establish(C) them(D) 200 million years ago

4 If unwanted pet turtles are unleashed(A) into the wild(B), many will die and those which(C) exist will threaten the lives of native plants and animals(D)

5 Close to(A) 73 percentage(B) of the United States’ population is concentrated(C) in metropolitan areas, and more than(D) half the population live in the South and West

6 Among(A) the most remarkable eyes are those of the dragonfly(B); for this insect(C) has compound eyes make up(D) of tiny eyes

7 Our civilization is commonplace(A) to us that(B) rarely we stop(C) to think about its complexity(D) Between(A) mountains and the plateau lies(B) the great Valley, which is rich in(C) fertile crop lands and apples orchards(D)

9 During World War II(A), many Eskimos served in the Army or Navy(B); another(C) worked on all field or supplied meat to the armed force(D)

10 Every city in the United States(A) has traffic(B) problems because the amount(C) of cars on American streets (D)and highways is increasingly every year

II WORD FORMS

1 He doesn't know much about the subject, but he IS very (ENTHUSIASM) The charity is totally upon money from the public (DEPEND)

3 The bank has been very in helping me sort out my late husband's finances (COOPERATE)

4 Is military service in your country? (COMPEL) It was of you not to offer her a drink (HOSPITABLE) The carpet is available in various (WIDE)

7 The plane grounded because of poor (VISIBLE) He resigned for a of reasons (VARIOUS)

9 He wrote so many sentences in his essay (GRAMMAR)

10 They daren't leave their children for even a moment (ATTEND) 11 On a day we receive about fifty letters (TYPE)

12.They had a large over the other party at the last election (MAJOR) 13 , he survived the crash without injury (MIRACLE)

14 He is very generous and everyone admires his (SELF) 15 He wrote a letter to me (THREAT)

16 He was enormously when my father died (SYMPATHY) 17 They witnessed the miraculous of some people in the air crash

(SURVIVE)

18 Help is immediately sent to the of the earthquake (SURVIVE) 19 The police appealed to the crowd for _ (RESTRAIN) 20 To be honest, I find the film rather (OFFEND)

III WRITING: Complete the sentences using the cues Dear Mr Salt,

81 I / very interested/ read/ your article/ global warming/ the newspaper 82 I / afraid/ I not agree/ you/ number/ points

83 Firstly/ you say/ some/ scientists/ not believe / theory 84 Who/ be/ these scientists?/ I/ never/ hear then

85 Secondly/ it/ not true/ global warming/ receive/ too/ much/ publicity 86 On/ contrary/ we/ need/ make/ public/ aware/ such danger

87 We/ must/ all put/ pressure/ authorities/ if / we/ want/ something/ 88 Lastly/ you/ seem/ suggest/ do/ nothing/ case/ scientist/ wrong 89 Well/ if/ they / right/ it / too late/ by/ time we/ find out

Yours truly, Dr Pepper IV READING

II From the words given in the box, choose the most suitable word for each lank There are more words than blanks, so you don’t need all of them.

INSURANCE PREMIUMS AND PROBABILITY

knows collect calculate age charge protection chance profit insures company

(72)

example, insurance companies (2) _statistics which show how many people dies at what (3) _ each year or how many fires there are each year The insurance company uses these statistics to (4) _

insurance premiums Premiums are the amount of money that an individual or business pays for insurance Here is an example of how the (6) _ calculates the premiums It (7) _ that percent of the people who are 50 years old will die next year If the company (8) _ 1,000 people who are 50 years old, that know that 20 will die If the twenty people are insured for $100,000 each, then the company must pay $2,000,000 Therefore, they will (9) _ the 1,000 insured people $2,000,000 plus the costs and (10) _ for the company This is a very simple example, but all insurance premiums are based on the probability or odds of something happening

ELECTRONIC FUNDS TRANSFER

bills buys employees employer carry deposit customer system pay receives bank withdraw

In the future, machines will take the place of many bank tellers A new (1) _, electronic funds transfer (EFT) allows the bank or the (2) _ to move money from one account to another For example, a worker (3) _ her money paycheck, her salary Her (4) _, the company that she works for, can (5) _ her salary into the checking account directly She does not have to go to the (6) _ Her bank can also pay her monthly (7) _: the telephone, the water, the gas, and the electricity It will also be possible for her to pay for food at the supermarket by EFT The supermarket will automatically (8) _ money from her account at the bank to pay for the food that she (9) _

Some employees already deposit their (10) _ checks in the bank directly Maybe, in the future, people will not (11) _ money, and machines will (12) _ for everything

IV WRITING: Rewrite the sentence using the word in the blanks These words must not be altered in any way.

1 It would be difficult for me to finish the work by the weekend (difficulty) I ……… the work by the weekend

2 Harry’s home is still in Spain, is it? (lives) Harry ……… he?

3 When Sandra walked out of the meeting, she didn’t say goodbye to anyone (without) Sandra left ……… goodbye to anyone

4 You can borrow my bike if you are in a hurry (mind) I ……… you my bike if you are in a hurry

5 Angus rarely takes a holiday (rare) It ……… take a holiday

6 we lost the game because of my mistake (fault) It was …… win the game

7 Are you planning to anything on Saturday? (plans) Do ………… Saturday?

8 Tim looks nothing like his father (take) Tim ……… his father at all

9 The film I saw last week was better than this one (good) This film ……… the one I saw last week 10 I regret giving Dennis my phone number (Dennis) I wish ……… my phone number

ĐỀ KHẢO SÁT HSG THPT KIM THÀNH LẦN NĂM HỌC : 2008-2009 MÔN: TIẾNG ANH

(73)

Listen and choose the correct answer for each question May Balls are organized _

A in June B in May C on Saturday D August

2 At a May Ball, students often wear _

A diner jackets and bow ties B old dresses C jeans and T-shirts D All of the above

3 What is “ The Bath”?

A A public house B A student public bathing place C The name of a river D A rowing competition

4 In “ The Bumps” , each boat tries to _

A get wet B overtake their competitors C go along the river D bump into

other boats

5 Maypole is an English traditional dance often held in _

A the city B the countryside C all parts of England D winter

B PHONETICS

I Choose the word whose underlined part is pronounced differently from those of the others.

6 A Christmas B mechanic C machine D chronology

7 A encounter B enter C enlarge D entrust

II Choose the word that has a different stress pattern from the others

8 A amount B metal C intend D opinion

9 A utterance B attendance C performance D reluctance

10 A support B colleague C bilingual D evaluate

C GRAMMAR AND VOCABULARY

I Choose the word or phrase which best completes each sentence. 11 The police finally arrested the _ criminal

A famous B renowned C respectable D notorious

12 Warmth, moisture, and oxygen are three necessary requirements _ most seedlings A can cultivate B for cultivate C as cultivate D for cultivating 13 She _ for lost time by studying at weekends

A got up B put in C set about D made up

14 Your wish coincided _ mine in this situation

A to B with C within D on

15 in front of a camera lens changes the color of the light that reaches the film A Placed a filter B A filter is placed C A filter placed D When a filter placed 16 Sound comes in waves, and the higher the frequency, _

A higher is the pitch B the pitch is higher C the higher the pitch D pitch is the higher 17 She was too far away, so she _ you

A mustn’t see B hasn’t seen C couldn’t have seen D hadn’t seen

18 Out _ from its tiny cage

A did the bird fly B fly the bird C the bird flew D flew the bird

19 It sounds like something wrong with the car’s engine , we’d better take it to the garage immediately

A Otherwise B Without it C If not D If so

20 No matter _ hard he tries, he never seems _ to the work satisfactorily

A what/able B how/ able C when/enable D how/enable

II Give the correct form of the words in brackets.

21 These quantities are for the order of orders received (sufficient) 22 I’m not saying he is _, but he is not very good with money (honest) 23 The _ she gave last night was marvelous (perform)

24 She has one of the biggest _ in Britain ( collect)

25 They managed to find three miners who were still _ three days after the underground explosion.( live)

26 Cats are supposed to have nice _ (live) lives

27 “ Look after your mother,” were his _ words (die) dying

28 He claimed that his _ had caused him to become a criminal.( bring up) upbringing 29 It was a difficult _ and she almost lost the baby (pregnant)

30 The government had promised to deal with the problems of _ among young people (employ) III Choose the underlined word or phrase in each sentence that needs correcting

31 Drying(A) foods by means(B) of solar energy is a(C) ancient process applied(D) wherever food and climatic conditions make it possible

32 Bill Gates built his microcomputer software company(A) into one of the largest in the nation(B), and in doing so (C)became one of the country’s wealthiest and most respected man(D)

(74)

34 Most babies will grow up to be as cleverer as their parents

35 It should not be assume(A) that the lower the(B) price, the happier(C) the buyer(D)

36 One out(A) of every(B) eight balloon in(C) the world are launched(D) at Albuquerque, New Mexico 37 Anyone reproducing(A) copyrighted works without permission of the holders of the copyrights are(B) breaking(C) the law(D)

38 America’s first satellite(A) exploded before(B) it had risen(C) three and a half feet(D) off the ground 39 With(A) animals both(B) humans, chewing(C) helps relieve(D) tension

40 Anthropologists agree(A) that our primitive ancestors(B) who inhabited the tropics probably have(C) natural protection against(D) the sun

D READING

I Read the following passage and decide which option A, B, C or D best fits each sentence.

If you are an environmentalist, plastic is a word you tend to say with a sneer or snarl It has become a symbol of our wasteful, throw-away society But there seems little(41) that it is here to stay, and the truth is, of course, that plastics have brought enormous (42) _,even environmental ones It is not really the plastics themselves that are the environmental (43) _ - it’s the way society chooses to use and (44) _ them

Almost al of the 50 or so different kinds of modern plastics are made from oil, gas , or coal -nonrenewable natural (45) _ We (46) _ well over three million tons of the stuff in Britain each year and sooner or later, most of its is thrown away A high (47) _ of our annual consumption is in the (48) _ of packaging, and this (49) _ about seven percent by weight of our domestic (50) _ Almost all of it could be recycled, but very little of it is, though the plastic recycling (51) _ is growing fast

The plastics themselves are extremely energy-rich , they have a higher caloric (52) _ than coal and one (53) _ of “recovery” strongly (54) _ by the plastic manufacturers is the (55) _ of waste plastic into fuel

41 A evidence B concern C doubt D likelihood

42 A pleasures B benefits C savings D profits

43 A poison B disaster C disadvantage D evil

44 A dispose B store C endanger D abuse

45 A resources B processes C products D fuels

46 A remove B import C consign D consume

47 A portion B amount C proportion D rate

48 A way B kind C form D type

49 A takes B makes C carries D constitutes

50 A refuse B goods C requirements D rubble

51 A manufacture B plant C factory D industry

52 A degree B value C demand D effect

53 A mechanism B measure C method D medium

54 A desired B argued C favored D presented

55 A conversion B melting C change D replacement

II From the words given in the box, choose the most suitable word for each blank There are more words than blanks, so you don’t need all of them.

operations automatic counterparts similarly microcomputer components partners unlike

INDUSTRIAL ROBOTS

One step beyond automated machines is the industrial robot, the heart and brain of which is the (56) _ (57) _ most automated machines, industrial robots can be programmed to a variety of tasks that are usually accomplished by human factory workers Like their human (58) _, industrial robots can be switched from one job to another and can be programmed to handle new tasks Thus far, robots have found their greatest use in assembling (59) _ However, they are swiftly branching from basic assembly (60) _ to construction and mining, and their most glamorous use of all, the exploration of oceans and outer space

III Read the passage and choose the correct answer.

The food we eat seems to have profound effects on our health Although science has made enormous steps in making food more fit to eat, it has, at the same time, made many foods unfit to eat Some research has shown that perhaps eighty percent of all human illnesses are related to diet and forty percent of cancer is related to the diet as well., especially cancer of the colon People of different cultures are more prone to contract certain illnesses because of the characteristic foods they consume

That food is related to illness is not a new discovery In 1945, government researchers realized that nitrates and nitrites (commonly used to preserve color in meat) as well as other food additives caused cancer Yet, these carcinogenic additives remain in our food, and it becomes more difficult all the time to know which ingredients on the packaging labels of processed food are helpful or harmful

The additives that we eat are not all so direct Farmers often give penicillin to cattle and poultry, and because of this, penicillin has been found in the milk of treated cows

(75)

A healthy diet is directly related to good health Often we are unaware of detrimental substances we ingest Sometimes well-meaning farmers or others who not realize the consequences add these

substances to food without our knowledge 61 How has science done a disservice to people?

A Because of science, disease caused by contaminated food has been virtually eradicated B It caused a lack of information concerning the value of food

C As a result of scientific intervention, some potentially harmful substances have been added to our food. D The scientists have preserved the color of meats, but not of vegetables

62 The word “ prone” is nearest in meaning to _.

A supine B unlikely C healthy D predisposed

63 What are nitrates used for?

A They preserve flavor in packaged foods B They preserve the colour of meat.

C They are the objects of research D They cause the animals to become fatter 64 FDA means _

A Food Direct Additives B Final Difficult Analysis

C Food and Drug Administration D Federal Dairy Additives 65 The word “ these” refers to

A meats B colors C researchers D nitrates and nitrites

66 The word “ carcinogenic” is closest in meaning to _.

A trouble-making B color-retaining C money-making D cancer-causing

67 Al of the following statements are true except _ A Drugs are always given to animals for medical reasons.

B Some of the additives in our food are added to the food itself and some are given to the living animals C researchers have known about the potential hazards of food additives for more than forty-five years D food may cause forty percent of the cancer in the world

68 The word “additives” is closest in meaning to _.

A added substances B dangerous substances C natural substances D benign substances 69 What is the best title for this passage?

A Harmful and Harmless Substances in Food B Improving Health through a Natural Diet C The Food You Eat Can Affect Your Health. D Avoiding Injurious Substances in Food 70 The word “ fit” is closest in meaning to _.

A athletic B suitable C tasty D adaptable

E WRITING

I A, B, C and D are four sentences which are written using the given words Choose the most appropriate sentence.

71 see/ telescope/ planets/ take/ new appearance.

A Sees through a telescope, the planets take up a new appearance B Seen through a telescope, the planets take on a new appearance C Seeing through a telescope, the planets take after a new appearance D To see for a telescope, the planets take on a new appearance

72 some people/ believe/ humans/ never/ use/ natural resources/ Earth.

A Some people believe that humans will never use away the natural resources of Earth B Some people are believed to be humans will never use up the natural resources of Earth C Some people believe that humans will be never used up the natural resources of Earth D Some people believe that humans will never use up the natural resources of Earth 73 conditions/ necessary/ completion/ this project/ not meet.

A The conditions necessary for the completion of this project have not been met B The conditions necessary for the completion of this project have not met C The conditions necessary of completion of this project have not been met D The conditions necessary for completion of this project were not met 74 first step/ scientific research/ decide/ gather/ data.

A The first step in a scientific research is to decide gathering data B The first step to scientific research decides to gather data C The first step of scientific research is to decide how to gather data D The first step to gather data is decided scientific research

75 there/ no trace/ poison/ coffee/ chemist/ analyze.

A There was no trace for poison of the coffee the chemist analysed B There were no traces in poison of the coffee the chemist was analysed C There was no trace of poison in the coffee the chemist analysed

D There was no trace of poison in the coffee the chemist has been analyzed

II Finish each of the following sentence in such a way that it means exactly the same as the sentence printed before it.

76 We were late because it rained heavily => But for………

77 We’ll stop now unless you have any further questions

(76)

=> Much ………

79 I dislike it when people criticize me unfairly

=> I object to people criticizing me / my being unfairly criticized ……… 80 I only recognized him when he came into the light

=> Not until ………

81 John only understood very little of what the teacher said => John could hardly ……… 82 Unless someone has a key, we can’t get into the house => We can only ……… 83 I’m sure you didn’t lock the front door Here is the key =>You can’t ……… 84 “Bring your swimming things in case it’s sunny.” => He told ………

85 Tim will be eighteen next week => It is Tim’s………

III Essay

Write an essay (from 200 to 250 words) about the following topic: The advantages and disadvantages of living in a big city.

ĐỀ KHẢO SÁT HSG THPT KIM THÀNH LẦN NĂM HỌC : 2008-2009 MÔN: TIẾNG ANH

THỜI GIAN : 150 phút

A LISTENING: Listen to a talk about hobbies and fill in the blanks Each blank needs ONE word only.

The word hobby is a shortened (1) of hobbyhorse, which has a decorated wooden framework with an imitation horse’s head attached to it Hobbies today include a vast range of activities In this sense, hobbies include games and sports, but they leave out purely spectator activities like watching television They also exclude

(2) and work done to make a living A hobby, like playing with a hobbyhorse, is an activity (3) _ from the ordinary (4) of life It should (5) the use of creativity and imagination and bring the reward of learning Some hobbies bring monetary rewards as well

B PHONETICS

I Choose the word whose bold part is pronounced differently from those of the others.

6 A attraction B construction C congestion D satisfaction

7 A danger B singer C manager D merger

II Choose the word that has a different stress pattern from the others

(77)

9 A environment B electricity C population D ecotourism

10 A organize B encourage C satisfy D operate

C GRAMMAR AND VOCABULARY

I Choose the word or phrase which best completes each sentence. 11 You must forgive my in these matters

A experience B inexperience C unexperience D imexperience

12 Vietnam was _ nation to host SEA Games 22

A the

B a

C

Ø

D one

13 The fire in our neighborhood last night burned down about 20 houses It’s that the police were not informed in time

A unsuited B inconvenient C alarming D regrettable

14 Although they had only been invited for lunch, they until supper time

A stayed on B stayed out C stayed up D stayed in

15 What a !

A new small nice car B nice small new car C new nice car, that is small D car new nice and small

16 _, we tried our best to complete it

A Difficult as the homework was B Despite the homework was difficult C Thanks to the difficult homework D As though the homework was difficult 17 The surface of the moon was shaped by meteorites craters of all sizes

A that impact formed B which forming C whose impact formed D are forming 18 The Roman used central heating systems very much like _

A those of today B today’s C those they D the systems which are

now

19 This scientific theory _ to be false

A had now proved B has now proved C has now been proved D had now been proved

20 Such that he would stop at nothing

A his ambition was B did his ambition C does his ambition D was his ambition II Give the correct form of the words in brackets Luyen tu 55

21 When did the toy come into ? (EXIST)

22 A lot of toys encourage children’s (IMAGINE)

23 He took the to turn painting into a full-time career (DECIDE) 24 He will be able to receive an _ from a government agency (ALLOW) 25 “Video recorders have _ my life,” he said (REVOLUTION)

26 It’s a idea for children to learn a foreign language at an early age (MARVEL) 27 The newly-built cinema shows a _ of films (VARY)

28 The factory has provided cheaper lately (PRODUCE)

29 The technology of computer science is having a _ effect on our lives (DRAMA) 30 is forcing employers to cut back the labour force (COMPETE)

III Choose the underlined word or phrase in each sentence that needs correcting

31 Because not food(A) is as nutritious(B) for a baby as(C) its mother’s milk, many(D) women are returning to the practice of breast feeding

32 There are(A) many different ways of comparing(B) the economy of one nation with those(C) of another(D)

33 Until diamonds are cut and polished(A), they(B) just like look(C) small grey stones(D) 34 I wish I have(A) a car It would(B)make life so much(C) easier(D)

35 Will(A) you like(B) something(C) to eat(D)?

36 Harvey never pays his bills(A) on time(B), and(C) his brother does too(D)

37 Drug addiction has(A) resulted of(B) many destroyed(C) careers, and expulsions(D) from school or college

38 The(A) train was supposed(B) arriving(C) at 11.30, but it was an hour late(D) 39 I don’t like(A) people told(B) me(C) what to do(D)

40 For more than(A) 450 years, Mexico City has been the economic, culture(B), and political center of the(C)Mexican people(D)

D READING

I Read the following passage and decide which option A, B, C or D best fits each sentence BTNC 203

The point at which physical decline with age begins adversely to affect a driver’s capability has not yet been thoroughly studied A survey of more than 3,000 road accidents in Michigan involving drivers aged over 55 showed that in eight out of ten (41) _ it was a driver over the age of 71 who had (42) _ collision by failing to yield, turning carelessly or changing lanes

Older drivers are obviously more (43) _ to injury in vehicle crashes, as well as being a potential higher (44) _ through their own driving (45) _

Reactions (46) _ in an emergency involves many different physical (47) _ such as the production of the nerve impulse, perception of the signal, (48) of response and transmission to the muscles

(78)

Part of the aging process, however, does include the (51) _ of experience, often in the

subconscious, which triggers (52) _ danger warnings than in younger drivers who have not experienced similar situations

This (53) of judgment heightens the perception of risk and often (54) _ older drivers to avoid a situation which might then (55) them to the test

41 A users B points C cases D attempts

42 A avoided B prevented C caused D activated

43 A likely B susceptible C possible D common

44 A degree B chance C factor D risk

45 A practice B activity C experience D behavior

46 A period B time C process D system

47 A events B parts C factors D forms

48 A choice B suggestion C section D preference

49 A improve B deteriorate C reduce D increase

50 A mature B ancient C older D elderly

51 A collection B addition C storage D summary

52 A sooner B earlier C former D later

53.A lack B maturity C absence D strength

54 A follows B progresses C leads D pulls

55 A fix B force C enable D put

II From the words given in the box, choose the most suitable word for each blank There are more words than blanks, so you don’t need all of them

printing interesting value vary change important plastic values precious MONEY

When we think of money, we usually think of currency, or metal coins and paper bills In the modern world, almost every country and every person uses coins and paper money to exchange for other objects of (56) _ The sizes and shapes of coins are different in various countries, and the size and color of paper money also (57) _ In India, for example, some coins have square sides In Japan, some coins have holes in the centre In the United States, all paper money is the same size and the same colour; only the (58) _on the bills is different

Until the twentieth century, most coins were made of (59) l metals such as gold and silver, or they contain at least some gold or silver By the middle of the twentieth century, all coins that were actually being used had no gold or silver and were made of less expensive metals, such as copper or aluminum

Paper money has replaced metal money for most purposes, and now paper money is being replaced by paper checks, (60) _ credit cards, and even electronic messages If this continues, we may soon find that we will not use money at all, but only a series of numbers in computers to buy and sell goods and services

III Read the passage and choose the correct answer Olyimpic 247

There are many causes of headaches, and most people suffer them at some time or other Although doctors have come a long way from the old days, when headaches were ascribed to evil spirits and

treatments ranged from cutting out part of the skull to concoctions of cow brain and goat dung, they are still not sure what sets off headaches

The most significant advance has been the acceptance that they are not the result of emotional stress Until recently, many doctors thought that imbalances in the body’s systems were to blame, but experts now believe it is the brain itself They point to malfunctioning chemicals, such as serotonin, whose job is to send messages to regulate the contraction and dilation of blood vessels in the brain

Monosodium glutamate, a flavor enhancer used in Chinese cooking, can cause headaches in some people, as many other common foods Red wine, aged cheese, coffee, chocolate, nuts, and preserved meats contain nitrates, caffeine, and tyramine, chemicals that may produce pounding headaches

Even though the exact culprit has yet to be found, there are plenty of treatments for prevention or cure Over-the-counter preparations such as aspirin are fine for treating the occasional headaches, but often exacerbate severe cases Beta blockers, usually used for lowering blood pressure, seem to head off

migraines Antidepressants are effective, too But doctors also recommend non-drug treatments such as relaxation techniques, which can be used in combination with medication, and diet modification, to cut out foods that cause attacks

61 According to the passage, many years ago, one way doctors tried to cure headaches was by _ A praying to spirits B sacrificing cows and goats C operating on the patient’s head D writing prescriptions

62 It is no longer believed that headaches are caused by

A emotional stress B malfunctioning chemicals in the brain C certain kinds of foods D contraction and dilation of blood vessels

63 According to the passage, doctors now believe that headaches are related to

A imbalance in the body’s systems B chemicals in the brain C emotional stress D high blood pressure

64 According to the passage, beta blockers can be used to _

A treat migraines B cause migraines C contract blood vessels D treat depression 65 According to the passage, severe headaches cannot be successfully treated by _

(79)

E WRITING

I A, B, C and D are four sentences which are written using the given words Choose the most appropriate sentence.

66 President s / greatest/ asset/ reputation/ honesty.

A The President’s the greatest asset for his reputation of honesty B The President’s greatest asset in his reputation for honesty

C The President’s greatest asset was found his reputation with honesty D The President’s greatest asset was his reputation for honesty

67 fall-off/ profit/ indicate/ something/ wrong/ the company s/ advertising policy.

A The fall-off about profits indicates that something wrong with the company’s advertising policy B The fall-off in profits indicates that something is wrong with the company’s advertising policy C The fall-off of profits indicated something to go wrong about the company’s advertising policy D The profits of fall-off indicates that something is wrong with the company’s advertising policy 68 impossible/ parents/ protect/ children/ every danger.

A It is impossible for parents to protect their children from every danger

B There was impossible so parents can’t protect their children from every danger C Parents are impossible to protect their children of every danger

D It is impossible as parents protecting their children from every danger 69 everyone/ expert/ paintings defects/ invisible

A Everyone but not the expert, the paintings’ defects were invisible B Everyone experts the paintings’ defects were invisible

C To everyone but the expert, the paintings’ defects were invisible D Experts are invisible to everyone and the paintings’ defects

70 single factor/ explain/ why / effects/ aging/ vary/ great/ individuals.

A No single factor explains why the effects of aging vary so greatly among individuals B A single factor explains why the effects f aging varies so greatly among individuals

C Single factors and explanations on why the effects of aging vary so greatly among individuals D No single factor and explanation effects why aging vary so great among individuals

II Finish each of the following sentence in such a way that it means exactly the same as the sentence printed before it Luyen Viet 145/124

71 “Will I ever find a job?” => Tom wondered ………

72 You should take a map because you might get lost in those mountains => In case ……… 73 Temperature is measured by a thermometer => A thermometer is ………

74 You remembered to post the letter, didn’t you? => You didn’t ……… 75 Mr Dryden mended the washing machine for me => I had ………

76 To get the 40% discount, you must buy all twelve books at the same time => You can only ………

77 It’s thought that the accident was caused by human error => The accident is ……… 78 If the work is finished by lunchtime you can go home => Get ……… 79 The police let him leave after they had questioned him => He was ……… 80 Do you have good relationship with your boss? => Are ……… ?

III Essay: Boi duong HSG 12 -page 126

Write a composition (350 words) about the following topic:

How movies and television influence people’s behavior? Use reasons and specific examples to support your ideas

Use the following ideas as suggestions:

(80)

ĐỀ KHẢO SÁT HSG THPT KIM THÀNH LẦN NĂM HỌC : 2008-2009 MÔN: TIẾNG ANH

THỜI GIAN : 150 phút

A LISTENING: Listen to Robert’s talk and answer the questions Your answers should not have more than three words.

1 Is Robert an university student?

………

2 When is Robert going to ask Angela to marry him? A few months ……….

3 Where are they going to live?

……… How many children are they going to have? ……… What does Robert’s father has?

……… B PHONETICS

I Choose the word whose underlined part is pronounced differently from those of the others.

6 A scared B learned C laughed D ragged

7 A vehicle B honest C heir D hospital

II Choose the word that has a different stress pattern from the others

8 A magnificent B significant C traditional D factually

9 A marvellous B attention C consider D excited

10 A property B introduce C mechanize D powerful

C GRAMMAR AND VOCABULARY

I Choose the word or phrase which best completes each sentence.

11 On the Moon air because the Moon’s gravitational field is too weak to retain an atmosphere

A there is no B where no C no D is no

12 they are tropical birds, parrots can live in temperate or even cold climate

A Despite B Even though C Nevertheless D But

13 of great apes, the gibbon is the smallest

A Four of the types B The four of types C Four types of the D Of the four types 14 Blindfish, which spent their whole lives in caves, _ eyes nor body pigments

A not any B neither C none D without

15 Released in 1915,

A D.W Griffith’s made an epic film about the Civil War, Birth of a Nation. B the Civil War was the subject of D.W Griffith’s epic film, Birth of a Nation. C D.W Griffith’s epic film Birth of a Nation was about the Civil War.

D the subject of D.W Griffith’s epic film Birth of a Nation was the Civil War. 16 His landlady doesn’t of his having parties

A appreciate B approve C support D consent

17 Hardly had the van turned the corner when one of the back wheels _

A broke away B turned C came off D rolled down

18 He didn’t know anyone at the wedding than the bride and the groom

A except B other C apart D rather = instead of/ in place of

19 It is necessary absolutely that everyone in the country his or her fair share of the taxes

A pay B pays C to pay D paying

20 I was enjoying my book, but I stopped _ a program on TV

A to read to watch B to read for watching C reading for watching D reading to watch II Give the correct form of the words in brackets Olympic 73

21 Visitors complained about the in the old museum (organize) disorganization 22 When Jenny made up for the play, she was (recognize) unrecognizable 23 He has joined the company in an capacity (advise) advisory

24 Tom was accused of stealing some documents (confidence) confidential 25 The cost of to the show is quite reasonable (admit) admission

26 That was a very _ thing to (coward) cowardly

(81)

30 I don’t like your behavior (child) childish

III Choose the underlined word or phrase in each sentence that needs correcting 31 Polar winters are(A) length(B), dark, and cold enough(C) to kill most(D) plants

32 People with an exceptionally(A) high intelligence quotient may not be the best(B) employees since they become bored of(C) their work unless the job is constantly changing(D)

33 The normally(A) force of gravity(B) at the Earth’s(C) surface is(D) called g

34 There are not many(A) people which(B) adapt to(C) a new culture without feeling some disorientation at first(D)

35 CAT scanners are used(A) not only for detecting conditions(B) but also for observation(C) the effects of therapy(D)

36 A desire(A) to eradicate irregular(B) spellings in English can being(C) traced(D) back to the sixteenth century

37 Ninety- seven percent(A) of the world’s(B)water is salt water is(C) found in the oceans(D) of the Earth 38 I’ll(A) be there(B) soonest(C) as I can(D)

39 Hardly I could(A) make out(B) what(C) the message was(D)

40 Alexander Graham Bell was(A) twenty -nine during(B) his(C) basis telephone patent was granted(D)in 1876

D READING

I Read the following passage and decide which option A, B, C or D best fits each sentence.

Stress is our body’s reaction to events or conditions that we cannot easily manage or (41) _ When we are troubled by something, we usually experience some type of (42) _ There are thousands of

conditions that (43) _ us to become tense These may be major and horrible: fighting for our lives in a war, being imprisoned, or facing death On the other hand, major (44) _ in our daily lives cause stress as well: waiting in (45) _, taking a quiz, or asking for or accepting a date Although we usually think of tress as something caused (46) _ unpleasant events, pleasant happenings also bring on stress Vacations, pay raises, and winning an award all(47) _ tension in our body

As new (48) _ is done on the effects of tress, new techniques are developed to help (49) with anxiety One aid is to plan for situations that we know will be (50) _ Some imagine the event before it happens, thus preparing their minds and bodies for the tension Other people relax or (51) _ at regular intervals to lessen the muscle strains in their bodies Still others (52) _ they are on a pleasant beach lying in the sunshine; soon their bodies relax and tension melts away Other people believe (53) _ exercise helps the body handle the problems of daily life And for still others, joining with other people for games or parties helps them to become less anxious It (54) _very little which method we use to relax; what does matter is finding the time to temporarily reduce the effects of modern life on our (55) _ and body

41 A resist B stop C halt D control

42 A stress B pressure C anxiety D strain

43 A enable B bring C take D cause

44 A things B occasions C events D conditions

45 A draft B line C sketch D stripe

46 A by B with C to D on

47 A build B add C create D give

48 A experiment B survey C questionnaire D research

49 A come B cope C manage D confront

50 A emotional B anxious C frightening D stressful

51 A think B mediate C wonder D consider

52 A maintain B intend C pretend D decide

53 A strong B violent C powerful D strenuous

54 A matters B helps C determines D pays

55 A skull B mind C spirit D brain

II From the words given in the box, choose the most suitable word for each blank There are more words than blanks, so you don’t need all of them

control deficient devote generation lead to proposing proper scarce seek solution Food plays an important part in the development of nations In country where food is (56) _, people have to spend most of their time getting enough to eat This usually slows down progress, because men have little time to (57) to science, industry, government, an art In nations where food is plentiful and easy to get, men have more time to spend in activities that (58) _ progress, human betterment, and enjoyment of leisure The problem of providing good food for everybody has not yet been solved Many wars have been fought for the (59) _ of rich food-producing lands But it is no longer necessary to go into war for food Nations are beginning to put scientific knowledge to work for a (60) _ to their food problems They work together in the Food and Agriculture Organization of the United Nations (FAO) to help hungry nations produce more food

III Read the passage and choose the correct answer.

Another critical factor that plays a part in susceptibility to colds is age A study done by the

(82)

The general incidence of colds continues to decline into maturity Elderly people who are in god health have as few as one or two colds annually One exception is found among people in their twenties, especially women, who show a rise in cold infections, because people in this age group are most likely to have young children Adults who delay having children until their thirties and forties experience the same sudden increase in cold infections

The study also sound that economics plays an important role As income increases, the frequency at which colds are reported in the families decreases Families with the lowest income suffer about a third more colds than families at the lower end Lower income generally forces people to live in more cramped quarters than those typically occupied by wealthier people, and crowding increases the opportunities for the cold virus to travel from person to person Low income may also adversely influence diet The degree to which poor nutrition affects susceptibility to colds is not yet clearly, but an adequate diet is suspected of lowering resistance generally

61 Which of the following is closest in meaning to the word particulars?

A minor errors B specific facts C small distinctions D individual people 62 What does the author claim about the study discussed in the passage?

A It contains many inconsistencies B It contradicts the results of earlier studies in the field C It specializes in children D Its results apparently are relevant for the population as a whole

63 It may be inferred from the passage that which of the following groups of people is most likely to catch colds?

A Infants boys B Young girls C Teenage boys D Elderly women

64 There is information in the second paragraph of the passage to support which of the following conclusions?

A Men are more susceptible to colds than women B Children infect their parents with colds C People who live in a cold climate have more colds than those who live in a warm one D People who don’t have children are more susceptible to colds than those who 65 The author’s main purpose in writing the last paragraph of this passage is to A explained how cold viruses are transmitted B prove that a poor diet causes colds C discuss the relationship between income and frequency of colds

D discuss the distribution of income among the people in the study 66 The word cramped is closest in meaning to _.

A cheap B crowded C depressing D simple

67 The author’s tome in this passage could best be described as _

A neutral and objective B humorous C tentative but interested D highly critical

E WRITING

I A, B, C and D are four sentences which are written using the given words Choose the most appropriate sentence.

68 She/pack/ job/ go/ India A She packed a job and went to India. B She packed a job of going to India

C She packs her job as going to India D She’s packed her job in and has gone to India 69 You/ no obligation/ help/ assistance/ voluntary.

A You are under no obligation to help as assistance is purely voluntary B You have no obligation for help although assistance is voluntary

C You are under no obligation to help although assistances is purely voluntary D You have no obligation to help and assistance voluntarily

70 Jane/ need/ third/ food/ buy/ picnic.

A Jane needed one third of the food she bought for the picnic B Jane needed one third of the food buying for the picnic C Jane needed one third food she bought for the picnic D Jane’s need is one third food has bought for the picnic

71 two years/ age/ children/ regular/ produce/ sentences/ three/ four words.

A Two years of age, many children are regularly produced sentences containing three or four words B At around two years of age, many children regularly produce sentences containing three or four words C At around two years age, many children regularly produce sentences contain three or four words D Two years of age are children who regularly produce sentences containing three or four words 72 John/ do/ best/ stand/ chance/ win/ medal.

A John is doing his best, he doesn’t stand a chance of winning the medal

B Although John s doing his best, he doesn’t stand a chance of winning the medal C John is doing his best to stand by chance to win the medal

D John is doing his best, but he stands a chance of winning the medal

II For each of the following ,write a new sentence as similar as possible in meaning to the original sentences, using the word given in bracket These words must not be altered in any way Luyen Viet 176

73 Martin hasn’t mentioned the party to me at all (word) Martin hasn’t ……… about the party 74 Apparently, Sheila wasn’t listening to me (appear)

Sheila ……… listening to me

(83)

76 “I won’t trust Frank with your money if I were you, Carl” I said (advised)

I ……….Frank with his money

77 I don’t know Leslie’s reason for resigning (idea)

I ……….Leslie resigned

78 Have you any desks in stock which are cheaper than this?( desk)

Is this ……… in stock?

III Essay: Write an essay ( 150-200 words) about the following topic? Viet luan -111 In your opinion, which job is the most interesting? Why?

ĐỀ KHẢO SÁT HSG THPT KIM THÀNH LẦN NĂM HỌC : 2008-2009 MÔN: TIẾNG ANH

THỜI GIAN : 150 phút

A LISTENING: Listen to the passage about the Americans’ ways of offering gifts and check whether the statements are true (T) or false (F)

Statements T F

1 Almost every American offers gifs to their friends or relatives In the United States, gift-givers need not follow any rules Americans often bring a gift of money to a funeral

4 A gift of money given to low-income elderly pensioners is not acceptable A gift voucher to a popular store is often appreciated in the United States Voucher (n): phiếu trả tiền

B PHONETICS

I Choose the word whose underlined part is pronounced differently from those of the others.

6 A pleasure B sound C same D best

7 A dosage B voyage C massage D carriage

II Choose the word that has a different stress pattern from the others

8 A religious B miserable C perform D include

9 A significant B quantity C committee D believable

10 A ambitious B constant C investment D exhaust

C GRAMMAR AND VOCABULARY

I Choose the word or phrase which best completes each sentence Olympic 283 11 Men still expect their jobs to take _

(84)

12 I hope you won't take if I tell you the truth

A annoyance B resentment C offence D irritation

13 Workers who not obey the safety regulations will be immediately

A refused B disapproved C rejected D dismissed

14 _ the gold medal he’ll have to better than that

A Winning B In order win C To win D So that he wins

15 She many problems when she first start with this job

A entered B ordered C encountered D counted

16 The President visit the area to see the devastation

A on first hand B at first hand C on first hands D at first hands 17 Alice have told him how mad she was at him, but I'm not sure what she told him

A should B must C may D can

18 I need to find an apartment before I can move I can find one in the next week or so, I will move to Chicago on the first of next month

A Provided that B Even if C Only D If only

19 each school year all the children were given copies of the school rules

A At first B At the beginning of C Immediately D To start with

20 His speech was highly appreciate because it was short and the point

A round B in C within D to

II Give the correct form of the words in brackets

21 Lisa is very _ She always manages to look good in photographs PHOTO photogenic

22 Leonard has been very _these past few days I wonder what he is trying to hide from us," Debbie said SECRET secretive

23 I've never known such a person QUARREL quarrelsome

24 He wrote the book alone, so he doesn't have a AUTHOR co-author

25 I don't care if you had had too much to drink Your behavior last night was quite _ DEFEND indefensible

26 The Wright brothers' success the beginning of a new transport SIGNIFICANCE signified 27 The doctor gave him an injection to the pain DIE deaden

28 Grandfather rarely showed the affection he felt for his family He was very _person DEMONSTRATE undemonstrative

29 She is very efficient and polite to the customers FAIL unfailingly 30.Does this suit you? ARRANGE

III Choose the underlined word or phrase in each sentence that needs correcting Olympic 283 31 It was after shortly(A) microscopes were introduced(B) at the beginning(C) of the seventeenth century that microorganisms were actually(D) sighted

32 The Egyptians first(A) discovered that drying(B) fruit preserved it, made it sweeter(C), and improving(D) its flavours

33 As her focus(A) changed(B), the love poetry that Edna St Vincent Millay produced(C) in the 1920's increasing(D) gave way to poetry dealing with social injustice

34 Recently in the automobile industry, multinational companies have developed to the(A) point where such few(B) cars can be described as having been made(C) entirely(D) in one country

35 Scientists believe that by(A) altering the genetic composition of plants it is possible todevelop

specimens that are(B) resisting(C) to disease and have increased(D) food value ,

36 Methane in wetlands comes from(A) soil bacteria that(B) consumes(C) organic plant matter

37 Newtonian physics accounts(A) from(B) the observation(C) of the orbits(D) of the planets and moons 38 It is(A) extremely(B) important for(C) an engineer to know(D) to use a computer

39 Gothic Revival architecture has(A) several basis(B) characteristics that distinguish(C) it from other(D) nineteenth- century architectural styles,

(85)

D READING

I Read the following passage and decide which option A, B, C or D best fits each sentence.HSG 12-116

Over the past thirty years or so, the methods used for collecting money from the public to (41) _ the developing world have changed out of all recognition, along with the gravity of all the problems faced, and the increasing (42) _among the population that something must be done At the beginning of this period, it would have been common to put (43) in a collecting box, perhaps on the street or at the church, or to receive a small “flag” to wear in the lapel The 1960s saw the development of shops which sold secondhand goods, (44) _ by the public, and which also began to sell articles manufactured in the developing world in projects set up by the parent (45) _, to guarantee a fair income to local people The next development was probably the charity "event", in which participants were (46) to run, cycle, swim or what have you, and collected money from friends and relatives according to how far or long they managed to keep going The first hint of what was to become the most successful means of (47)

money was the charity record, where the artists donated their time and talent, and the (48) _from sales went to a good cause This was perhaps a reflection of the fact that young people felt (49) _

concerned about the obvious differences between (50 ) in Europe and the United States, and that in most of Africa and Asia, and this concern was reflected in songs, besides being clearly shown on television The problems were becoming hard to (51 ) but a feeling of frustration was building up Why was so little being done? The huge success of Band Aid, and subsequent televised concerts, reflected the (52) _of the media, and of music in particular, but also differed in style from other events People phoned up in their thousands on the day and (53) _money by quoting their credit card numbers After all, if you have enough money to buy CDs and a stereo player ,you can (54) something for the world's (55) children

41 finance B aid C pay D loan

42 .habit B wish C clamour D awareness

43 A this B money C them D funds

44.A donated B freed C offered D awarded

45 A government B concerned C charity D company

46 A sponsored B invited C required D used

47.A borrowing B such C further D raising

48.A change B means C proceeds D rest

49 A it B increasingly C less D this

50 A being B life C them D lifestyles

51 A avoid B understand C define D implement

52 A mass B ability C style D power

53 A loaned B handed in C pledged D raised

54 A waste B add C deposit D afford

55 famine B underdeveloped C starving D own

II From the words given in the box, choose the most suitable word for each blank There are more words than blanks, so you don’t need all of them

side beside raising included rush brings about limited scattering disaster nor

HOW TRANSPORTATION AFFECTS OUR LIVES

Without transportation, our modern society could not exist We would have no metals, no coal, and no oil (56) would we have any products made from these materials Besides, we would have to spend most of our time (57) food - and the food would be (58) to the kinds that could grow in the climate and soil of our own neighborhoods Transportation also affects our lives in other ways Transportation can speed a doctor to the (59) of a sick person, even if the patient lives on an isolated farm It can take police to the scene of a crime within moments of being noticed Transportation enables teams of athletes to compete in national and international sports contests In times of (60) transportation can rush aid to persons in areas stricken by floods, famines, and earthquakes

III Read the passage and choose the correct answer On HSG 12 -136

(86)

when he established the Broadside Press in 1965 Randall was a librarian and poet in Detroit when he began the Press with his personal savings as a way to copyright the words to his ballad about a 1963 racial incident in which Whites killed three Black children The poem was printed as a broadside

"By creating the Broadside Press- the most successful poetry institution in the history of African American literature- Randall created something that had previously not existed in the United States - an" organization that would publish the works of Black poets," explains Professor Melba Boyd, a poet and former Press editor Historically, work by Black poets had been criticized for emphasizing political issues and not using the traditional poetic forms of the White literary establishment Thus, Black poets had found it difficult to get published

Boyd is producing a film documentary that will present Randall's biography as well as his poetry Randall served as general editor of the Press from 1965 to 1977 In the mid-seventies skyrocketing printing costs and the closing of many small bookstores to whom he had extended credit left the Press in financial straits Randall then sold the Press and slumped into a depression, but in the 1980's, he revived community support for the Press through the Broadside Poets Theater Boyd hopes her documentary on Randall will introduce more people to African American literature

61 According to the passage, the Broadside Press is most famous as a publisher of _

A criticism of traditional White poetry B biographies of famous African American poets C poetry written by African Americans D African American documentaries

62 Who paid the cost to start the Press?

A An organization Black writers B Dudley Randall C Professor Boyd D Many small bookstores

63 According to Professor Boyd, what significant change occurred because of the Broadside Press? A Black poets returned to traditional poetic forms

B Historical works about African Americans began to appear in print C The Black literary establishment began to emphasize political issues D It became easier for Black poets to get their work in print

64 What happened to the Broadside Press in the B It moved into a different community C It regained popular support D It helped support small bookstores during a depression

65 What did the Broadside Poets Theater do?

A It helped get support for the Broadside Press B It led Randall into a personal depression C It led the Broadside Press into financial difficulties D It supported many bookstores in the community

E WRITING

I Choose the one word or phrase which best completes the sentence Olympic 283

66 During the early period of ocean navigation, …… any need for sophisticated instruments and techniques

A so that hardly B when there hardly was C hardly was D there was hardly

67 Salt is manufactured in quantities that exceed those of most, , other commercial chemicals

A of all not B not if all are C are not all D if not all

68 Mr Harrison is he owns many palaces

A so a rich man that B such rich a man that C so rich a man that D as rich a man that 69 Nebraska has floods in some years,

A in others drought B droughts are others C while other droughts D others in droughts 70 I should be grateful if you me know if you have any vacancies

A would let B would rather let C had let D have let

II A, B, C and D are four sentences which are written using the given words Choose the most appropriate sentence.

71 bill /pay/ immediate/ electricity / turn off

A The bill was paid immediately if the electricity to turn off

(87)

72 student/ not turn/ paper/ Friday/ fail/ test

A The student didn't turn in the paper on Friday to fail the test

B The student who wasn't turned in the paper on Friday to fail the test C Any student not turn in the paper by Friday will fail the test

D Any student who does not turn in the paper by Friday will fail the test 73 She/ discover/ eight/ new comets/ course/ work

A She discovered eight new comets of course of her work B She discovered eight new comets with the course of her work

C She discovered eight new comets in the course of (=during) her work D She whose discovery of eight new planets in the course of her work 74 Small/ flatter/ orange/ tangerine/ peel/ easy/ section/ separate/ ready

A Smaller and flatter than an orange, a tangerine peels easily and its section separates readily

B Smaller and flatter than an orange, the peel of a tangerine is easy removed and its sections are readily separated

C Smaller and flatter than an orange, it's easy to peel a tangerine and to separate its sections

D Smaller and flatter than an orange, to peel a tangerine is easy And its sections can be ready separated 75 the area/ desert/ many plants/ bloom/ springtime

A The area is a desert because many plants bloom in the springtime B Although the area is a desert, many plants bloom there in the spring time C Despite the area is a desert:, many plants bloom there in the springtime D The area is a desert with many plants bloom in the springtime

III Finish each of the following sentence in such a way that it means exactly the same as the sentence printed before it.

76 He said he was not guilty of stealing the car => He denied ……… 77 I'm sorry now that I asked her to stay => Now I wish

………

78 They couldn't trace who had supplied the information in the first place The source ………

79 There is always trouble when he comes to visit us => Whenever ………

80 Is this the only way to reach the city centre? => Isn't there ……… ? 81 He never suspected that the money had been stolen => At no time

………

82 I have never seen such a mess in my life! => Never in ……… ! 83 Is it essential to meet your aunt at the station? => Does your aunt ……… ? 84 My father speaks very little English => My father speaks hardly ……… 85 Although the play received good notices, not many people went to see it

Despite ……… III Essay

Write a paragraph of 200 words about one of your hobbies, following these guidelines  Name of your hobby

 How you enjoy it  Why you enjoy it  When you started it

 How much time you spend on it  How long you will keep it

ĐỀ KHẢO SÁT HSG THPT KIM THÀNH LẦN NĂM HỌC : 2008-2009 MÔN: TIẾNG ANH

(88)

A LISTENING Toefl -page 496-Part B question 31-33 Diana wants to change her

A class B work place C main subject D job

2 She would like to work in

A an art museum B any museum C business D a voluntary organisation

3 Professor Lane advises Diana to

A give up her job B give up learning C stay at home D consider her decision According to Mr Lane, good mangers are needed in _

A all organizations B private companies only C museums only D nonprofit foundations only Mr Lane suggests that Diana should

A take a few course in art history B work full time at the local art museum

C Both A and B D like her job

B PHONETICS

I Choose the word whose underlined part is pronounced differently from those of the others.

6 A exclamatory B exclusion C exclamation D excursion

7 A above B another C obesity D cover

8 A home B comb C tomb D dome

II Choose the word that has a different stress pattern from the others

9 A economical B personality C preparatory D entertainment 10 A original B straightforward C particularly D innocence C GRAMMAR AND VOCABULARY

I Choose the word or phrase which best completes each sentence.

11 When we fly to Texas, we’re going to _ over in Alaska to visit some friends

A stop B come C arrive D go

12 _ she’s got the job that she likes, she’s a lot happier

A Since when B Just as C Now that D Just now

13 I am _ aware of the need to obey the rules of the competition

A well B far C much D greatly

14 The candidate still expects to be selected the results of the latest opinion poll

A without B apart C nevertheless in spite of +Cl D notwith’standing = in spite of+ N

15 _ a fire, hotel guests are asked to remain calm

A As a result of B By the reason of C In the event of D In the time of 16 The football match was held in Hanoi Four _ police sent there to keep order

A dozen/ were B dozens/ weres C dozens of/ was D dozen of/ are

17 No sooner had the rain left than he _

A comes B had come C has come D came

18 Pythons lived in rugged tropical areas heavy rainfall and forests

A they have B that have C where the D have

19 Have you got some kind of on you?

A identification B unidentified C identify D identity

20 The photograph will show you

A what does our village look like B what our village looks like C how does our village look like D how our village looks like

II Give the correct form of the words in brackets Ex 16 Luyen Tu-page 46 He was given a light sentence because of his (YOUNG) youth

2 I am tired of your _behaviour When are you going to grow up? (CHILD) childish

3. _You could hardly think she has two girls She just

looks so young (TEEN) teenage (between 13 -19)

4. She divorced him because of his to the children (KIND)

unkindness

5. The evening was spent playing and talking (ENJOY) enjoyably

6. The keys were locked inside the car; , a side window

was open (LUCK) Luckily

7. He was turned down for the job because he

was (QUALIFY) unqualified

8. In parts of the country the situation is terrible As many as 20% of

the working population is without a job (EMPLOY) unemployment

(89)

Competitive

10. _The government is expected to take against the level

of unemployment (ACT) action

III Choose the underlined word or phrase in each sentence that needs correcting Olympic 142 31 The radio series(A) Woman's Hour has been(B) in the air(C) for years, and it's still(D) very popular

32 We could visit(A) the Railway Museum as well(B) because(C) it's only a stone throw(D) from here

33 I didn't know anyone at the party(A) and felt like(B) a fish from(C) water among all Jane's art college friends(D)

34 Scientists have been searching(A) for signs that mutations triggering(B) by the A bomb blasts, has passed on(C) to survivors' children(D)

35 She is considered(A) being(B) an expert on(C) animal behavior and often appears on television(D)

36 John died of(A) an attack heart(B) while he was proving(C) that he could run(D) the Marathon 37 Out of(A) a country's health budget, a large proportion(B) should divert(C) from treatment to spending(D) on health education and preventative measures

38 The independence(A) on(B) motor vehicles has given rise(C) to major problems, including environmental pollution(D), depletion of oil resources, traffic congestion and safety ,

39 I realized some day(A) that he hadn't been(B) to see me for several weeks and on enquiring(C) whether he was away, I was informed(D) that he was dying

40 The pressure on young people(A) performing(B) well at school in order to compete for work(C) is a possible(D) cause of the problem

D READING

I Read the following passage and decide which option A, B, C or D best fits each sentence HSG 12-70

When you are in Britain, you must never complain Complaining is very un-British If you are (41) _ waiting half an hour in a shop, if a bus (42) _ is rude to you, if the water brings you food ice-cold- you keep your mouth shut The (43) _ upper lip is the British way Other nationalities mightmake a (44) _, protest loudly or call for the manager, but not the British

Remember also that British ears are (45) not tuned to hear complaints A friend of mine was a regular (46) _ at a famous and expensive London restaurant Every day at 2p.m and 9p.m the (47) _ manager would come out (as he had been doing for the last 37 years), go from table to table and (48) _'Did you enjoy your meal?' For 37 years, hundreds of thousands of properly (49) _ up English people had replied to him: 'Very much indeed.' The man would smile, say 'Thank you very much', and (50) _ to the next table

One day, however, the lunch was so (51) _ that my friend (Dutch mother, Albanian father) decided to tell him the naked truth So, when the antiquated manager (52) _ at his table as usual and asked; 'Did you enjoy your meal, sir?' my friend replied: '(53) _, not at all It was appalling.' T0 which the manager

gave his (54) , obsequious smile, said: 'Thank you very much, sir', and moved on, quite (55) _

41.A made B kept C stayed D held

42

.A conductor B attendant C assistant D steward

43

.A hard B inflexible C firm D stiff

44

.A discussion B argument C quarrel D fuss

45

.A simply B easily C utterly D modestly

46

.A supporter B purchaser C customer D guest

47

.A mature B elderly C outdated D vintage

48

.A inquire B query C request D probe

49

.A raised B grown C educated D brought

50 A motion B progress C stride D shift

51 A offensive B painful C abominable D harrowing

(90)

53 A Sincerely B Largely C Bluntly D Frankly

54 A customary B average C commonplace D daily

55 A convinced B fulfilled C satisfied D complete

II From the words given in the box, choose the most suitable word for each blank There are more

words than blanks, so you don t need all of them test 15-188

alike as a result beneficial circulate create dose extent fortunately harmful include like prescribe relief sensation

All drugs can affect the body in both helpful and (56) ways For example, a particular drug may produce a stronger heartbeat, (57) _ from pain, or some other desired effect But that drug, (58) _any other

drug, can also cause undesired effects - especially if the (59) _is too large Such effects might (60) _fever, high blood pressure, or constipation

Most drugs produce changes throughout the body because the drugs (61) _through the blood stream (62) _ most drugs used to affect one part of the body also affect other parts For example, physicians sometimes (63) _morphine to relieve pain Morphine depresses the activities of cells in the brain and thus reduces the (64) _of pain But morphine also alters the function of cells elsewhere in the body It may decrease the rate of breathing, produce constipation, and (65) _ other undesired effects III Read the passage and choose the correct answer HSG 11-161

What is meant by the term economic resources? In general, these are all the natural, man-made, and human resources that go into the production of loads and services This obviously covers a lot of ground: factories and farms, tools and machines, transportation and communication facilities, all types of natural resources, and labor Economic resources can be broken down into two general categories: property resources - land and capital - and human resources - labor and entrepreneurial skills

What economists mean by land? Much more than the non-economist Land refers to all natural resources that are usable in the production process: arable land, forests, mineral and oil deposits, and so on What about capital? Capital goods are all the man-made aids to producing, storing, transporting, and distributing goods and services Capital goods differ from consumer goods in that the latter satisfy wants directly, while the former so directly by facilitating the production of consumer goods It should be noted that capital as defined here does not refer to money Money, as such, produces nothing

The term labor refers to the physical and mental talents of humans used produce goods or services (with the exception of a certain set of human talents, entrepreneurial skills, which will be considered separately because of their special significance) Thus the services of a factory worker or an office worker, a ballet dancer or an astronaut all fall under the general heading of labor

66 What is the author's main purpose in writing this passage? A To explain the concept of labor

B To criticize certain uses of capital

C To contrast capital goods and consumer goods D To define economic resources

67 In the second sentence of paragraph 1, the author uses the expression "This obviously covers a lot of ground " to indicate that

A the factories and farms discussed in the passage are very large B economic resources will be discussed in great depth

C the topic of economic resources is a broad one D land is an important concept in economics

68 When non-economists use the term "land", its definition A is much more general than when economists use it B is much more restrictive than when economists use it C changes from place to place

D includes all types of natural resources

(91)

70 The skills of the following could be considered examples of labor, as defined in the passage EXCEPT A artists and scientists

B workers who produce services, not goods C office workers

D entrepreneurs E WRITING

I Choose the sentence which is closest in meaning to the given one. 71 At no time did the two sides look likely to reach an agreement A The two sides had no time to reach an agreement

B The two sides never looked likely to reach an agreement

C If the two sides had had time, they would have reached an agreement D The two sides never look like each other

72 It won’t be long before her husband comes back A Her husband will be returning quite soon

B Her husband will come back later than planned C There's likelihood that her husband won't return soon D Possibly, her husband won't come back as soon as he wants

73 Impressed as we were by the new cinema, we found it rather expensive. A We were very impressed by the new' cinema, but found it rather expensive

B We were not impressed by the new cinema at all because it looked rather expensive C We weren't as much impressed by the new cinema's look as its cost

D The new cinema was more expensive than we expected 74 Hardly are appeals allowed against the council's decisions A The council always allows appeals against its decisions

B It's too hard for the council to allow appeals against its decisions C Allowing appeals against its decisions is not good

D The council rarely allows appeals against its decisions

75 There was not much resemblance between the final version and the final draft A The final version is very similar to the initial draft

B The final version is quite different from the initial draft C The final version and the initial draft resemble in many ways D The initial draft is exactly the same as the final version

II Finish each of the following sentence in such a way that it means exactly the same as the sentence printed before it HSG 11-126

76 The students regretted that they had missed the lecture

The students regretted not 77 If you miss the programme you can't really judge

Unless you see/watch 78 It was more of an argument than a discussion

It was not so much an discussion as an argument 79 I write to him almost everyday

Hardly a day passes by/goes by without me/my writing to him 80 I never intended to go to the meeting

I never had any intention of 81 I only recognized him when he came into the light

Not until :

82 That rumour about the politician and the construction contract absolutely false There is absolutely no truth in the rumor about

83 One runner was too exhausted to complete the last lap of the race One runner was so :

(92)

We were never made/forced/obliged to 85 It is quite pointless to complain

There’s no ………point/sense in

III Essay HSG 12 Test 6-page 141

Write a composition (1bout 300-350 words) about the following topic;

Some people prefer to eat at restaurants Other people prefer to prefer and eat food at home Which you prefer? Use specific reasons and examples to support your answer

Essay 1: How movies and television influence people s behavior? Use reasons and specific examples to support your ideas.

Use the following ideas as suggestions:

-make people more violent -make us less active -broaden our knowledge -help reduce stress

Do movies and television affect our behavior? A special concern is whether movies and television make children and society more violent I believe that movies and television influence our behavior, both for the better and for the worse

Movies make people more violent The more we see violent acts on television, the less sensitive we become to them Eventually violence doesn't seem wrong We may even commit violent acts ourselves This is especially true because we don't always realize that violence has consequences Actors can be killed and come back for another movie Sometimes we confuse that with reality We forget that killing someone is permanent

Movies and television also influence our behavior because they make us less active Looking at films is a passive activity If we watch too much, we become unhealthy, both mentally and physically We stop using our own imagination when we see things acted out for us Mental laziness becomes physical laziness; we'd rather watch sports on TV than play sports ourselves We'd rather visit with the characters on "Seinfeld" or "Friends" than go chat with our own neighbors Imaginary people have exciting lives.' Is it any wonder that some people would rather live a fantasy life than their own?

Of course, watching movies and television can also be good for us It can give us a broader window on the world For example, seeing movies can expose us to people of different races and cultures We can then overcome some prejudices more easily Recently there have been more handicapped people in films, and this also helps reduce prejudice

The best influence on our behavior is that movies and television reduce stress Watching films, we can escape our own problems for a little while Also, sometime movies show positive ways to resolve problems we all face While TV and movies shouldn't be a way to hide from life, sometimes they can help us cope

It is true that movies and television can influence our behavior negatively However, I also believe that they influence our behavior in positive ways How they affect you depends on how much you watch, what you watch, and how you respond to what you watch

Essay 2: Write an essay ( 150-200 words) about the following topic

In your opinion, which job is the most interesting? Why?

In my opinion, the choice of a career is of the utmost importance It determines one's whole life Therefore after careful consideration I prefer to become a tourist guide Being a tourist guide I can carry out the dreams I've fostered for years

First of all this job is suitable for my ability In the contact with foreigners I can regularly improve this practical foreign language think that only through long practice will I be able to drill my speaking and listening comprehension skills in English

Then through travelling, my knowledge is increasingly developed and broadened A

Vietnamese saying says: "You had better go for an excursion to know the places here and there Staying at home with your mother, you can never be wise" I'll know the country and the people thoroughly wherever I've set foot on

Then while guiding the tourists in their sight-seeing of beautiful landscapes and historical places of our country, I'll have a chance to tell them about our age-old culture and our courageous and unyielding struggles against foreign invaders

(93)

deep meaning in my heart!

For the above reasons, I will try my best to become a tourist guide In order to realize my dream, I have to try my best to improve my knowledge in all fields such as history, foreign language, communication skills, and so on

Essay 3: Write a composition (1bout 300-350 words) about the following topic;

Some people prefer to eat at restaurants Other people prefer to prefer and eat food at home Which you prefer? Use specific reasons and examples to support your answer

Some people like to eat out at food stands and restaurants, while others like to prepare food at home Often it depends on the kind of lifestyle people have Those with very busy jobs outside the house don't always have time to cook They like the convenience of eating out Overall, though, it is cheaper and healthier to eat at home

While eating in restaurants is fast, the money you spend can add up When I have dinner at a restaurant with a friend, the bill is usually over twenty dollars I can buy a lot of groceries with that much money Even lunch at a fast food stand usually costs five or six dollars for one person That's enough to feed the whole family at home

Eating at home is better for you, too Meals at restaurants are often high in fat and calories, and they serve big plates of food - much more food than you need to eat at one meal If you cook food at home, you have more control over the ingredients You can use margarine instead of butter or your potatoes, or not put so much cheese on top of your pizza At home you can control your portion size You can serve yourself as little as you want In a restaurant, you may eat a full plate of food "because you paid for it"

It's true that eating out is convenient You don't have to shop, or cook, or clean up But real home cooking doesn't have to take up a lot of time There are lots of simple meals that don't take long to make In fact, they're faster than eating out, especially if you think of the time you spend driving to a restaurant, parking, waiting for a table, waiting for service, and driving home

(94)

ĐỀ KHẢO SÁT HSG THPT KIM THÀNH LẦN NĂM HỌC : 2008-2009 MÔN: TIẾNG ANH

THỜI GIAN : 150 phút

A LISTENING: Listen to the passage and give short answers to the questions Where are the family?

2 How many children Marsha and Ed Gibson have? What are the boys’ parents telling the boys?

4 When is the father leaving the house? What the children want?

B PHONETICS

I Choose the word whose underlined part is pronounced differently from those of the others.

6 A insist B desire C preserve D resound

7 A lunar B rumour C ruler D pure

II Choose the word that has a different stress pattern from the others

8 A departure B dynamite C equipment D prevention

9 A auction B guidebook C party D resort

10 A chimpanzee B clinical C evidence D exercise

C GRAMMAR AND VOCABULARY

I Choose the word or phrase which best completes each sentence.

11 He cannot ignorance as his excuse; he should have known what was happening in his

own department

A insist B plead C refer - D defend

12 Visitors are to beware of pickpockets

A commanded B informed C notified D advised

13 He did not share his secrets with many people but he in her

A confessed B concealed C consented D confided

14 Before you can start a business, you will have to raise the necessary

A investment B income C savings D capital

15 As the drug took the patient became quieter

A force B influence C action D effect

16 He spoke _ to her, and she began crying immediately

A insensibility B insensitiveness C insensitive D insensitively

17 The mountaineers had to _ severe cold and high winds

A outlive B attain C go through D endure

18 The unpleasant smell in the restaurant _ me off my dinner

A set B put C sent D took

19 If he loses consciousness, give him a sip of brandy to bring him

A up B back C round D over

20 We have into your claim of wrongful dismissal but can find nothing to support it

A probed B looked C examined D investigated

II Give the correct form of the words in brackets Test 17-LUYen Tu

21 Our journey took nearly five hours It was much than I'd thought (FAR) farther/ further 22 Jeremy has just got _to a girl he's known since he was a boy (ENGAGE) engaged

23 Oscar had eaten so much that he had to his belt (LOOSE) loosen 24 The dictionaries are with the other _ books (REFER) reference

(95)

26 She bites her nails all the time It is a sign of (NERVOUS) nervousness 27 This coffee is too to drink (HEAT) hot

28 I really don't think he has the _ o this job (ABLE) ability

29 The professor explained his ideas with great (CLEAR) clarity/clearness

30 He rescued a child from drowning and was given a medal for his (BRAVE).bravery III Choose the underlined word or phrase in each sentence that needs correcting Olympic 111 31.That novel is definitely (A) a dense packed (B) narrative, but one which requires (C) a vast knowledge of cultural background or an(D) excellent encyclopedia

32 The doctor suggested that he lay (A) in bed (B) for several (C) days as a precaution against further damage (D) to the tendons

33 That (A) these students have improved (B) their grades because of their participation (D) in the test review class

34 The professor is (A) thinking to go (B) to the conference on aerodynamics (C) next month(D) 35 His father does not (A) approve of him to go (B) to the banquet without dressing (C) formally (D) 36 If it had not been (A) for the computerized register tape from the grocery (B)store I would never (C)have been able to figure on (D) expenditures

37 Our new office building will be located (A) downtown in the corner (B) of (C) Euclid Avenue and (D) East Ninth Street

38 The coach was depending for (A) his team to win (B) the game so that(C) they would have a chance to play in the Super Bowl(D)

39 Many(A)of the population in the rural areas(B) is composed of(C) manual laborers(D)

40 The(A) Department of Foreign Languages are(B) not located in(C) the new building opposite(D) the old one

D READING

I Read the following passage and decide which option A, B, C or D best fits each sentence 310 DH page 94 (T 118)

'PARTNER IN CRIME DID IT ALL FOR LOVE'

A property expert who was described as a man who would have had bright future if He had not fallen for a mysterious and beautiful woman, was (41) _ to five years in prison yesterday

Kevin Rogers, 39, of Hurst Road, Horsham, Sussex was employed by Selford Holdings, a London company with a broad (42) of investments in hotels, golf courses and shopping centers His downfall began when he met someone (43) _ as "La Contessa"

"I was completely in her (44) _ I did everything she asked me It didn't matter to me how dangerous it was or that I was (45) the law," he told the court

"La Contessa", whose real name was Pilar Gutierrez, (46) an agent for Rogers' employers She found property in Spain and Portugal that was about to be (47) down or needed extensive repairs and then (48) _ Selford Holdings to buy them The London company did not realize that the prices they were paying were considerably more than the real (49) of these properties: The original owners paid her very large (50) of money each time this happened

Rogers whose job it was to estimate how much the property was worth, admitted that he accepted a large (51) of the Gutierrez made in this way Rogers claimed that at first he (52) of what she was doing 'When I finally (53) _ through it all, I warned her not to it However, was already in love with her, so instead of telling my employers what was really going on, I became her partner in crime,' he said

Gutierrez, who had been (54) _ of accepting illegal payments of money and other crimes was last seen in England on the same day Rogers was arrested She is believed to be (55) somewhere in South America

41 A punished B sentenced C subjected D ordered

42 A range B width C extent D number

43 A referred B called C spoken D known

44 A command B strength C power D use

45 A breaking B cutting C smashing D crashing

46 A handled B behaved C treated D acted

47 A pulled B brought C put D taken

48 A advised B recommended C suggested D posed

(96)

50 A sums B numbers C totals D area

52 A sensible B sensitive C aware D apparent

53 A saw B looked C glimpsed D watched

54 A criticized B attacked C accused D doubted

55 A hiding B concealing C covering D escaping

II From the words given in the box, choose the most suitable word for each blank There are more

words than blanks, so you don t need all of them Test 22-Luyen DH (page 191)

As a result curious earliest in addition invented took like occupation pottery purchased raise record remains set up term

The first human beings probably lived about 2.5 millions years ago But man did not begin to (56) _ history until he had invented writing only about 5,000 years ago The period before man began to write is called prehistory, and the (57) prehistoric man refers to people who lived during that period.

Prehistoric man (58) _the first steps in building civilization The (59) people were all hunters In time, many hunters learned to plant crops and (60) animals for food, and they became farmers Prehistoric man (61) simple tools, and he discovered how to make fire He painted the first pictures and shaped the first (62) And he built and governed the first cities

Because early man kept no written records, scientists search for bones, tools, and other prehistoric (63) _ They study these objects to learn what early man looked (64) , how he lived, and how he developed

modern man Most of the tools that have been found and studied are made of stone (65) _ , the entire period during which early man lived has been called the Stone Age

III Read the passage and choose the correct answer 310 DH-page 192(Test 59)

Hatred is a fundamental emotion that has deep roots in society and culture Psychologists believe that group identity and cohesion depend on a age extent on having common enemy It seems that the existence of “bad guys” is important element in defining who we are within a larger realm It could be said that human beings love to hate,

The first signs appear early in life when a child, faced with blame for some mistake, immediately accuses another child or an inanimate object ad1 as a teddy bear Later, on the schoolyard playground, children in rival groups vie for attention and influence These basic responses translate into more powerful emotions later in life

One area where deep-rooted hatred is exhibited is in the ethnic clashes that constantly occur around the globe These conflicts are not only over territory but also involve emotional issues of group identity and unity of purpose For many, there is no 'us' without a 'them' to hate In a world where conflict between superpowers is on the decline, it may be that humanity will have difficulty adapting to a state of mutual respect and cooperation

66 Which of the following is the best title for the passage? A Roots of Society B Group Unity C A Basic Emotion D Social and Cultural Problems

67 According to the passage, what is believed to be an important aspect of defining group identity? A Facing a common enemy B Being reluctant to hate

C Accepting blame for past actions D Ignoring a larger realm

68 In paragraph 2, the word vie could best be replaced by which following?

A cooperate B compete C manipulate D defend

69 According to the passage, early childhood responses to blame

A are not related to stronger feelings in adulthood B are complex expressions of emotions C demonstrate how human beings love to hate D are not well understood by psychologists 70 The author suggests that when children make mistakes, they

A join rival gangs on schoolyard playgrounds B rarely accept responsibility for their actions C need emotional support from personal objects like teddy bears D readily admit to their errors

E WRITING

I Choose the best option to complete each sentence Olympic page 214 71 _ for overall health

A Extra fiber in one's diet is helpful B Extra fiber in one's helpful diet

C Helpful one's diet is extra fiber D One's diet is helpful extra fiber 72 _on the environment for the gratification of its needs

A Each organism to depend B All organisms depending

(97)

73 the fifth largest among the nine planets that make up our solar system A The Earth being B The Earth is C That the Earth is D Being the Earth 74 After seeing a movie based on a novel,

A the book was read by many people B the book made many people read it

C many people want to read the book D the reading of the book interests many people

75 Before _, they used horse - drawn wooden carts

A farmers have had tractors B tractors owned by farmers

C farmers had tractors D having tractors farmers

II Finish each of the following sentence in such a way that it means exactly the same as the sentence printed before it Luyen Viet page 179 (test 13)

76 John had not been to London before (visit) => It ………London 77 'I'll see you later, Anne,' he said (told) => He ………see her later 78 There were more students in school in 1991 than in 1992 (as)

There were in school in 1991 as in 1992

79 When I was in the department store, someone thought I was a shop assistant (me) When I was in the department store, someone a shop assistant 80 I wonder how she learnt to speak English so well (like)

I know how she learnt to speak English so well

81 Although he can't swim himself, Dan is very keen that his children should learn (unable) Despite himself, Dan is very keen that his children should learn

82 I am totally convinced that our team will win (chance) => In my opinion, there …….our team losing 83 It's a good thing you wrote the letter or we wouldn't have known what happened (you)

We wouldn't have known what happened ………… that letter.(interested)

84 Ballet doesn’t interested Sonia => Sonia ……… ballet

85 Visitors to the zoo are not allowed to feed the animals (must) => The animals … by visitors to the zoo

III A, B, C and D are four sentences which are written using the given words Choose the most appropriate sentence.

86 The students did not take to their new lecturer

A The students didn't understand what the new lecturer said

B The new lecturer was unpopular with his students

C The new lecturer didn't care his students D The students didn't follow the new lecturer

87 For such an experienced and able teacher, discipline was problem

A The teacher found it hard to observe discipline because she was experienced B Experience and discipline make her a good teacher

C With experience and ability, being a teacher was not a problem D For a teacher of experience and ability, discipline was not a problem 88 No matter how hard Fred tried to start the car, he didn't succeed

A Fred tried very hard to start the car, and succeeded B However hard Fred tried, he couldn't start the car C It's hard for Fred to start the car because he never succeeded

D Fred tried hard to start the car, and with success

89 Throughout his life, the fisherman suffered from great poverty.

A The fisherman was so poor that he died young B Although the fisherman was poor, he led a great life

C Poverty prevented the fisherman from enjoying life D The fisherman’s life was one of great poverty

90 There is not much to choose between the two essays.

A Both essays are great B One essay is just as bad as the other C One of the essay is optional D Either essay can be chosen

(98)

I Use the suggestions to write a compete letter. Dear Sir, ( page 60- LV)

1 I / be / present/ student/ final year/ City High School/ Harare/ Zimbabwe I / study/ English/ five years/ now

3 June / this year/ First Certificate examination/ University of Cambridge/ hope / pass/ good grade Also/ expect/ obtain/ High School Certificate/ about /same time/ high marks/ particularly/ biology Ambition/ enter/ London University due course/ read for/ degree/ genetics

6 Realize/ necessary / G.C.S/ qualify/ entrance / university

7 Very much/ hope/ accepted/ your College/ prepare/ G.C.E/ Chemistry/ Biology/ English too Would like/ start/ early autumn/ not/ waste/ time

9 Perhaps/ add father and mother/ both/ doctors/ so/ brought up/ scientific environment 10 Grateful/ you send/ official application form/ and inform/ other information /required Yours faithfully,

II Reading

give food necessary structures living store shape addition controls carry surrounds center

CELLS

Cells are the smallest part (or unit) of life All (1 ) things are made of cells, and all cells have the same parts: nucleus, cytoplasm, and membrane The nucleus is the (2) _ of the cell It contains the chromosomes and the genes The genes (3 ) the information that tells the cell what to

Chromosomes and genes help to make the cells The cytoplasm (4) _ the nucleus in the cell It contains several different things Different kinds of cells have different things in their cytoplasm The cell membrane is around the cell The membrane (5) -the flow of liquids in and out of the cell

Plant cells are different from animal cells In (6) _ to the nucleus, cytoplasm, and membrane, they contain a cell wall, vacuoles, and chloroplasts The cell wall is around the membrane It is made of cellulose, and it is not living It gives plant cells a (7) _like a box Vacuoles are round (8) in the cytoplasm A membrane surrounds them, and they contain water and other things The cell uses the vacuoles to (9) things They sometimes contain pigments These pigments (10) _ flower cells their colors Chloroplasts are also in the cytoplasm They contain a green pigment, chlorophyll, and they make (11) for the plant Chloroplasts are (12) _ for photosynthesis

(99)

returned ended freedom fought means racism MAHATMA GANDHI

Mahatma Gandhi was one of the great leaders of the world He wanted (1) and peace for all people He helped to free India from British control

His name was Mohandas Karacham, but people called him Mahatma This name ( 2) _ “ great soul” Mohandas Karacham was born on October 2, 1869 in Porbandar, India He studied law in London, and in 1893 he began to (3) law in South Africa

In South Africa, Indians did not have the same rights as others Gandhi did not like this (4) _ He believedthat everyone should have the same rights He worked for the Indian people in South Africa for twenty-one years At one time, he (5) a strike of Indian miners The Indian (6) _workers did not go to work Gandhi did not use violence He fought with nonviolence, resistance, and strikes The Indian word for nonviolent resistance is "satyagraha."

In 1915 Gandhi (7) to India He became the (8) of the Indian Nationalist Movement, and he worked for the independence of India He wanted India to be free from Great Britain Gandhi (9)

for freedom He did not want India to be a British (10) He continued to use satyagraha, nonviolent means such as strikes and fasting Gandhi's nonviolent fight for independence (11 ) _ when Britain gave India her independence in 1947 On January 30, 1948, Gandhi was assassinated in New Dehli-he was shot wDehli-hen Dehli-he was on tDehli-he way to a prayer meeting Gandhi died for peace, and tDehli-he whole world (12) him for his peaceful way of getting independence for his country

Olympic -258

Artists use caricature to distort the human face or figure for comic effect, while at the same time capturing an identifiable likeness and suggesting the essence of personality or character beneath the surface The humor lies in the fact that the caricature is recognizable, and yet exaggerated

From its origins in Europe as witty sketches, caricature grew through the eighteenth and nineteenth centuries, becoming enormously popular in the United States early in this century In the 1920s and 1930s especially, this lively form of illustration appeared in newspapers and magazines throughout the country The caricaturists in this era drew their portraits of important figures primarily to entertain In spirit their work was closer to the humor of the fast- developing comic strip and gag cartoon than to the sting of political satire Their subjects were more often amused than offended by their amiable attacks Which of the following words does NOT describe a typical caricature?

A humorous B distorted C topical D solemn

2 Producing a caricature is most like A singing a song B making a speech

C reciting a poem D telling a joke

3 According to the passage, the people who were caricatured were often

A angered B puzzled C entertained D shocked

4 Which of the following does the passage tell us?

A Caricature was popular in early twentieth century America B Caricature is common in times of stress

C Caricature evolved into a serious mode of self expression

D Caricature is one of the few art forms that originated in America The paragraph immediately following the passage probably discusses".- A the early history of American comedy films

B popular jokes of the twentieth century

(100)

I Use the suggestions to write a compete letter.

Dear Sir,

( page 60- LV)

1 I / be / present/ student/ final year/ City High School/ Harare/ Zimbabwe.

2 I / study/ English/ five years/ now.

3 June / this year/ First Certificate examination/ University of Cambridge/ hope / pass/

good grade.

4 Also/ expect/ obtain/ High School Certificate/ about /same time/ high marks/ particularly/

biology.

5 Ambition/ enter/ London University due course/ read for/ degree/ genetics.

6 Realize/ necessary / G.C.S/ qualify/ entrance / university.

7 Very much/ hope/ accepted/ your College/ prepare/ G.C.E/ Chemistry/ Biology/ English

too.

8 Would like/ start/ early autumn/ not/ waste/ time.

9 Perhaps/ add father and mother/ both/ doctors/ so/ brought up/ scientific environment.

10 Grateful/ you send/ official application form/ and inform/ other information /required

Yours faithfully,

II Reading

1

give food necessary structures living

store shape addition controls carry surrounds

center

CELLS

Cells are the smallest part (or unit) of life All (1 ) things are made of cells, and

all cells have the same parts: nucleus, cytoplasm, and membrane The nucleus is the

(101)

the information that tells the cell what to Chromosomes and genes help to make

the cells The cytoplasm (4) _

the nucleus in the cell It contains several different

things Different kinds of cells have different things in their cytoplasm The cell membrane

is around the cell The membrane (5) -the flow of liquids in and out of the cell

Plant cells are different from animal cells In (6) _ to the nucleus, cytoplasm, and

membrane, they contain a cell wall, vacuoles, and chloroplasts The cell wall is around the

membrane It is made of cellulose, and it is not living It gives plant cells a (7) _like a

box Vacuoles are round (8) in the cytoplasm A membrane surrounds them, and they

contain water and other things The cell uses the vacuoles to (9) things They

sometimes contain pigments These pigments (10) _ flower cells their colors

Chloroplasts are also in the cytoplasm They contain a green pigment, chlorophyll, and they

make (11)

for the plant Chloroplasts are (12) _ for photosynthesis

2

colony mine practice leader respects organize

returned ended freedom fought means racism

MAHATMA GANDHI

Mahatma Gandhi was one of the great leaders of the world He wanted (1)

and

peace for all people He helped to free India from British control

His name was Mohandas Karacham, but people called him Mahatma This name ( 2) _

“ great soul” Mohandas Karacham was born on October 2, 1869 in Porbandar, India He

studied law in London, and in 1893 he began to (3) law in South Africa

In South Africa, Indians did not have the same rights as others Gandhi did not like this (4)

_ He believedthat everyone should have the same rights He worked for the Indian

people in South Africa for twenty-one years At one time, he (5) a strike of Indian

miners The Indian (6) _workers did not go to work Gandhi did not use violence He

fought with nonviolence, resistance, and strikes The Indian word for nonviolent resistance

is "satyagraha."

In 1915 Gandhi (7) to India He became the (8) of the Indian Nationalist

Movement, and he worked for the independence of India He wanted India to be free from

Great Britain Gandhi (9)

for freedom He did not want India to be a British

(10) He continued to use satyagraha, nonviolent means such as strikes and fasting

Gandhi's nonviolent fight for independence (11 ) _ when Britain gave India her

independence in 1947 On January 30, 1948, Gandhi was assassinated in New Dehli- he was

shot when he was on the way to a prayer meeting Gandhi died for peace, and the whole

world (12) him for his peaceful way of getting independence for his country.

III Reading Comprehension

Artists use caricature to distort the human face or figure for comic effect, while at the

same time capturing an identifiable likeness and suggesting the essence of personality or

character beneath the surface The humor lies in the fact that the caricature is recognizable,

and yet exaggerated

From its origins in Europe as witty sketches, caricature grew through the eighteenth and

nineteenth centuries, becoming enormously popular in the United States early in this

(102)

satire Their subjects were more often amused than offended by their amiable attacks

Which of the following words does NOT describe a typical caricature?

A humorous

B distorted

C topical

D solemn

2 Producing a caricature is most like

A singing a song

B making a speech

C reciting a poem

D telling a joke

3 According to the passage, the people who were caricatured were often

A angered

B puzzled C entertained

D shocked

4 Which of the following does the passage tell us?

A Caricature was popular in early twentieth century America

B Caricature is common in times of stress

C Caricature evolved into a serious mode of self expression.

D Caricature is one of the few art forms that originated in America.

5 The paragraph immediately following the passage probably discusses".-

A the early history of American comedy films

B popular jokes of the twentieth century

C the change in America caricature in the 1940s

D how to develop skills as a caricature

ĐỀ KHẢO SÁT HSG THPT KIM THÀNH LẦN 10 NĂM HỌC : 2008-2009 MÔN: TIẾNG ANH

THỜI GIAN : 150 phút A LISTENING

1 What does the man plan to wear during the summer months?

A a cool hat B casual shoes C pans

2 What is one thing the man is NOT going to pack for the winter season?

A a coat B some sweaters C a jacket

3 What is the weather like in the spring?

A It’s windy B It’s rainy C It’s cool

4 What is an example of an occasion where the man might need formal clothes?

A A party B A business meeting C A wedding

5 What did the man wear to his high school graduation?

A Jeans and tennis shoes B A casual shirt and tie C a suit and dress shoes B PHONETICS

I Choose the word whose underlined part is pronounced differently from those of the others.

6 A stopped B landed C suited D wicked

7 A gypsy B huge C strange D piggy

II Choose the word that has a different stress pattern from the others

8 A involve B guidance C enhance D society

9 A arrange B nature C inherit D creation

10 A envelope B amazing C passenger D reference

C GRAMMAR AND VOCABULARY Olympic page 334 I Choose the word or phrase which best completes each sentence

11 I don't want lots of excuses, I just want to hear the _truth

A clear B plain C pure D right

12 I didn't want to discuss the matter but he insisted on bring it _

A up B out C about D over

(103)

A mess B stir C fuss D bother

14 Not until about a century after Julius Caesar landed in Britain actually conquer the island

A the Romans did B did the Romans C the Romans D Romans that

15 Mary is _ her sister

A as pretty but more intelligent as B as pretty but more intelligent than C more intelligent and prettier than D prettier and more intelligent than 16 Since the ditch is full of water, last night

A it must be raining B there might have rain C it must have rained D there might have been rain

17 _ our project was unnecessary, we would not have invested in it

A Had we thought B Unless has we thought C If had we thought D As we had ever thought

18 They stopped working on the project , they could not afford enough money

A therefore B as C because D for (“,” before)

19 Only after the atomic bomb and development in air travel _ _science fiction really become popular

A had created / had taken off/ was B had been created/ had been taken off/ has C had been created/ had taken off/ did D had been created/ had taken off / had 20 Primitive men had to suffer from scurvy, is caused by a lack of dietary Vitamin C

A that B which C from that D for which

II Give the correct form of the words in brackets Olympic page 272

21 The problem today is the (responsible) which the scientist has towards society Responsibility

22 The development of science has made many new and _ (fright) things possible frightening 23 Thousands of people could be destroyed by using only a few _ (fate) germs fatal

24 This leads to questioning the values of our present (moral) morality

25 Perhaps, we should now consider carefully the effects of present research on future (civil) cilvilization

26 The scientist is in a difficult position as his personal (repute) often results from successful experiments reputation

27 He is not responsible for himself only, but for the rest of (man) also mankind 28 My brother is working for a shampoo ( advertise) company in Hanoi advertising 29 The manager is seeking for ( depend) employees dependable

30 Only a/an (educate) person could behave like that It’s awful! uneducated

III Choose the underlined word or phrase in each sentence that needs correcting –Olympic page 290 31 When I (A) arrived at her house I found she (B) has been out with her (C) new fiend without (D) leaving me a note

32 (A)The new bridge (B) make possible to (C) cross the river (D) easily and quickly - 33 (A) I'm writing (B) in pencil so I can (C) rub out my mistakes (D) easy

34 (A) According to (B) a team of scientists, (C) there are evidence that Mount Everest is still (D) rising

35 (A) The passengers blamed the (B) airline (C) of the (D) delay

36 Insurance rates are not (A) the same for different people (B)because they are not likely (C) have the (D) same risk

37 The men (A) stole the (B) old woman(C) of her savings, and (D) stole $2,000 from her 38 (A) Can't you think (B) of (C) anything more intelligent (D) to tell?

39 In (A) today's competitive markets, even small businesses (B) had better to advertise on TV and radio in order (C) to gain (D) a share of the market

40 He saw (A) an empty shop (B) while (C) walks around (D) the town one day D READING

I Read the following passage and decide which option A, B, C or D best fits each sentence BDHSG12-137

(104)

The majority of those questioned said a good salary and career (43) were their main reason for working But (44) numbers did not believe their employers offered either In general, the survey found that most felt that (45) _ of life was more important than (46) _ and company perks Most would prefer employers to offer (47) _ hours, challenging tasks and job (48) rather than perks such as company cars and private health care Many employers’ (49) to understand this meant more than a third worried about their work on holiday, and 40 per cent took days off (50) _ when not ill

Workers were also (51) by the conditions they had to work in A fifth struggled with (12) technology, badly lit offices and chairs which caused backache Half said their (53) would increase if their environment improved

On the plus side, the biggest (54) was the friendship offered by colleagues, and it appears that the office also affords the chance to flirt with colleagues, make (55) calls to friends abroad, steal stationery and play computer games

41 A weight B force C heaviness D pressure

42.A published B printed C publicized D proclaimed

43 A outlooks B odds C prospects D views

44 A important B impressive C heavy D significant

45 A quality B calibre C excellence D worth

46 A rank B status C degree D grade

47 A pliable B elastic C amenable D flexible

48 A safety B security C sanctuary D protection

49 A failure B defeat C deficiency D lack

50 A indisposed B unwell C injured D sick

51 A pestered B inflamed C irritated D ruffled

52 A behind the times B expired C out-of-date D invalid

53 A fertility B capacity C value D productivity

54 A compensation B damages C reimbursement D atonement

55 A idiosyncratic B unique C personal D individual

II From the words given in the box, choose the most suitable word for each blank There are more words than blanks, so you don’t need all of them LD-Test 2

decay known found number discover ancient necessary important use means lived called

CARBON-14 DATING

There are many uses for radioactive carbon (C-14) One important (56 ) _ is in archaeology Archaeologists use radioactive carbon to (57) the age of ancient plants C-14 comes from the sun and is

(58) in all living things Archaeologists can calculate the age of plants that (59) in the past because radioisotopes (60) _ at the same rate The rate of decay is (61) the half-life The half-life is the time

(62) _ for half of the atoms to decay Radioactive carbon (C-14) has a half-life of 5,730 years That (63) _ that after 5,730 years, half of the radiocarbon atoms remain; one half of the radioactive atoms are still in the sample After 11,460 years, one-fourth of the atoms are still in the sample, and after 17,220 years one-eighth of the atoms remain Archaeologists can measure the (64) _ of radioactive atoms and discover the age of (65) plant material

III Read the passage and choose the correct answer Olympic –page 259

In its short history, the art of motion pictures has frequently undergone changes that seemed fundamental, such as that resulting from the introduction of sound It exists today in styles that differ significantly from country to country and in forms as diverse as the documentary created by one man with a hand- held camera and the multimillion- dollar "epic", involving hundreds of performers and technicians Despite its diversity, however, an essential unchanging nature can be discerned in most of its

(105)

experience For one thing, there is something mildly hypnotic about the illusion of movement that holds the attention and may even lower critical resistance Also, the accuracy of the motion picture image is compelling because it is made by a nonhuman, scientific process And, the motion picture gives what has been called a strong sense of being present: the film image always appears to be in the present tense There is also the concrete nature of film it appears to show the actual people and things No less important than any of the above are the conditions under which the motion picture ideally is seen, where everything helps to dominate the spectator He or she is taken from the everyday environment, partially isolated from others, and comfortably seated in an air- conditioned auditorium There, the darkness concentrates

attention and prevents comparison of the image on the screen with the objects or people around the viewer For a while, the motion picture unfolds the world in which the spectator lives

66 Which of the following statements best expresses the main idea of the passage? A Motion pictures vary greatly from country to country

B The fundamental characteristics of motion pictures remain unchanged C Motion pictures have been modified over the years

D The styles of motion pictures have become significantly different

67 Which of the following statements does the author NOT include as part of the motion picture experience?

A The viewing environment controls the spectator B The spectator is somewhat apart from other viewers C The viewer becomes fascinated by the action on the screen D The spectator realizes that events on the screen are false

68 Which one of the following activities is the nature of movies most comparable to?

A dancing at a party B drawing a picture C describing a painting D reading a book

69 The author probably believes that the main purpose of movies is to allow the audience to _ A see actual places, people or things B be entertained in a comfortable setting C escape from the real world D enjoy the results of a scientific process 70 Which of the following conclusion does the passage support?

A Motion projected on the screen captivates viewers B Viewers feel detached from the action on the screen

C The viewing environment is of minor importance D Documentaries and epics have universal appeal

E WRITING

I Choose the sentence among A, B, C and D which has the nearest meaning to the one given. Olympic page 291+ TNTANC page 245

71 When I was younger I used to go climbing more than I now

A Now I don't go climbing anymore B I used to going climbing when I younger C Now I don't go climbing as much as I did D I don't like going climbing anymore 72 I regret going to bed so late last night

A I wish I didn't go to bed so late last night B I wish I went to bed early last night C I wish I hadn't gone to bed so late last night D I wish I had gone to bed late last night

73 He got over his illness in three months A It took to get over his illness in three months B It took three months to him to get over his illness C It took him three months to get over his illness D It took three months for his illness to get over

74 She was so busy that she couldn't answer the phone A She was very busy that she couldn't answer the phone B She was too busy to answer the phone

(106)

D She was very busy so that she couldn't answer the phone 75 I didn't listen to him, and I didn't succeed

A If I listened to him, I would succeed B If I had listened to him, I would succeed

C If I listened to him, I would have succeeded D If I had listened to him, I would have succeeded

76 Although Christopher was the stronger of the two, his attacker soon overpowered him A Christopher's attacker was too weak to defeat him

B Christopher was weaker, but he overpowered his attacker C Christopher lost because he had no power

D Despite his superior strength, Christopher was soon overpowered by is attacker 77 You're under no obligation to accept their offer

A You're obliged to accept their offer B You're not obliged to accept their offer at all

C You can please yourself whether to accept their offer or not D B and C 78 My father finds maps hard to follow

A My father always gets lost B My father can't read maps at all

C My father has trouble following maps D Map-reading is not interesting to my father at all 79 You need not this job

A You have to this job B This needs to be done C This job needs not be done D You need this job

80 She has lost her appetite recently

A She hasn't had any food recently B Her appetite has been very good

C She has gone off food recently D She has eaten a lot of food recently

II Finish each of the following sentence in such a way that it means exactly the same as the sentence printed before it Olympic page 179+ 288

81 John was shocked when he heard his child swearing It came as a shock to John when he heard …

82 Attendance at the exhibition has been down this year.(attended) The exhibition has not been so well attended this year

83 Because the chemicals were properly combined, the prescription has brought about the unexpected result

=> Because of the proper combination of the chemicals, … 84 The man’s life was one of great poverty

Throughout his life, the man lived in / suffered from great poverty/ was very poor 85 Her success went beyond her expectation

Never had she expected that she was so successful 86 I would anything to prevent him from going there There’s nothing I wouldn’t to prevent

87 Tony’s very charming, but I wouldn’t trust him Charming as/though Tony is, I wouldn’t trust him 88 It was easy for us to find the house

We had no difficulty(in) finding the house

89 Your car might break down on the rough mountain road, so take some spare parts and a tool-kit In case your car breaks …

90 The students’ rebellious behaviors should have been severely punished The students deserved severe punishment for their

III Essay Olympic page 98

In about 200 words, write about the following topic:

“Some possible solutions to environmental damage.”

(107)

Jake: Well, not really I still have to buy some clothes Ashley: Well, what's the weather like where you're going?

Jake: Well, uh, it's really hot in the summer, so I'm going to buy some shorts, sandals, and a few T-shirts

Ashley: What about the rest of the year?

Jake: People say that the fall can still be warm until November, so I'm going to buy some jeans and a few casual shirts

Ashley: Will you need any warm clothes for the winter?

Jake: Well, the weather doesn't get too cold, but it often snows in the mountains, so I'm going to buy a couple of warm sweaters, a jacket, and a hat I don't have room in my suitcase to pack a coat, so I'm going to wait until I get there and buy it when I really need it

Ashley: Are you going to take anything else?

Jake: They say it rains cats and dogs in the spring, but again, I'll probably just wait and pick up a raincoat or an umbrella later on But, I'm going to take a good pair of shoes because I plan on walking to and from school everyday

Ashley: Do you need any clothing for formal occasions?

Jake: Well, you never know when you might needs something on the spur of the moment for a wedding or maybe someone's graduation, or a nice date, so I'll probably take some nice slacks, a dress shirt, and a couple (of) crazy ties or two

Ashley: Um, that makes sense

Jake: And I'll just rent a suit or tuxedo if I need anything more formal Hey, maybe I'll get married Ashley: You? Married? Hah!

Jake: Wait What are you trying to say?

Ashley: I just can't imagine you decked out in a tuxedo for any formal occasion [What?!] I mean, for high school graduation, you wore an old pair of jeans and tennis shoes

Jake: Hey, there was a reason for that, so let me explain [Yeah, ha, ha.] No, really You see, it goes like this

Olympic page 300

21 You shouldn't interrupt someone in _ (sentence) 22 A friend is one who likes power (dominate)

(108)

24 Mrs.Lan's shop is from the others in the street It's hard to tell the difference (distinguish) 25 Thieves dressed as police guards at a Boston museum (power)

26 The pipes in the house started rusting, the water (color) 27 His family suffered from his _ (expend)

28 You are entitled to receive a government allowance for each who is living with you (depend) 29 The oil spill has caused damage to the environment (calculate)

30 Please ensure that your child's sports clothes are clearly marked in _ ink (delete) ĐỀ KHẢO SÁT HSG THPT KIM THÀNH LẦN 11 NĂM HỌC : 2008-2009

MÔN: TIẾNG ANH THỜI GIAN : 150 phút

A LISTENING: Listen to the passage and give short answer to each question Listen to me –Unit 8-CD1

1 What is the weather like outside? Is it snowing?

3 How many people are sitting around the table?

4 The union has two demands: the first demand is asking for a 10% increase in pay What is the second demand ?

5 How many days has the strike lasted? B PHONETICS

I Choose the word whose underlined part is pronounced differently from those of the others.

6 A exaggerate B exist C anxiety D complexion

7 A little B whistle C gentle D battle

II Choose the word that has a different stress pattern from the others

8 A context B conference C confidence D consolidate

9 A product B postpone C purpose D postcard

10 A computer B competent C compatriot D museum

C GRAMMAR AND VOCABULARY

I Choose the word or phrase which best completes each sentence Olympic page 352 11 He has traveled inEurope

A effectively B exclusively C extensively D exhaustively

13 I like him very much He is

A so good a man B quite a good man C quite good man D A or B

14 Hadn’t you watched that movie last night, you _ sleepy now

A wouldn’t have been B might have not been C wouldn’t be D wouldn’t have being

15 The village had been flooded the year before so it _ plenty of green pasturage then

A contained B contains C has contained D would have contained

16 When he returned home from work, he found the door

A unlocking B unlocked C to be unlocked D have unlocked

17 They go to the seaside they should be disturbed by the noise of the city

A in order that B so that C for D lest

18 you discover his weakness, the easier the task should become

A Quicker B The more quickly C The fastest D The easiest

19.I wish I’d this calculator before I bought it It doesn’t seem to be working

A tried on B tried out C worked on D worked out

20 He said he was debt and asked me _ a loan _ $50

A on/ for/ of B in/ for/ with C on/ by/ of D in/ for/ of

II Give the correct form of the words in brackets Test 28-page 50_LTV

21 Third time lucky! After two _ attempts, Mark's finally passed his driving test (SUCCESS) 22 A newcomer is with the area around him or her (FAMI LIAR)

23 People usually have hobbies for (ENJOY)

24 They imagined that they made the rules, but in , they were mere puppets (REAL) 25 Street noise is one of the of living in the city (ADVANTAGE)

26 Studies show that smoking cigarettes is (HEALTH)

27 Some universities in the United States have dormitories (CO- EDUCATE) 28 A person is one who is kind (THINK)

29 There are various _ for the party (POSSIBLE)

30 The author gives three examples of the culture shock.(ORIENT)

(109)

32 Folding fans(A) arrived in Europe in the sixteenth century from China, where they had been(B) common(C) used for more than(D) 500 years

33 Volcanologists make(A) the jot of(B) classifying(C) volcanoes as(D) extinct, dormant, or active 34 People turn pale(A) when they are frightened(B)because the blood in our(C) cheeks is

directed(D) to the heart

35 Artificial(A) sweeteners known(B) as cyclamates banned(C) because of evidence that they caused cancer in laboratory(D) rats

36 An energy(A) source, liquid water, and organic molecules(B) consider(C) the three ingredients essential for(D) life

37 Every naturalized citizens(A) of the United States must be(B) at least(C) 18 years old(D) to vote 38 The opossum plays(A) dead(B), when(C) it is, in fact, live(D)

39 Writer Mark Twain was born(A) in 1835, the year of(B) Halley's Comet, and his(C) died in 1910, the year that(D) Halley's comet returned

40 Rubies and emeralds are like(A) because they both come from(B)the mineral corundum, which is also used to(C) make(D) sandpaper

D READING

I Read the following passage and decide which option A, B, C or D best fits each sentence TNTANC page 201 (Ex 11)

From the moment they leave the security of their accustomed environment, travelers are at risk (41) … arise not just from strange diseases they meet on their travel~ but from other factors, too: seemingly uninspiring home (42) such as safe water (43) …… , sanitation and public hygiene controls, legal safety standards for motor vehicles and road (44) … , are easily taken for granted, but simply not exist in many countries Environmental factors such as arduous conditions, (45) … climate, and high altitude may constitute a danger; and so may travelers' own behavior, free from the (46) of the daily routine, and determined to have a good time with scant (47) … for the consequences

When illness or injury occur abroad, travelers are again at a disadvantage- from (48)…… to communicate with a doctor on account of language or cultural difficulties, or being unable to find a doctor owing to (49)… of the (50) …… medical system There may be a complete (51) … of skilled medical care, or of medical facilities of a (52) …… acceptable to travelers from technologically sophisticated countries

When symptoms of an illness (53) … abroad not appear until after return home, a final hazard becomes apparent; the symptoms may be (54) … , may pass (55)… , and the correct diagnosis may not be considered until it is too late

41 A Complications B Changes C Hazards D Questions

42 A comforts B helps C cares D aids

43 A stores B collection C levels D supplies

44 A maintenance B correction C improvement D quality

45 A worsening B unusual C sudden D adverse

46 A restraints B assurances C certainties D regulations

47 A knowledge B awareness C regard D need

48 A inability B difficulty C inflexibility D timidity

49 A misuse B doubt C ignorance D disbeliefs

50 A local B district C neighborhood D area

51 A breakdown B failure C disruption D absence

51.A type B design C standard D degree

53 A received B formed C gained D acquired

54 A unfamiliar B unlikely C unpleasant D uncovered

55 A unrecognized B unknown C unforeseen D unearthed

II From the words given in the box, choose the most suitable word for each blank There are more

words than blanks, so you don t need all of them LD Test 24-page 193

after at convinced despite difficulty distracted dozes during falls from lose nods of perform problem

(110)

he (61) …… off for a few seconds or more He (62)…… completely asleep unless he is kept active continuously

Persons who go without sleep for more than three days have great (63) _ thinking, seeing, and hearing clearly They have periods of hallucinations, (64)…… which they see things that not really exist They also confuse daydreams with real life and (65) …….track of their thoughts in the middle of a sentence

III Read the passage and choose the correct answer Toefl page 272 (Q 1-12-Key 558)

The technology of the North American Colonies did not differ strikingly from that of Europe, but in one respect, the colonists enjoyed a great advantage Especially by comparison with Britain, Americans had a wonderfully plentiful supply of wood

The first colonists did not, as many people imagine, find an entire continent covered by a climax forest Even along the Atlantic seaboard, the forest was broken at many points

Nevertheless, all sorts of tine trees abounded, and through the early colonial period, those who pushed westward encountered new forests By the end of the Colonial era, the price of wood had risen slightly in eastern cities, but wood was still extremely abundant

The availability of wood brought advantages that have seldom been appreciated Wood was a foundation of the economy Houses and all manner of buildings were made of wood to a degree unknown in Britain Secondly, wood was used as a fuel for heating and cooking Thirdly, it was used as the source of important industrial compounds, such as potash, an industrial alkali; charcoal, a component of gunpowder; and tannic acid, used for tanning leather

The supply of wood conferred advantages, but had some negative aspects as well Iron at that time was produced by heating iron ore with charcoal Because Britain was so stripped of trees, she was unable to exploit her rich iron mines But the American Colonies had both iron ore and wood; iron production was encouraged and became successful However, when Britain developed coke smelting, the Colonies did not follow suit because they had plenty of wood and besides, charcoal iron was stronger than coke iron Coke smelting led to technological innovations and was linked to the emergence of the Industrial Revolution In the early nineteenth century, the former Colonies lagged behind Britain in industrial development because their supply of wood led them cling to charcoal iron

66 What does the passage mainly discuss? A The advantages of using wood in the colonies

B The effects of an abundance of wood on the colonies C The roots of the Industrial Revolution

D The difference between charcoal iron and coke iron 67 The word is closest in meaning to _

A realistically B dramatically C completely D immediately

68 Which of the following is a common assumption about the forests of North America during the Colonial period?

A They contained only a few types of trees B They existed only along the Atlantic seaboard C They had little or no economic value D They covered the entire continent

69 The word abounded is closest in meaning to .

A were present in large numbers B were restricted to certain areas

C were cut down D were cultivated

70 According to the passage, by the end of the Colonial period, the price of wood in eastern cities A rose quickly because wood was becoming so scarce

B was much higher than it was in Britain C was slightly higher than in previous years

D decreased rapidly because of lower demand for wood 71 Why does the author mention gunpowder in line 13?

A To illustrate the negative aspects of some industrial processes B To give an example of a product made with wood compounds C To remind readers that the Colonial era ended in warfare

(111)

A consulted B gathered C provided D restricted 73 The phrase "follow suit" means

A the same thing B make an attempt C have the opportunity D take a risk 74 According to the passage, why was the use of coke smelting advantageous?

A It led to advances in technology B It was less expensive than wood smelting

C It produced a stronger type of iron than wood smelting D It stimulated the demand for wood

75 The phrase "cling to" is closest in meaning to

A try to develop b avoid c continue to use D reconsider E WRITING

I Choose the sentence A, B, C and D which is closest in meaning to the one in italic Olympic 215 76 I can't help feeling worried about Tom

A I find it impossible not to worry about Tom B I can nothing to help Tom C I can glad to have someone like Tom to worry about D I don't worry about Tom 77 She told us to help ourselves to the apples in the basket

A She offered us some apples in return for our help

B She wanted our help to pick the apples and put them in a basket C She let us have as many apples as we wanted

D She wanted us to all the work ourselves 78 May I suggest Monday for our trip to Bristol?

A I think we must go to Bristol on Monday B I think we would be allowed to go to Bristol on Monday

C I think Monday is the only day we can manage for the trip D I think Monday would be a good day for us to go to Bristol 79 She usually spends one hour driving to work everyday A It usually takes her one hour to drive to work every day B She usually works one hour every day

C She does not usually drive to work

D It usually takes her one hour to work on her every day

80 They would never have accepted his money if they had known his plans A They knew what he wanted to do, so they refused his money

B They agreed with his wishes because they were glad to have his money C They didn't know his plans and never took money from him

D They took the money he offered them without realizing his purposes

II Finish each of the following sentence in such a way that it means exactly the same as the sentence printed before it LV page 135

81 Thanks to his aunt's legacy of$10,000 he was able to buy the house he wanted Had his ………

82 I didn't realize he was your brother until I saw the photograph It was only ………

83 The President is the statesman I admire most of all

There is

84 Don't you wish you could get out more in the evening? Don't you get fed …… ?

85 It's nobody's fault that the meeting was cancelled

Nobody :

86 He’ll settle down Then his improvement will improve Once ……….he settles/ has settled

87 She never seems to succeed, even though she works hard

However

88 It was more of an argument than a discussion It was not so

89 I write to him almost every day

Hardly

(112)

Mary reported III Essay

LD page 193-Test 25

accurately approaching departing despite far

fortunately guide instead of likely Long-range

owinf; to possible protect search vessels

Radar is an electronic device that can 'see' great distances (1) _ fog, rain, snow, and darkness It can find and (2) locate missiles, aircraft, ships, cities, rainstorms, and mountains Radar uses radio waves

(3) light waves, which the human eye uses in seeing This makes it (4) for radar to locate many kinds of objects at (5) greater distances than the eye can see

Radar became an important military device during World War II Today, networks of radar lookout stations guard the United States and Canada 24 hours a

day against (6) missiles and airplanes Patrol planes and ships (7) _ the oceans with radar for hostile ships and aircraft Airports use

radar to (8) planes safely to earth in fog or storms Ships use it to steer clear of other (9) _ or icebergs Radar helps weathermen warn of

(lO) hurricanes and tornadoes Exercise 29

Everyone was shocked It was a murder (SENSE) He felt very when he failed the exam again (COURAGE) That diamond is a antique (PRICE)

It gives me great to welcome our speaker (PLEASANT) He used all his to force the door open (STRONG)

He was too to tell his teacher about the stupid mistake (SHAME) The police are interested in the sudden of the valuable painting (APPEAR)

I believe you because I know you are (TRUE) I feel so that I'm going to bed (SLEEP)

10 The cost of must be paid by the buyer (CARRY) Exercise 30

I was late because how much time I needed (ESTlMA TE) He wrote the book alone, so he doesn't have a (AUTHOR) They're an incredibly family You should see how much food they throw away (WASTE)

You shouldn't leave things on car seats in full view of people walking past (VALUE)

I've never had the same opinions as my fathcr In fact when I was younger we were always having (AGREE)

Most people who work feel that they are (PA

I Essay : Neighbors are the people who live near us In your opinion, what are the

qualities of a good neighbor? Use specific details and examples in your answer.

(113)

are a lucky person You have someone who is respectful of your property, who is helpful

with the little day-to-day problems that arise, and who is supportive in times of

crisis.

A good neighbor respects your property This means she asks for your permission

before doing something that may affect you She does not plant a huge tree between your

houses without asking you how you feel about it If she wants to put up a fence, she tells

you about her plans first

A good neighbor is willing to lend a hand when you need a little help He lends you

some milk if you run out, or gives you a ride if your car breaks down He lets your children

stay at his house if you get stuck working overtime You the same for him Both of you

help make each other's lives easier

When you go through a crisis, like a death in the family, a good neighbor volunteers

to help in any way she can She might something small, like prepare a few meals for you.

Or, she might something big, like help you get through the sadness of a funeral

A neighbor can be as close as a good friend, or more like a distant acquaintance

Either way, a good neighbor is someone who respects you and supports you as he can We

should all be lucky enough to have good neighbors

II Interpreting the table about the number of total fires and acres burned in the USA

( 1980-2000)

The table summarizes the number of total fires and acres burned in the USA between

1980 and 2000 There were fewer fires in 1990 than in 1980, but more acres of forest were

destroyed The number of fires was almost the same in both years 1990 and 2000; however,

there was a big difference of nearly million acres burned - about 8.4 million in 2000 and

nearly 5.5 million in 1990

(Conclusion: Optional)

III Essay: Some possible solutions to environmental damage”

- The essay must have three parts

1 Introduction: must have a topic sentence (2,5)

2 Body: should have the following ideas

-Keep rare animals in their natural habitat

- Control the burning of forests

- Control the cutting down of trees

- Not use disposable products

- Use environment - friendly products

- Not build campfires in national parks

- Pass laws restricting environmental damage

- Treat sewage before discharging into the sea

- Pay heavy fine for people who break the law

- Use anti - pollution devices

- Change people's awareness

3 Conclusion: should express personal opinion of the author.

(114)

(Forest Cover in million hectares)

Country

Total Forest Cover

(1990)

Total Forest Cover

(2005)

Forest Loss

(1990-2005)

Cambodia

12.9

10.4

2.5 (19.3%)

Indonesia

116.5

88.5

28 (24%)

Laos

17.3

16.1

1.2 (6,7%)

The table shows the deforestation situation in Southeast Asia (Cambodia, Indonesia,

and Laos) between 1990 and 2005 In general, Laos had larger forest cover than Cambodia,

and Indonesia had the largest of the three, countries In 1990, there was a difference of

nearly 102 million hectares of forest cover between Cambodia and Indonesia - 12.9 million

in Cambodia and 116.5 million in Indonesia Meanwhile, the difference was about 99

million hectares between Laos and Indonesia Within 15 years (from 1990 to 2005), the

overall figures showed that Indonesia had the highest percentage of total change or forest

loss while Laos had the lowest - 24% in Indonesia and only 6.7% in Laos

Ngày đăng: 27/04/2021, 00:10

TỪ KHÓA LIÊN QUAN

TÀI LIỆU CÙNG NGƯỜI DÙNG

TÀI LIỆU LIÊN QUAN

w